Sie sind auf Seite 1von 91

Gastroenterology and Hepatology

Esophagus
The normal esophagus is a functional organ that is vital for actively transporting food from the mouth into the stomach and for protecting the airway from aspiration of gastric contents. The esophagus has three functional zones: the upper esophageal sphincter, the esophageal body, and the lower esophageal sphincter. The two sphincters are normally in a contracted state, which prevents the retrograde passage of gastric contents. They relax and remain open during swallows for easy passage of food into the stomach. Normal swallowing involves complex coordination of oropharyngeal and esophageal function, which is initiated by voluntary movements of the tongue that propel a food bolus into the pharynx. Involuntary relaxation of the upper esophageal sphincter and contraction of the pharynx (with closure of the larynx to prevent aspiration) safely move the bolus into the esophagus, where peristaltic movements actively transport food through the swallow-induced relaxed lower esophageal sphincter and into the stomach. In adults, the esophageal body, excluding the sphincters, is 18 to 22 cm long. The esophageal wall includes mucosa, submucosa, and muscularis propria. The mucosa consists of stratified squamous epithelium and joins gastric columnar mucosa at the lower esophageal sphincter. Unlike other regions in the gut, the esophageal wall has no serosa.

Symptoms of Esophageal Disorders


Careful attention to the patients presenting symptoms often leads to the correct diagnosis and helps to plan the appropriate diagnostic studies. Cardinal esophageal symptoms include dysphagia, odynophagia, heartburn, regurgitation, and sometimes chest pain. Dysphagia is defined as difficulty swallowing or food sticking and may be the presenting symptom in patients with mucosal inflammation (for example, esophagitis), mechanical obstruction (for example, stricture, tumor, Schatzkis ring), or an esophageal motility disorder (for example, achalasia or diffuse esophageal spasm) (Figure 1). Patients with mechanical obstruction usually report dysphagia predominantly for solids, whereas those with motility disorders have dysphagia for both solids and liquids. The sensation of dysphagia may occur in either the cervical or the substernal area. Cervical dysphagia suggests an abnormality in either the esophagus or the hypopharynx, whereas substernal dysphagia suggests an esophageal location. Odynophagia is a sensation of pain with swallowing and often signifies infectious esophagitis or an ulcerated esophageal lesion. Infectious causes are most frequently due to Candida albicans, herpes simplex virus, and cytomegalovirus. Odynophagia may also be caused by ulcerations resulting from acid-peptic disease or pill-induced lesions. Heartburn is a sensation of retrosternal burning that often moves into the chest and occasionally is accompanied by excessive salivation (waterbrash). Heartburn is a common symptom and is especially likely to be precipitated by a large meal containing fat, coffee, or alcohol. It usually develops within 30

Symptoms of Esophageal Disorders

Dysphagia

Barium Esophagogram

Solids Only (Mechanical) EGD

Solids and Liquids (Neuromuscular) Manometry

Intermittent

Progressive

Intermittent

Progressive

Chronic Heartburn

Weight Loss

Chest Pain

Chronic Heartburn Aspiration

Lower Esophageal Ring

Peptic Stricture

Carcinoma

Diffuse Esophageal Spasm

Scleroderma

Achalasia

FIGURE 1. Differential diagnosis of dysphagia and tests to evaluate dysphagia. In a patient with dysphagia, an initial evaluation by barium esophagography is very useful. In those with a history of dysphagia predominantly for solid foods (left side of the algorithm), upper endoscopy may be indicated because mechanical abnormalities are the most likely cause. Intermittent dysphagia, especially in a patient who eats rapidly, may suggest a lower esophageal ring (Schatzkis ring), whereas a patient with chronic heartburn may have a peptic stricture. Older patients with dysphagia for solid foods and significant weight loss may have esophageal cancer. Patients reporting dysphagia for both solid foods and liquids (right side of the algorithm) may have an esophageal motility (neuromuscular) disorder, which can be diagnosed by manometry. Patients with diffuse esophageal spasm often report associated chest pain and intermittent symptoms. Patients with progressive systemic sclerosis (scleroderma) due to reduced lower esophageal sphincter pressure and weak esophageal motility may develop chronic reflux with dysphagia or heartburn after dysphagia as a result of a weak esophageal muscle. These patients may also have dysphagia as a result of esophagitis, with or without strictures. Finally, patients with achalasia may report aspiration and regurgitation of undigested food products.
EGD = esophagogastroduodenoscopy (upper endoscopy)

minutes to 2 hours after a meal, may be associated with an acid or bitter taste, is made worse when the patient lies down or bends over, and may awaken the patient from sleep. Heartburn is usually caused by acid-peptic irritation of the esophagus, which results in esophageal mucosal damage (esophagitis) in 20% to 50% of patients. Heartburn is often accompanied by regurgitation, which is an effortless retrograde movement of gastric contents into the upper esophagus or the mouth. Both heartburn and regurgitation occur in normal persons, especially 2

Diagnosis of Esophageal Disorders

postprandially. Most people with heartburn have mild symptoms and tend not to seek medical attention until symptoms fail to improve or progress after selftreatment. However, heartburn and regurgitation can also be the classic presenting symptoms in patients with gastroesophageal reflux disease (GERD) and may also occur in patients with peptic ulceration of the stomach or duodenum and in those with delayed gastric emptying. Regurgitation of undigested food may also be a symptom of a pharyngeal pouch (Zenkers diverticulum) or of an esophageal or gastric outlet obstruction. Patients with esophageal disorders may occasionally have chest pain as the predominant symptom. Because of its retrosternal location, radiation, and often constricting quality, such pain may mimic angina. However, many of these patients do not have ischemic heart disease after undergoing cardiac evaluation and may therefore have an esophageal disorder. Noncardiac chest pain of esophageal origin may be due to GERD, an esophageal motility disorder, or increased esophageal visceral perception (increased sensitivity of the esophageal mucosa to gastric acid).

Diagnosis of Esophageal Disorders


Evaluation of patients with a suspected esophageal disorder often involves a detailed history, focusing on the cardinal esophageal symptoms, followed by selective diagnostic tests to confirm the diagnosis (Table 1). Both esophageal anatomy and function may be assessed. Upper endoscopy is a well-tolerated, safe, and frequently utilized study that is performed using conscious sedation. It allows direct visualization and sampling of the esophageal mucosa. Upper endoscopy is indicated for patients with dysphagia or odynophagia as well as for those with warning symptoms of malignancy. It is also the only reliable test for diagnosing esophagitis or Barretts esophagus in patients with a longstanding history of GERD. Manometry evaluates the contractile activity of the esophagus. It is commonly used to determine possible esophageal motility disorders in patients with dysphagia and normal findings on upper endoscopy. The pH probe must be positioned correctly, since accurate placement of this electrode requires knowledge of the location of the lower esophageal sphincter. Manometry is also used to rule out an esophageal motility disorder prior to surgical fundoplication. Ambulatory 24-hour esophageal pH monitoring quantitates esophageal exposure to gastric acid in patients with persistent typical or atypical symptoms
TA B L E 1 Diagnostic Tests for Esophageal Disorders

Test Upper endoscopy

Indications Dysphagia Odynophagia Longstanding GERD (rule out Barretts esophagus) Dysphagia (after normal upper endoscopy) Suspected esophageal motility disorder Verification of correct positioning of pH probe Preoperative evaluation for antireflux surgery Evaluation of suspected GERD Preoperative evaluation for antireflux surgery Evaluation of suspected oropharyngeal dysphagia Preoperative evaluation for antireflux surgery

Manometry

24-Hour ambulatory pH monitoring Video-pharyngoesophagoscopy

GERD = gastroesophageal reflux disease

Gastroesophageal Reflux Disease

of GERD but without evidence of esophageal mucosal damage. Ambulatory pH monitoring is also helpful in documenting abnormal esophageal gastric acid exposure prior to antireflux surgery and in assessing the effectiveness of medications or surgical procedures in patients with refractory symptoms of acid reflux. Contrast-enhanced radiographic studies assess esophageal structural abnormalities and function. When combined with fluoroscopy, radiographic studies can assess the extent and frequency of acid reflux as well as the effectiveness of esophageal clearance. Video-pharyngoesophagography is useful for assessing oropharyngeal and esophageal causes of dysphagia. It may also be used for preoperative evaluation of esophageal anatomy and function prior to surgical fundoplication.

Gastroesophageal Reflux Disease


How is the diagnosis of gastroesophageal reflux disease (GERD) usually established? What is the appropriate initial therapy for patients with GERD? When is upper endoscopy indicated for patients with GERD? What new endoscopic procedures are available for treatment of patients with GERD?

Locke GR 3rd, Talley NJ, Fett SL, Zinsmeister AR, Melton LJ 3rd. Prevalence and clinical spectrum of gastroesophageal reflux: a population based study in Olmstead County, Minnesota. Gastroenterology. 1997;112:1448-56. PMID: 9136821

GERD is one of the most common gastrointestinal disorders in Western industrialized countries. The most frequent mechanism is transient relaxation of the lower esophageal sphincter (prolonged relaxation of the sphincter that is not associated with a swallow and that results in exposure of the esophagus to gastric acid). In some patients, the presence of a large hiatal hernia and reduced lower esophageal sphincter pressure may be contributing factors. The primary symptom of GERD is heartburn, which occurs intermittently in almost 40% of otherwise healthy adults and daily in almost 10% of otherwise healthy adults in the United States (Locke et al.) Heartburn is caused by the reflux of gastric contents into the esophagus, which results in esophageal mucosal damage.

Diagnosis of Gastroesophageal Reflux Disease


Manifestations of GERD can be divided into three categories: typical symptoms, atypical symptoms, and complications (Table 2). Heartburn and regurgitation are the most typical and most specific symptoms. Approximately 20% to 50% of patients with these symptoms have endoscopic evidence of esophageal tissue injury (esophagitis) (Figure 2). Neither the pattern nor the severity of symptoms is a predictor of the presence of esophagitis. In patients with typical symptoms, the diagnosis of GERD is based on selfreported heartburn and regurgitation. Since almost all patients are treated empirically, upper endoscopy is usually not essential for differentiating erosive from nonerosive GERD. The diagnosis is often confirmed by the patients response to acid suppressive therapy. Upper endoscopy is indicated for patients with chronic GERD symptoms for more than 1 year in order to identify Barretts esophagus or for those with warning symptoms suggesting complicaTA B L E 2 Manifestations of Gastroesophageal Reflux Disease

Typical FIGURE 2. Upper endoscopy. Severe esophagitis without evidence of Barretts epithelium. (See Color Plate 1, at back of book.) Heartburn Regurgitation

Atypical Angina-like chest pain Asthma Chronic cough Chronic laryngitis Hoarseness Sore throat Globus Dental erosions

Complications Barretts esophagus Esophagitis (ulcerations and erosions) Stricture Adenocarcinoma Hematemesis and melena Anemia

Gastroesophageal Reflux Disease

tions of GERD (dysphagia, bleeding, weight loss, and choking). Diagnostic testing, including upper endoscopy and ambulatory 24-hour esophageal pH monitoring, is also indicated when patients suspected of having GERD do not respond to aggressive acid suppressive therapy. Patients with GERD may also have atypical extraesophageal symptoms. Because heartburn and regurgitation do not usually occur, the diagnosis is often delayed. The endoscopic finding of esophagitis is less common in patients with atypical GERD than in those with typical symptoms of acid reflux disease. Ambulatory 24-hour esophageal pH monitoring may be used to correlate the onset of a patients atypical symptoms with esophageal exposure to gastric acid. However, the finding of abnormal esophageal acid reflux does not confirm a causal association between atypical symptoms and GERD. Therefore, as in patients with typical symptoms, improvement in patients with atypical symptoms after aggressive acid suppressive therapy is the best way to document a causal association. Patients with GERD may also develop severe, sometimes life-threatening complications, such as marked erosive or ulcerated esophagitis, dysphagia secondary to peptic stricture, Barretts esophagus, or even carcinoma. Approximately 10% to 15% of patients with chronic heartburn and regurgitation will have endoscopic and histologic changes in the esophageal mucosa consistent with Barretts esophagus. The diagnosis of Barretts esophagus is usually suspected at upper endoscopy (Figure 3) and confirmed after biopsy specimens of esophageal epithelium show intestinal metaplasia. Patients with chronic GERD symptoms who are 50 years of age and older, especially white men in this age range, should undergo upper endoscopy to screen for Barretts esophagus. If Barretts esophagus is present, enrollment in an endoscopic surveillance program is appropriate (Sampliner).

FIGURE 3. Upper endoscopy. Barretts esophagus. (See Color Plate 2, at back of book.)

Treatment of Gastroesophageal Reflux Disease


Treatment options for patients with GERD range from lifestyle modification to gastric antisecretory drugs (H2-receptor antagonists, proton pump inhibitors) to surgical fundoplication (Table 3). The goals of therapy are complete relief of symptoms, healing of esophagitis, prevention of complications, and long-term maintenance of a disease-free state. GERD is a chronic disease, and most patients require lifelong therapy. Lifestyle changes, such as modifying eating habits, avoiding provoking foods, losing weight, and elevating the head of the bed, may effectively relieve mild or intermittent symptoms in about 25% of patients. However, the response is often partial and temporary and is frequently not much different from a placebo response. Acid suppressive therapy is the mainstay of medical therapy for GERD. Although H2-receptor antagonists are effective for many patients, the most rapid and complete relief of symptoms is achieved with proton pump inhibitors (Chiba et al.). These agents block the final pathway of acid release from the parietal cells via suppression of H+/K+adenosine triphosphatase. Patients treated with proton pump inhibitors have faster healing of esophagitis and resTA B L E 3 Effectiveness of Treatment of Gastroesophageal Reflux Disease

Sampliner RE. Practice Parameters Committee of the American College of Gastroenterology. Updated guidelines for the diagnosis, surveillance and therapy of Barretts esophagus. Am J Gastroenerol. 2002;97:1888-95. PMID: 12190150 Chiba N, De Gara CJ, Wilkinson JM, Hunt RH. Speed of healing and symptom relief in grade II to IV gastroesophageal reflux disease: a meta analysis. Gastroenterology. 1997;112:1798-810. PMID: 9178669

Treatment Placebo Lifestyle modifications H2-receptor antagonists Proton pump inhibitors Surgical fundoplication

Effectiveness (Symptom Relief) 20%25% 20%25% 50%60% 85%95% 85%95%

Esophageal Cancer

KEYPOINTS

Transient lower esophageal sphincter relaxation is the most common mechanism for development of gastroesophageal reflux disease (GERD). Heartburn and regurgitation are the most common presenting symptoms of GERD. Barretts esophagus is present in up to 10% of patients with GERD. Upper endoscopy is indicated for patients who remain at high risk for Barretts esophagus despite responding to antireflux therapy. Diagnostic tests are usually reserved for patients with GERD who do not respond to medical therapy or who require evaluation prior to surgical fundoplication. A proton pump inhibitor is the most appropriate initial therapy for patients with GERD. The goals of therapy for patients with GERD are complete relief of both acute and chronic symptoms and prevention of complications. Several new endoscopic procedures are available for treatment of patients with GERD. Only patients with complete resolution of typical or atypical reflux symptoms following medical therapy should be considered for surgical fundoplication.

olution of symptoms compared with those treated with H2-receptor antagonists. Proton pump inhibitors are safe medications. Concerns about complications resulting from long-tem use of these agents are decreasing after 15 years of careful follow-up studies. Although they are more expensive than H2-receptor antagonists, proton pump inhibitors are cost effective because they decrease the need for expensive diagnostic studies. Use of a proton pump inhibitor for 1 month with a subsequent change to an H2-receptor antagonist followed by discontinuation of therapy if symptoms do not recur (step-down therapy) is gaining popularity as the initial treatment for patients with GERD rather than initiating therapy with an H2-receptor antagonist followed by a change to a proton pump inhibitor in nonresponders (step-up therapy). Acid suppressive therapy for patients with atypical GERD symptoms should be more aggressive and of longer duration (3 to 4 months). Some patients who require long-term proton pump inhibitors for symptom control may prefer surgical fundoplication, especially since the advent of less invasive laparoscopic procedures. The best surgical candidates are patients who have GERD documented by esophagitis on upper endoscopy or by abnormal 24-hour esophageal pH monitoring and who have responded to medical therapy. Documenting a causal association between the presence of GERD and improvement of symptoms with use of proton pump inhibitors is especially critical for patients with atypical manifestations who are referred for surgical fundoplication. Esophageal manometry is important for all patients prior to fundoplication in order to document normal peristalsis and rule out other esophageal motility disorders, such as achalasia and progressive systemic sclerosis (scleroderma). Patients electing surgery should have the procedure performed by an experienced surgeon and must understand that new symptoms such as dysphagia, gas-bloat, increased flatulence, and diarrhea may occur postoperatively. The United States Food and Drug Administration has recently approved several new endoscopic procedures for treatment of GERD. These include endoscopic suturing, radiofrequency ablation, and injection of an inert polymer to bolster the lower esophageal sphincter and decrease gastroesophageal reflux. Studies show short-term effectiveness of these procedures for up to 1 year in association with improved control of symptoms and decreased need for proton pump inhibitors. To date, no placebo-controlled trials have been reported. Complications, including esophageal perforation and death, have been reported. Long-term clinical efficacy studies and cost analyses are not yet available.

Esophageal Cancer
What is the importance of Barretts esophagus in the development of esophageal cancer? What are the current recommendations for surveillance upper endoscopy in patients with Barretts esophagus? What is the most appropriate local staging modality for patients with esophageal cancer?

Esophageal adenocarcinoma is the most serious complication of GERD, and the incidence of this neoplasm has been rapidly increasing in the United States since the 1970s. It occurs most often in white men. Presenting symptoms include dysphagia, weight loss, and regurgitation (see Figure 1). Patients with

Esophageal Motility Disorders

locally invasive adenocarcinoma may have pulmonary symptoms such as cough as a result of aspiration or of tracheoesophageal fistula. Adenocarcinoma develops in patients with Barretts esophagus at a rate of 1 cancer/250 patient-years of follow-up, which is an annual incidence of almost 0.5%. Patients with Barretts esophagus typically develop progressively higher grades of dysplasia, which is a cytologic and architectural change that is the first step in the neoplastic process. Patients with Barretts esophagus require endoscopic surveillance in order to identify progression to high-grade dysplasia. Such surveillance results in improved long-term survival. Current recommendations for patients without dysplasia include surveillance upper endoscopy every 2 to 3 years with mucosal biopsy every 2 cm along the length of Barretts epithelium. More frequent studies are recommended for patients with low-grade dysplasia, and surgical esophagectomy is recommended for those with high-grade dysplasia in order to prevent adenocarcinoma. Research studies are currently focusing on ways to enhance identification of tumor markers that predict progression of Barretts metaplastic epithelium to high-grade dysplasia. These studies include flow cytometric analysis for the presence of chromosomal abnormalities, oncogenes, mutated tumor suppressor genes (for example, the p53 gene), and possible other factors. Unfortunately, most patients diagnosed with esophageal adenocarcinoma are not enrolled in endoscopic surveillance programs. Cancer is often first discovered at upper endoscopy after a patient presents with dysphagia and weight loss. Accurate staging of tumors is required to determine appropriate treatment options. Endoscopic ultrasonography with fine-needle aspiration and CT scans of the abdomen are important complementary staging studies. However, endoscopic ultrasonography is more accurate than CT scanning for measuring the depth of tumor invasion and detecting regional lymph node metastases. Patients with early-stage adenocarcinoma are offered surgical resection for cure, whereas those with late-stage tumor and metastases usually receive palliative treatment. Squamous cell carcinoma is also a frequent cause of cancer of the esophagus. It is more common in black persons than in white persons of the same age. Cigarette smoking and alcohol consumption are risk factors. Nutritional factors may also be important because of the geographic variation in the incidence of esophageal squamous cell carcinoma in the United States and worldwide.

KEYPOINTS

Esophageal adenocarcinoma most often develops in patients with Barretts esophagus. Identification of patients with Barretts esophagus and subsequent endoscopic surveillance to detect high-grade dysplasia result in improved longterm survival.

Esophageal Motility Disorders


What are the most common presenting symptoms in patients with an esophageal motility disorder? What diagnostic tests should be performed in patients suspected of having an esophageal motility disorder? What treatment options are available for patients with achalasia?
TA B L E 4 Classification of

Dysphagia and chest pain are the predominant symptoms in patients with esophageal motility disorders. Dysphagia for both solids and liquids typically occurs (see Figure 1) and may be associated with regurgitation or vomiting of the lodged food particles. Esophageal motility disorders are currently classified according to the abnormalities found during esophageal manometry (Table 4). Except for achalasia, the clinical significance of many of these abnormalities is yet to be determined.

Esophageal Motility Disorders Achalasia Diffuse esophageal spasm Nutcracker esophagus Hypertensive lower esophageal sphincter Hypotensive lower esophageal sphincter Ineffective or weak esophageal motility disorder

Esophageal Motility Disorders

Achalasia
Achalasia is an esophageal motility disorder characterized by poor relaxation of the lower esophageal sphincter and lack of coordinated peristalsis in the esophagus. The incidence of achalasia in the United States is 1/100,000 population per year. The etiology is unknown but is believed to be viral or autoimmune. Patients present with dysphagia for both solids and liquids associated with regurgitation of food or swallowed saliva. The diagnosis is suspected by the history and confirmed by either barium swallow (barium esophagography) or esophageal manometry. Barium swallow findings of a dilated esophagus with a smooth, tapered narrowing at the distal end (birds beak appearance) and to-and-fro movement in the esophageal body are classic for achalasia (Figure 4). Endoscopic evaluation of the gastric cardia is essential for all patients suspected of having achalasia in order to rule out tumors that could result in similar clinical, radiographic, and manometric findings (pseudoachalasia). Although there is an association between longstanding achalasia and esophageal squamous cell carcinoma, the incidence of this neoplasm is so low that endoscopic surveillance is not deemed cost effective for patients with achalasia. Pneumatic dilation and surgical myotomy are the most appropriate treatment options for long-term relief of symptoms in patients with achalasia (Vaezi and Richter). Neither is curative, but both are designed to relieve the distal esophageal obstruction caused by the dysfunctional lower esophageal sphincter. Both provide good long-term results. Pneumatic dilation is performed as an outpatient procedure and has a low perforation rate (<5%). A Heller myotomy may be performed laparoscopically. Endoscopic injection of botulinum toxin, a potent inhibitor of acetylcholine release, is reserved for elderly patients or those who cannot undergo pneumatic dilation or surgery. However, because the effectiveness of botulinum toxin wanes over time, long-term management requires repeated injections of toxin at various intervals. Calcium channel blockers or nitrates, both of which decrease lower esophageal sphincter pressure, are sometimes used in patients who do not respond to botulinum toxin, but results are usually not dramatic.

FIGURE 4. Barium esophagogram. Achalasia, as indicated by a dilated esophagus with retained barium and narrowing of the distal esophagus (birds beak appearance).

Diffuse Esophageal Spasm, Nutcracker Esophagus, and Hypertensive Lower Esophageal Sphincter
Diffuse esophageal spasm, nutcracker esophagus, and hypertensive lower esophageal sphincter are distinct spastic esophageal motility disorders. Patients often present with episodic chest pain and less often with dysphagia and are usually first evaluated for cardiac disease. The diagnosis is established by esophageal manometry, although a corkscrew appearance of the esophagus during barium swallow is also suggestive of diffuse esophageal spasm. Manometry shows more than 20% simultaneous contractions accompanied by peristalsis in patients with diffuse esophageal spasm and demonstrates high-amplitude peristaltic contractions in those with nutcracker esophagus. The cause of the chest pain is poorly understood. Some patients have associated GERD, whereas others respond to smooth muscle relaxants (calcium channel blockers and nitrates) or psychotropic agents. However, the overall response to these agents is poor. Ineffective esophageal motility disorder, defined as weak contractions of the esophageal body, as well as reduced lower esophageal sphincter pressure, may be associated with GERD, Barretts esophagus, and progressive systemic sclerosis (scleroderma).

KEYPOINTS

Chest pain and dysphagia for solids and liquids may be the presenting symptoms in patients with an esophageal motility disorder. Esophageal manometry establishes the diagnosis in patients suspected of having an esophageal motility disorder. Achalasia is most often treated with pneumatic dilation or surgical myotomy.

Vaezi MF, Richter JE. Diagnosis and management of achalasia. Am J Gastroenterol. 1999;94:3406-12. PMID: 10606295

Infectious Esophagitis and Pill-induced Esophagitis

Infectious Esophagitis and Pill-induced Esophagitis


What is the most common infectious cause of esophagitis? Which medications most commonly cause pill-induced esophagitis? Which diagnostic study should be done first when evaluating a patient with odynophagia?

Candida albicans is the most frequent cause of infectious esophagitis. Infections due to cytomegalovirus and herpes simplex virus are also common. A primary esophageal infection seldom occurs in an otherwise normal individual. Most often, it develops in patients who are immunosuppressed as a result of HIV infection, prolonged neutropenia after cancer therapy, organ transplantation, or administration of immunosuppressive medications. The frequency of infectious esophagitis may be decreasing in the United States as a result of the widespread use of antimicrobial prophylaxis for cytomegalovirus infection in transplant recipients and combination antiretroviral therapy for patients with HIV infection. Patients with infectious esophagitis usually report odynophagia. Symptoms of Candida esophagitis vary from mild discomfort to severe pain with swallowing. The presence of oral candidiasis (thrush) may help establish the diagnosis, since oropharyngeal candidiasis is often associated with esophageal involvement. However, the absence of thrush does not exclude esophageal disease. Upper endoscopy with biopsy is the most sensitive and specific test for diagnosing esophageal candidiasis (Figure 5). Oral therapy is the mainstay of treatment for Candida esophagitis and typically involves administration of systemically active oral azoles, such as fluconazole or itraconazole. Herpes simplex virus esophagitis and cytomegalovirus esophagitis occur with equal frequency in transplant recipients. Esophagitis due to herpes simplex virus is relatively uncommon in patients with HIV infection. The diagnosis is often suspected at upper endoscopy and confirmed by esophageal brushings, biopsy, or viral culture. Herpes simplex virus esophagitis is treated with oral acyclovir (a nucleoside analog). If the virus is resistant to acyclovir, foscarnet may be used. Cytomegalovirus esophagitis is treated with intravenous ganciclovir, foscarnet or, more recently, cidofovir. Patients with pill-induced esophagitis may also have odynophagia or dysphagia. Pill-induced esophagitis is twice as common in women as in men and affects persons of all age groups. The most commonly incriminated drugs are antibiotics (tetracyclines), nonsteroidal anti-inflammatory drugs, bisphosphonates (alendronate and risedronate), potassium chloride, quinidine, and ferrous sulfate. Most patients have normal esophageal structure and function. Factors favoring retention of pills include advanced age of the patient, decreased esophageal motility, extrinsic esophageal compression, and use of gelatin capsules and large pills. Pills are more likely to stick in the esophagus if taken without water or while supine. Retained pills or capsules dissolve in the esophagus. If the retained medication is caustic, the esophageal mucosa is injured, which may cause ulceration or stricture. The most common site of injury is the junction of the proximal and middle thirds of the esophagus, where esophageal peristalsis is weak and the aortic arch is compressed. Prevention of pill-induced esophagitis requires enhanced esophageal transit of swallowed pills. Patients should therefore drink at least 4 ounces of fluid with any pills and avoid the supine position for at least 30 minutes post-pill ingestion. In addition, patients with a known esophageal disorder should avoid pills that are caustic to the esophagus.

FIGURE 5. Upper endoscopy. Candida esophagitis. (See Color Plate 3, at back of book.)

KEYPOINTS

Odynophagia is the most common symptom in patients with infectious esophagitis. Candida albicans is the most common cause of infectious esophagitis. Antibiotics, nonsteroidal anti-inflammatory drugs, bisphosphonates, calcium chloride, quinidine, and ferrous sulfate are common causes of pill-induced esophagitis. All patients with odynophagia require upper endoscopy.

Dyspepsia and Non-Ulcer Dyspepsia

Stomach and Duodenum


Dyspepsia and Non-Ulcer Dyspepsia
How are dyspepsia and non-ulcer dyspepsia defined? Is there currently one treatment that is effective for most patients with non-ulcer dyspepsia?

Case 1 A 42-year-old woman has a 3-year history of mild to moderate epigastric pain without radiation. The pain is episodic and is sometimes, but not always, related to stress. Antacids and overthe-counter H2-receptor antagonists provide minimal relief. She has not had weight loss, heartburn, or vomiting, although she does have occasional nausea. Abdominal ultrasonography, which was ordered by her gynecologist, is normal. Physical examination and routine laboratory studies are normal. Dyspepsia describes a heterogeneous group of upper gastrointestinal symptoms and can include epigastric fullness, postprandial pain and discomfort, belching, bloating, nausea, and food intolerance. The patient in Case 1 most likely has non-ulcer dyspepsia (or functional dyspepsia), which is a diagnostic term that is usually used after patients with dyspeptic symptoms have normal routine evaluations (physical examination, routine laboratory studies, upper endoscopy, abdominal ultrasonography). The term probably encompasses various distinct gastric pathophysiologic abnormalities, such as Helicobacter pylori infection, gastric dysmotility, gastroparesis, impaired postprandial fundic accommodation, delayed gastric emptying, and visceral hypersensitivity. However, the relationship of these abnormalities to symptoms is uncertain. Psychosocial factors may also play a role. Detection of most pathophysiologic defects requires relatively invasive tests, such as gastric scintigraphy or gastric motility studies, neither of which is widely available. Testing for these abnormalities (other than routine diagnostic studies) is therefore not warranted in routine clinical practice. Therapy for dyspepsia due to peptic ulcer disease is discussed below. Currently, no therapy has been proved to be completely effective for non-ulcer dyspepsia. Acid suppressive therapy using proton pump inhibitors may lead to complete relief of symptoms in 25% to 50% of patients (Bytzer and Talley). The prokinetic agents cisapride and domperidone are possibly effective in a small number of patients with non-ulcer dyspepsia but are not currently available in the United States. The 5-HT4 agonist tegaserod is currently undergoing clinical testing for treatment of non-ulcer dyspepsia. The efficacy of 5-HT3 antagonists (alosetron, granisetron, and ondansetron) for treating patients with this disorder has not been demonstrated, and the usefulness of agents that affect gastric accommodation (such as 5-HT1 agonists and nitric oxidereleasing agents) requires further study. Tricyclic antidepressants are widely used for treating functional gastrointestinal disorders despite a lack of large, controlled studies of their effectiveness. Psychological treatment, such as cognitive-behavioral therapy, may be helpful in selected patients. In any patient with non-ulcer dyspepsia, treatment is empirically directed toward the principal symptoms. If the primary symptom is dyspepsia, antisecretory therapy should be given. Simethicone should be administered for belching and gas, and prokinetic agents such as metoclopramide can be prescribed for symptoms of gastroparesis, although adverse effects can occur. Patients who do not respond to empiric therapy should be tested for H. pylori infection, even though only a few patients will have improved symptoms after eradication of this organism.

Bytzer P, Talley NJ. Dyspepsia. Ann Intern Med. 2001;134:815-22. PMID: 11346316

KEYPOINTS

Non-ulcer dyspepsia is a diagnostic term used to describe a disorder that has various presenting symptoms and is possibly due to several different underlying pathophysiologic mechanisms. Treatment of non-ulcer dyspepsia is empiric and is directed toward the most prevalent presenting symptom.

10

Peptic Ulcer Disease

Peptic Ulcer Disease


What are the major risk factors for peptic ulcer disease? What are the typical presentations in patients with uncomplicated peptic ulcer disease? What are the complications of peptic ulcer disease? When should a patient with dyspepsia be referred for early upper endoscopy? Which noninvasive screening test is preferred for determining the presence of H. pylori infection?

Approximately 90% of patients with peptic ulcer disease have H. pylori infection or are taking a nonsteroidal anti-inflammatory drug (NSAID), or both, at the time of diagnosis. Other less frequent causes of peptic ulcer disease are listed in Table 5. Factors postulated to influence the development of peptic ulcer disease, such as cigarette smoking, alcohol consumption, corticosteroid administration, and psychological stress, do not appear to be independent risk factors for this disorder in the absence of NSAID exposure or H. pylori infection. Although many foods and beverages cause dyspepsia, there is no good evidence that any specific food causes peptic ulcer disease or that a bland or dairy-rich diet is useful in its treatment. H. pylori is the most common cause of peptic ulcer disease and is currently found in 70% to 80% of patients with duodenal ulcers and in approximately 50% of those with gastric ulcers. However, the association between H. pylori and peptic ulcer disease is decreasing in industrialized nations, most likely because of the declining prevalence of H. pylori in these countries. The best evidence for H. pylori as the cause of peptic ulcer disease is the dramatic effect that eradication of this organism has on recurrence of duodenal ulcers (from an annual recurrence rate of 70% to 80% without eradication to a rate of less than 10% with eradication). H. pylori is less frequently associated with gastric ulcers than with duodenal ulcers, partly because a significant proportion of gastric ulcers are caused by ingestion of NSAIDs.

TA B L E 5 Causes of Peptic Ulcer

Disease Very Common Helicobacter pylori infection Nonsteroidal anti-inflammatory drugs Less Common Malignancy Stress ulcerations Viral infections (herpes simplex virus type 1, cytomegalovirus) Uncommon or Rare Zollinger-Ellison syndrome Cocaine-induced ulcers Crohns disease Systemic mastocytosis Myeloproliferative disorders with basophilia Idiopathic (nonH. pylori) hypersecretory ulcers Radiotherapy to the abdomen Hepatic artery infusion of 5-fluorouracil

Patient Presentation Complicated and Uncomplicated Peptic Ulcer Disease


From a management perspective, it is important to determine whether a patient has an uncomplicated or a complicated ulcer. Patients with uncomplicated peptic ulcers usually have abdominal pain that is typically epigastric, rhythmic, and described as burning, sharp, or gnawing. The pain often occurs in the middle of the night. Food and antacids usually provide temporary relief, and patients may report a history of self-treatment with antacids and H2-receptor antagonists. Physical examination usually discloses epigastric pain, which is the most frequent finding in patients with an uncomplicated peptic ulcer. However, this finding is neither sensitive nor specific for diagnosing this disorder. Because peptic ulcer disease is often asymptomatic, a patient may first be evaluated because of a complication. Complications include bleeding, perforation, obstruction, and penetration and are usually more prevalent in elderly patients. Bleeding ulcers are commonly manifested by melena or hematemesis. Rarely, peptic ulcers may bleed chronically, so that patients present with iron deficiency anemia or occult blood in the stools. Perforated ulcers are characterized by severe, sudden abdominal pain that is often associated with shock. Peritoneal signs (fever, abdominal pain, rebound tenderness, and preferential flexion of the hips and knees) are usually present on physical examination. The primary symptoms of obstructed ulcers are nausea, early satiety, and vomiting that typically occurs 30 to 60 minutes after meals. At upper endoscopy,

11

Peptic Ulcer Disease

TA B L E 6 Alarm Symptoms

(Indications for Early Endoscopy) in Patients with Dyspepsia New-onset symptoms after age 50 years Anorexia Dysphagia Gross or occult gastrointestinal bleeding Unexplained anemia Unintentional weight loss Significant vomiting Upper gastrointestinal barium study suspicious for cancer

obstructed ulcers are usually visualized in the pyloric channel and duodenal bulb, which are areas where the gastrointestinal lumen naturally narrows. On physical examination, patients may have a succussion splash that is caused by retained fluid within a distended stomach. Penetrating ulcers extend into adjacent organs, most commonly the pancreas. The clinical presentation of penetrating ulcers is similar to that of uncomplicated ulcers except that the pain is generally more severe and more persistent in the former.

Management of Peptic Ulcer Disease


Most patients with uncomplicated peptic ulcer disease present with dyspepsia or other upper gastrointestinal symptoms. The differential diagnosis includes nonulcer dyspepsia, gastroesophageal reflux disease (GERD), biliary tract disease, pancreatitis, and cancer. GERD is usually suggested by the classic symptoms of substernal heartburn and acid regurgitation. If GERD is suspected and no other significant findings or alarm features are present (Table 6), antisecretory therapy is indicated. If biliary tract or pancreatic disease is suspected, liver and pancreatic blood tests and abdominal ultrasonography should be obtained first (Figure 6). If additional findings suggestive of a gastric malignancy or an ulcer complication are present, upper endoscopy should be performed. Any abnormalities found on upper endoscopy should be treated specifically. If the patient

Yes

Consider whether symptoms are classic for GERD without alarm features

Dyspepsia

Consider whether symptoms are likely to be biliary or pancreatic disease

Yes

Empiric therapy with an antisecretory drug

No

Alarm Features Present or Age > 50 years

No

Liver / pancreatic blood tests and abdominal ultrasonography

Yes Endoscopy

No Stop NSAIDS Hp Serology Positive Negative Symptom-directed Therapy

Positive

Negative

Treat Specific Disease

Symptom-directed Therapy

Treat for Hp

Symptoms Resolve

Symptoms Persist

Symptoms Persist

Symptoms Resolve

Endoscopy
FIGURE 6. Algorithm for evaluation of dyspepsia. Symptom-directed therapy is discussed in the text.
GERD = gastroesophageal reflux disease; Hp = Helicobacter pylori; NSAIDs = nonsteroidal anti-inflammatory drugs

12

Helicobacter pylori Infection

has dyspepsia without alarm features, all NSAIDs should be stopped and a serologic test for H. pylori should be performed. If H. pylori infection is not present, therapy for non-ulcer dyspepsia is begun (see earlier discussion). Symptoms that persist after completion of empiric therapy should be evaluated by upper endoscopy. If endoscopic findings are positive for H. pylori, treatment of H. pylori infection is begun. Repeat upper endoscopy is warranted if symptoms persist following completion of H. pylori therapy. Patients with complications of peptic ulcer disease require urgent attention. If perforation is suspected clinically and free air in the abdomen is demonstrated radiographically, urgent referral for surgery is usually warranted. If symptoms are present that suggest malignancy or major upper gastrointestinal bleeding, early upper endoscopy is indicated. Whether surgical intervention is needed for obstructed ulcers is typically determined after several days of conservative management that includes gastric decompression by nasogastric tube and administration of antisecretory agents to assess whether ulcer edema resolves and obstructive symptoms subside. The need for surgery is confirmed by upper endoscopy.

KEYPOINTS

Approximately 90% of patients with peptic ulcers are infected with H. pylori or are taking nonsteroidal anti-inflammatory drugs, or both. Complications of peptic ulcer disease include bleeding, obstruction, perforation, and penetration. Empiric therapy for non-ulcer dyspepsia should be initiated for patients with peptic ulcers who do not have concomitant H. pylori infection.

Helicobacter pylori Infection


Which diseases are most frequently associated with H. pylori infection? Should patients with suspected H. pylori infection undergo upper endoscopy as the initial diagnostic study? What is the first-line and second-line therapy for treatment of H. pylori infection?

As noted earlier, H. pylori infection is the most common cause of peptic ulcer disease. Most H. pylori infections are acquired in childhood, usually persist throughout life, and resolve spontaneously in only a small number of persons. In adults, the prevalence increases with increasing age, which is a cohort effect as a result of a higher infection rate among children born in the first half of the 20th century than among those born in the second half of the century. The prevalence of H. pylori infection is higher in lower socioeconomic groups and in Hispanics, blacks, and Asians. The mode of transmission is probably by the fecaloral or oraloral route or by the gastricoral route as a result of vomitus.

Diseases Associated with Helicobacter pylori Infection


In addition to peptic ulcer disease, H. pylori infection may be associated with several other gastrointestinal disorders, including gastritis, non-ulcer dyspepsia, gastric cancer, GERD, and mucosa-associated lymphoid tissue (MALT) lymphoma. Although a causal relationship has been suggested for H. pylori and some extraintestinal conditions, including coronary artery disease, recent data do not suggest a relationship between H. pylori infection and nongastrointestinal disorders.

Gastritis
Approximately 95% of patients with H. pylori infection have an associated asymptomatic, antral-predominant gastritis consisting primarily of infiltrates of polymorphonuclear cells and lymphocytes. In some patients, the fundic mucosa may become atrophic and metaplastic, and intestinal epithelium may be present. Patients with this type of gastritis are at increased risk for gastric ulcers and cancer. However, most people with H. pylori gastritis never develop peptic ulcer disease or cancer.

13

Helicobacter pylori Infection

Veldhuyzen van Zanten SJ, Talley NJ, Blum AL, Bolling-Sternevald E, Sundin M, Junghard O. Combined analysis of the ORCHID and OCAY studies: does eradication of Helicobacter pylori lead to sustained improvement in functional dyspepsia symptoms? Gut. 2002;50(Suppl 4):iv26-30; discussion iv31-2. PMID: 11953343 Walker MM. Is intestinal metaplasia of the stomach reversible? Gut. 2003;52:1-4. PMID: 12477745

Non-Ulcer Dyspepsia
Whether or not H. pylori causes dyspepsia is controversial. Meta-analyses of randomized trials of patients with non-ulcer dyspepsia who were treated for H. pylori infection suggest that symptoms may resolve in approximately 10% of these patients after therapy (Veldhuyzen van Zanten et al.).

Gastric Cancer
Patients with H. pylori infection may develop atrophic gastritis and intestinal metaplasia in the stomach. Up to 10% of patients with long-term chronic antral gastritis, atrophic gastritis, and intestinal metaplasia will develop gastric cancer of the intestinal type. The risk of gastric cancer is much higher for patients with gastric ulcers than for those with duodenal ulcers. Endoscopic biopsy to exclude cancer should therefore follow a radiologic diagnosis of gastric ulcer; however, biopsy is not usually needed for a diagnosis of duodenal ulcer. Gastritis improves after eradication of H. pylori. Whether this leads to regression of gastric atrophy and intestinal metaplasia has not been determined definitively, as several, but not all, studies have shown improved findings after H. pylori eradication (Walker). Eradication of H. pylori is unlikely to result in complete resolution of gastric atrophy and intestinal metaplasia or in a reduced risk of gastric cancer. However, H. pylori eradication may reduce the frequency of metachronous tumors in patients with early gastric cancer who have undergone partial gastric resection.

Gastroesophageal Reflux Disease


Recent epidemiologic studies suggest that the cytotoxin-associated gene A (cagA) strain of H. pylori, when found in patients with corpus gastritis, may be protective against the development of severe esophagitis and Barretts esophagus (Vaezi et al.; Queiroz et al.). This may occur because corpus gastritis inhibits acid secretion by the parietal cells, making less acid available for reflux.

Vaezi MF, Falk GW, Peek RM, Vicari JJ, Goldblum JR, Perez-Perez GI, et al. Cag-A positive strains of Helicobacter pylori may protect against Barrett esophagus. Am J Gastroenterol. 2000;95:2206-11. PMID: 11007219 Queiroz DM, Rocha GA, Oliverira CA, Rocha AM, Santos A, Cabral MM, et al. Role of corpus gastritis and cagA-positive Helicobacter pylori infection in reflux esophagitis. J Clin Microbiol. 2002;40:284953. PMID: 12149341 Fischbach W. Helicobacter pylori eradication therapy in primary high-grade gastric MALT lymphoma. Gastroenterology. 2002;123:39. PMID: 12105882 Steinbach G, Ford R, Glober G, Sample D, Hagemeister FB, Lynch PM, et al. Antibiotic treatment of gastric lymphoma of mucosa-associated lymphoid tissue. Ann Intern Med. 1999;131:8895. PMID: 10419446

Mucosa-associated Lymphoid Tissue (MALT) Lymphoma


H. pylori infection has been reported in up to 90% of patients with low-grade mucosa-associated lymphoid tissue (MALT) lymphomas. MALT lymphomas, which constitute a subset of non-Hodgkins lymphomas, are low-grade clonal neoplasms of B lymphocytes that arise from lymphoid aggregates in the lamina propria. The normal stomach does not contain organized lymphoid tissue. However, H. pylori infection may incite development of lymphoid tissue that often forms aggregates and follicles from which MALT lymphomas may arise. Approximately 50% of patients with MALT lymphomas have complete regression of their tumors after H. pylori eradication, and about 33% have partial regression (Fischbach; Steinbach et al.). Patients who have normal-appearing gastric mucosa on upper endoscopy, tumor confined to the mucosa on histologic studies, and no lymph node involvement (tumor stage 1, T1, N0) are much more likely to achieve complete remission with antibiotics than are patients with gastric ulcers, gastric masses, or submucosal, serosal, or nodal disease. Patients with more advanced disease should be considered for initial treatment with traditional forms of therapy for gastric lymphoma.

Diagnosis of Helicobacter pylori Infection


Nonendoscopic studies are preferred for the initial diagnosis of H. pylori infection. These include serum antibody studies, urea breath tests, urea blood tests, and stool examination for H. pylori antigen. Blood, serum, and saliva can be analyzed for the presence of IgG antibody to H. pylori. Qualitative in-office antibody tests are also available. Because antibody test results may continue to 14

Helicobacter pylori Infection

be positive for months after successful eradication of infection, they are not recommended for determining H. pylori status after treatment. Urea breath tests or urea blood tests involve the oral administration of urea in which nonradioactive 13C or radioactive 14C has been substituted for 12C. Expired breath or venous blood can be collected for measurement of labeled carbon dioxide. These tests diagnose active infection and may be used before or after treatment. False-negative results may occur in patients who recently took antibiotics, bismuth-containing compounds, proton pump inhibitors, or possibly H2-receptor antagonists. Patients should not take proton pump inhibitors for at least 2 weeks before undergoing a urea breath test or urea blood test for assessment of H. pylori. The various tests for H. pylori infection are shown in Table 7. Follow-up testing after treatment is not generally required but is advised for patients with a history of an ulcer complication.

Treatment of Helicobacter pylorirelated Ulcers


Selected agents approved by the United States Food and Drug Administration for treatment of H. pylori infection are listed in Table 8. The eradication rates for these agents, based on data from controlled clinical trials, are also shown. Bismuth-based triple therapy (usually bismuth, metronidazole, and tetracycline) eradicates H. pylori in approximately 80% of patients enrolled in clinical trials. The triple regimen is usually combined with an H2-receptor antagonist or a proton pump inhibitor. Bismuth-based triple therapy has been less effective in clinical practice because of the complexity of the regimen, the large number of tablets required, and the frequent development of side effects. The chance of successful H. pylori eradication is reduced in patients who take 60% or less of the prescribed medications. Proton pump inhibitors do not eradicate H. pylori infection when given alone. Combining a proton pump inhibitor with one, or preferably two, antibiotics (for example, clarithromycin plus amoxicillin) provides eradication therapy. Optimal treatment is likely to require a three-drug combination given twice daily for 1 to 2 weeks. In clinical practice, the success rate of first-line therapies for eradication of H. pylori is at best 70% to 80%. Re-infection following cure of H. pylori infection is reported in less than 2% of patients each year. Therefore, persistent H. pylori infection after attempted eradication is more likely a failure of therapy than of development of re-infection. There is currently a high prevalence of H. pylori resistant to clarithromycin (approximately 10%) and metronidazole (approximately 35%) in the United
TA B L E 7 Characteristics of Commonly Used Tests for Detection of Helicobacter pylori Infection

Test Non-endoscopic tests Antibody test (serum) Antibody test (whole blood) Enzyme-linked immunosorbent assay (serum) Stool antigen test Urea breath test Urea blood test Endoscopic tests* Urease test (biopsy) Histology Culture
*Cost of test does not include cost of endoscopy.

Sensitivity

Specificity

Approximate Cost

88%94% 67%85% 86%94% 94% 90%96%

74%88% 75%91% 78%95% 92% 88%98%

$10$30 $10$30 $40$100 $50$100 $250$300 $20$100

88%95% 93%96% 80%98%

95%100% 98%99% 100%

$6$20 $60$250 $150

15

Gastropathy Induced by Nonsteroidal Anti-inflammatory Drugs

TA B L E 8 FDA-Approved Combination Regimens for Treatment of Helicobacter pylori Infection in Patients with Peptic Ulcer

Regimen* Omeprazole, 40 mg qd, plus clarithromycin, 500 mg tid, for 2 weeks followed by omeprazole, 20 mg qid for 2 more weeks Lansoprazole, 30 mg tid, plus amoxicillin, 1000 mg tid, for 2 weeks Ranitidine bismuth citrate, 400 mg bid, plus clarithromycin, 500 mg tid, for 2 weeks followed by ranitidine bismuth citrate, 400 mg bid for 2 more weeks Bismuth subsalicylate, 525 mg qid, plus metronidazole, 250 mg qid, plus tetracycline, 500 mg qid (Helidac) for 2 weeks. A standard-dose H2-receptor antagonist or a proton pump inhibitor should be started at the same time and continued for a total of 4 weeks. Lansoprazole, 30 mg bid, plus clarithromycin, 500 mg bid, plus amoxicillin, 1000 mg bid, for 2 weeks
FDA = Food and Drug Administration; qd = daily; bid = twice daily; tid = 3 times daily; qid = 4 times daily *The first two regimens are not recommended because of the low success rate. Ranitidine bismuth citrate is no longer available in the United States.

Effectiveness 64%74% 70% 73%84% 77%82%

88%

KEYPOINTS

Disorders commonly associated with H. pylori infection include gastritis, non-ulcer dyspepsia, gastric cancer, gastroesophageal reflux disease, and mucosa-associated lymphoid tissue (MALT) lymphomas. Nonendoscopic studies such as serum antibody testing are preferred for the initial evaluation of a patient with suspected H. pylori infection. The preferred treatment for H. pylori infection is a 14-day three-drug combination regimen. Most patients do not need a follow-up study to evaluate the effectiveness of H. pylori eradication.

States. Clinical studies of susceptibility testing show that successful H. pylori eradication after initial failure can be achieved in up to 75% of patients by using second-line therapies consisting of a proton pump inhibitor plus amoxicillin plus either clarithromycin or metronidazole for 14 days (Suerbaum and Michetti). Susceptibility testing is usually not done in clinical practice. If a patient has a history of prior exposure to either clarithromycin or metronidazole, the subsequent treatment regimen should not contain the previously used antibiotic.

Gastropathy Induced by Nonsteroidal Anti-inflammatory Drugs


What complications typically occur from use of nonsteroidal anti-inflammatory drugs (NSAIDs)? How can these complications be prevented?

Suerbaum S, Michetti P.Helicobacer pylori infection. N Engl J Med. 2002;347:1175-86. PMID: 12374879

Mild degrees of topical gastric mucosal bleeding and erosions commonly develop in up to 80% of patients who use traditional nonsteroidal anti-inflammatory drugs (NSAIDs) such as indomethacin and aspirin. NSAID gastropathy typically is not associated with clinically significant consequences. Asymptomatic ulceration may occur in up to 40% of users of traditional NSAIDs. However, serious gastrointestinal adverse events such as ulceration associated with perforation, pain, or bleeding occur in 1% to 4% of NSAID users. Because of the widespread use of NSAIDs by persons in industrialized nations, this 1% to 4% risk translates to a significant number of serious ulcer complications and deaths from these drugs annually.

Treatment and Prevention of NSAID-Induced Gastropathy


Therapy for NSAID-induced gastropathy differs, depending on whether the goal is to heal or to prevent development of an NSAID-induced ulcer. In the attempt to heal an NSAID-induced ulcer, the first step is always to stop the NSAID. Once the NSAID is stopped, rapid ulcer healing is usually achieved by administering standard doses of H2-receptor antagonists or proton pump inhibitors. For patients who must continue to take NSAIDs, treatment with a proton pump inhibitor will achieve ulcer healing more rapidly than treatment with either ranitidine or misoprostol. One strategy to reduce NSAID-induced gastrointestinal complications is to prescribe drugs that, when administered concomitantly with NSAIDs, will protect against ulceration. Because most patients who take NSAIDs chronically will 16

Gastropathy Induced by Nonsteroidal Anti-inflammatory Drugs

never develop clinically significant ulceration, the ideal candidates for cotherapy are those considered at high risk for NSAID-induced ulcers (Table 9) (Laine). Various co-therapies have been used, including H2-receptor antagonists, prostaglandins, and proton pump inhibitors. The four H2-receptor antagonists (cimetidine, famotidine, nizatidine, and ranitidine) do not prevent NSAID-associated gastric ulcers when given at the usual ulcer-healing doses. However, when famotidine is administered at a high dose (40 mg twice daily), the development of NSAID-induced ulcers is effectively reduced. Misoprostol, the synthetic prostaglandin-E1 analog, reduces NSAIDrelated gastrointestinal adverse events by approximately half (that is, from 1% to 0.5%) (El-Serag et al.). However, misoprostol may cause dose-related diarrhea and is not effective for treating NSAID-associated dyspepsia. A combination tablet of misoprostol and the NSAID diclofenac is available that reduces the incidence of diarrhea and has a favorably low ulceration rate. Proton pump inhibitors (omeprazole, lansoprazole, rabeprazole, pantoprazole, and esomeprazole) are approximately as effective as misoprostol in preventing NSAID-induced ulcers (Graham et al.). Proton pump inhibitor cotherapy is also effective for reducing ulceration rates in patients at high risk for NSAID-induced ulcers who take either low-dose daily aspirin alone or together with an NSAID or a cyclooxygenase-2 (COX-2) inhibitor (Chan et al.).

Laine L. Approaches to nonsteroidal antiinflammatory drug use in the high-risk patient. Gastroenterology 2001;120:594-606. PMID: 11179238 El-Serag HB, Graham DY, Richardson P, Inadomi JM. Prevention of complicated ulcer disease among chronic users of nonsteroidal anti-inflammatory drugs: the use of a nomogram in cost-effectiveness analysis. Arch Intern Med. 2002;162:2105-10. PMID: 12374519 Graham DY, Agrawal NM, Campbell DR, Haber MM, Collis C, Lukasik NL, et al. Ulcer prevention in long-term users of nonsteroidal anti-inflammatory drugs: results of a double-blind, randomised, multicenter, active- and placebo-controlled study of misoprostol vs lansoprazole. Arch Intern Med. 2002;162:169-75. PMID: 11802750 Chan, FK, Hung LC, Suen BY, Wu JC, Lee KC, Leung VK, Hui AJ, et al. Celecoxib versus diclofenac and omeprazole in reducing the risk of recurrent ulcer bleeding in patients with arthritis. N Engl J Med. 2002;347:2104-10. PMID: 12501222 Bombardier C, Laine L, Reicin A, Shapiro D, Burgos-Vargas R, Davis B, et al. Comparison of upper gastrointestinal toxicity of rofecoxib and naproxen in patients with rheumatoid arthritis. VIGOR Study Group. N Engl J Med. 2000;343:1520-8. PMID: 11087881

NSAIDs with Improved Gastrointestinal Safety Profiles


Studies have shown that patients taking selective COX-2 inhibitors (celecoxib, rofecoxib, valdecoxib) have approximately the same rate of gastrointestinal ulcers found on endoscopy as those taking placebo. In addition, serious gastrointestinal complications are reduced by about 50% in patients taking COX-2 inhibitors compared with those taking traditional NSAIDs (Bombardier et al.). However, use of low-dose aspirin reduces the gastrointestinal protective benefit of selective COX-2 inhibitors. Although COX-2 inhibitors are associated with fewer gastrointestinal complications, these agents cause increased adverse events in other organ systems (for example, cardiovascular events) that may reduce or eliminate their overall benefits compared with traditional NSAIDs. This may be especially true for patients at risk of cardiovascular disease. If patients with an active ulcer take a selective COX-2 inhibitor, ulcer healing may be delayed (as also occurs in ulcer patients who are taking traditional NSAIDs). These patients should be treated with proton pump inhibitors. In addition to selective COX-2 inhibitors, several other established NSAIDs have safety profiles with a documented or a potential safety advantage when compared with other agents of the NSAID class. Older NSAIDs that are clinically associated with safer gastrointestinal profiles include etodolic acid, nabumetone, and nonacetylated salicylates such as salsalate. A newer agent, meloxicam, may also belong in this category.

KEYPOINTS

Patients with NSAID-induced ulcers are frequently asymptomatic. A significant number of serious gastrointestinal adverse events and death are associated with NSAID use. The first step in treating an NSAIDinduced ulcer is to stop the NSAID. Famotidine (at high doses), misoprostol, and proton pump inhibitors are usually effective in preventing NSAID-induced ulcerations. Patients treated with selective COX-2 inhibitors have fewer gastrointestinal complications than those treated with traditional NSAIDS.

TA B L E 9 Patients at High Risk for NSAID-induced Ulcers

(Candidates for Prophylaxis) Patients with a history of peptic ulcer with or without complications Patients requiring concomitant use of corticosteroids or anticoagulants Elderly patients (especially 75 years of age or older) Patients taking high doses of NSAIDs or combinations of more than one NSAID (including low-dose aspirin) Patients with comorbid diseases (e.g., functionally compromising heart disease)
NSAID = nonsteroidal anti-inflammatory drug

17

Gastric Polyps and Gastric Adenocarcinoma

Gastric Polyps and Gastric Adenocarcinoma


What are the most common types of gastric polyps and gastric adenocarcinomas? How can gastric submucosal tumors be differentiated from gastric polyps?

Demetri GD, von Mehren M, Blanke CD, Van den Abbeele AD, Eisenberg B, Roberts PJ, et al. Efficacy and safety of imatinib mesylate in advanced gastrointestinal stromal tumors. N Engl J Med. 2002;347:472-80. PMID: 12181401

KEYPOINTS

Hyperplastic gastric polyps have a small risk of progressing to adenocarcinoma. Adenomatous gastric polyps have a higher potential for malignant transformation. The optimal treatment of adenomatous polyps is endoscopic polypectomy. The optimal treatment for cure of gastric adenocarcinoma is partial or subtotal gastric resection.

The two most common types of gastric polyps are hyperplastic and adenomatous polyps, each of which is associated with atrophic gastritis. Hyperplastic polyps develop more often, typically measure less than 2 cm, and have a small risk (about 1%) for progression to adenocarcinoma. Adenomatous polyps have a higher potential for malignant transformation and should therefore be completely removed, preferably by endoscopic polypectomy. Patients with familial adenomatous polyposis have a relatively high incidence of gastric adenomatous polyps. Although commonly referred to as polyps, gastric submucosal tumors can be confused with gastric polyps of mucosal origin. Common polypoid-like gastric submucosal tumors include gastrointestinal stromal tumors and neuroendocrine tumors such as carcinoid. Submucosal tumors can usually be differentiated from polyps by upper endoscopy or, if necessary, by endoscopic ultrasonography. Gastric stromal tumors are usually small, asymptomatic, and benign. They may, however, cause abdominal pain or gastrointestinal bleeding or behave in a malignant fashion. Such tumors characteristically express the KIT protein, a transmembrane tyrosine kinase receptor. Imatinib mesylate, an orally administered tyrosine kinase inhibitor, is effective for treating advanced malignant or metastatic gastrointestinal stromal tumors (Demetri et al.). Gastric carcinomas are usually classified as either intestinal or diffuse. The intestinal variant is becoming less prevalent in the United States. It is usually well differentiated and is associated with H. pylori infection, chronic atrophic gastritis, and intestinal metaplasia. The diffuse variant is usually poorly differentiated, is not associated with chronic gastritis, and is characterized by the presence of signet-ring cells and excess mucus production. The prognosis for patients with gastric adenocarcinoma is primarily related to the depth of tumor penetration through the gastric wall, irrespective of the extent of nodal involvement. Penetration can be assessed by either preoperative endoscopic ultrasonography or by examination at the time of surgery. Early gastric cancer is limited to the gastric mucosa or submucosa. Advanced gastric cancer extends into the muscular layers of the stomach and has a poorer prognosis. Although endoscopic removal of gastric mucosal cancers has had some promising results, the most effective treatment for cure is partial or subtotal gastric resection.

Gastroparesis
What new therapeutic agents are available for treatment of gastroparesis? What new treatments are available for patients with gastroparesis associated with pylorospasm?

Gastroparesis is a clinical syndrome of nausea, vomiting, early satiety, and postprandial bloating with evidence of delayed gastric emptying. Associated conditions include diabetes mellitus, gastric surgery and/or vagotomy, progressive systemic sclerosis (scleroderma), amyloidosis, hypothyroidism, anorexia nervosa, intestinal pseudo-obstruction, dysautonomias, and gastric dysrhythmia. Gastroparesis is considered to be idiopathic when other conditions have been excluded. Associated pathophysiologic processes include gastric antral hypomotility, pylorospasm, impaired fundo-antral coordination, and small bowel dysmotility.

18

Acute Pancreatitis

Patients with gastroparesis should receive nutritional support and prokinetic agents. Nutritional support is best achieved by oral liquid supplements or blenderized foods. Enteral feedings are preferred to parenteral nutrition. The enteral nutrients can be given as oral supplements or as a direct infusion into the small bowel after placing a percutaneous endoscopic jejunostomy. First-line pharmacologic therapy involves use of a prokinetic agent, which stimulates gastric emptying. Intravenous erythromycin is the drug of choice for acute-onset gastroparesis. Metoclopramide is the preferred agent for chronic gastroparesis but has extrapyramidal neurologic side effects and can cause increased prolactin concentrations. A newer 5-HT4 agonist, tegaserod, accelerates gastric emptying in patients with dyspepsia and delayed gastric emptying. As some patients with gastroparesis also have pylorospasm, several novel approaches have been used to target dysfunction of the pylorus. These include the phosphodiesterase-5 inhibitor sildenafil (Viagra) and endoscopic injection of botulinum toxin into the pylorus. Electrical stimulation of the stomach by means of surgically implanted gastric pacemakers has been used for patients with refractory disorders. Although symptoms improve relative to baseline findings following pacemaker implantation, gastric emptying is not normalized. The mechanism responsible for symptomatic improvement with use of gastric pacemakers is therefore unclear. Surgical options as treatments of last resort include surgical placement of a jejunostomy tube or total gastrectomy.

KEYPOINTS

Management of gastroparesis includes enteral nutritional support plus administration of prokinetic agents. Administration of sildenafil or botulinum toxin may be effective in treating patients with gastroparesis associated with pylorospasm. Electrical stimulation of the stomach by means of surgically implanted gastric pacemakers may be effective for patients with refractory gastroparesis.

Pancreas
The most common disorders of the pancreas are acute and chronic pancreatitis, pancreatic cancer, mucin-secreting and cystic neoplasms, and pancreatic endocrine tumors.

Acute Pancreatitis
What are the two most common causes of acute pancreatitis? What are the mortality rates for patients with acute pancreatitis? Which diagnostic studies should be done initially when evaluating patients with possible acute pancreatitis? What treatment options are available for patients with different types of acute pancreatitis? What are the complications of acute pancreatitis?

Case 2 A 68-year-old woman with type 2 diabetes mellitus and congestive heart failure is evaluated because of the abrupt onset of nausea, vomiting, and epigastric pain that radiates to her back. On physical examination, temperature is 38.8 C (101.8 F), pulse rate is 106/min, respiration rate is 17/min, and blood pressure is 145/95 mm Hg. Her sclerae are icteric. Abdominal examination discloses epigastric tenderness and distended bowel. The leukocyte count is 16,200/L, serum aspartate aminotransferase is 264 U/L, serum amylase is 2343 U/L, serum total bilirubin is 6.2 mg/dL, and serum albumin is 2.6 g/dL. Acute pancreatitis is an inflammatory process that may also involve organs adjacent to the pancreas and may adversely affect remote organs. The incidence of

19

Acute Pancreatitis

Cohn JA, Friedman KJ, Noone PG, Knowles MR, Silverman LM, Jowell PS. Relation between mutations of the cystic fibrosis gene and idiopathic pancreatitis. N Engl J Med. 1998;339:653-8. PMID: 9725922 Sharer N, Schwartz M, Malone G, Howarth A, Painter J, Super M, Braganza J. Mutations of the cystic fibrosis gene in patients with chronic pancreatitis. N Engl J Med. 1998;339:645-52. PMID: 9725921 Gorry MC, Gabbaizedeh D, Furey W, Gates LK Jr, Preston RA, Aston CE, et al. Mutations in the cationic trypsinogen gene are associated with recurrent acute and chronic pancreatitis. Gastroenterology. 1997;113:1063-8. PMID: 9322498 Whitcomb DC, Gorry MC, Preston RA, Furey W, Sossenheimer MJ, Ulrich CD, et al. Hereditary pancreatitis is caused by a mutation in the cationic trypsinogen gene. Nat Genet. 1996;14:141-5. PMID: 8841182

pancreatitis has increased several-fold in Western industrialized countries over the past 2 to 3 decades, and the mortality rate is 5% to 6%. Gallstones are the most common cause of acute pancreatitis, and excess alcohol consumption is the second most common cause (Table 10). When a cause cannot be identified, the disorder is classified as acute idiopathic pancreatitis. Genetic mutations have been identified in patients with various forms of pancreatitis (Table 11). Mutations in the cystic fibrosis transmembrane conductance regulator (CFTR) have been found in some patients with acute idiopathic pancreatitis as well as in those with chronic pancreatitis (Cohn et al.). In one series, 18 of 134 consecutive patients with chronic pancreatitis had the CFTR mutation (Sharer et al.). However, these patients did not have abnormal sweat chloride levels, lung disease, or other features typical of cystic fibrosis. Mis-sense mutations in the cationic trypsinogen gene have been identified in patients with acute and chronic pancreatitis (Gorry et al.). Such mutations have also been found in patients with hereditary pancreatitis, which is a rare early-onset autosomal dominant disorder with 80% penetrance and variable expression (Whitcomb et al.). Hereditary pancreatitis usually occurs in persons younger than 20 years of age. Calcific pancreatitis usually results and is often accompanied by pseudocyst formation, exocrine and endocrine failure, and pancreatic cancer. Since only 60% of patients with hereditary pancreatitis have trypsinogen gene mutations, other as yet unidentified genetic mutations may be present in persons with this disorder.

TA B L E 1 0 Causes of Acute and Chronic Pancreatitis

Acute Pancreatitis* Gallstones Alcohol consumption Hyperlipidemia Drugs (e.g., azathioprine/6-mercaptopurine, thiazide diuretics, oral contraceptives) Trauma (blunt or penetrating) Surgery Postendoscopic retrograde cholangiopancreatography Chronic Pancreatitis* Alcohol consumption Prior severe acute pancreatitis Hereditary Autoimmune Tropical
*When a cause cannot be identified, the pancreatitis is considered idiopathic.

TA B L E 1 1 Genetic Mutations Associated with Pancreatitis

Genetic Mutation Cystic fibrosis transmembrane conductance regulator (CFTR) Cationic trypsinogen genes (R122H, N29, R117H)

Type of Pancreatitis Acute Chronic Acute Chronic Hereditary Tropical

Pancreatic secretory trypsin inhibitor (PSTI/SPINK1)

20

Acute Pancreatitis

Diagnosis of Acute Pancreatitis


Patients with acute pancreatitis have varied presentations. Severe upper or diffuse abdominal pain usually occurs. The pain is sudden in onset and is associated with nausea and vomiting. Increased circulating pancreatic enzyme activity, increased trypsinogen 2 levels, or both, are common (Hedstrom et al.; Smotkin and Tenner). Although most patients have serum amylase and lipase values of at least 2 to 3 times the upper limits of normal, approximately 30% have amylase elevations of less than 3 times normal. Patients with alcoholic pancreatitis tend to have lower amylase values at presentation. Therefore, a normal amylase value should not exclude the diagnosis of pancreatitis in the appropriate clinical setting (Lankisch et al.). The sensitivity and specificity of serum amylase and lipase assays are similar. Both enzymes can be falsely elevated in patients with intra-abdominal inflammation, renal insufficiency, or increased production of nonpancreatic enzymes. Hypertriglyceridemia interferes with the accuracy of the amylase assay and may cause a false-negative result. Lipase assays are somewhat more sensitive for detecting alcoholic pancreatitis and for diagnosing pancreatitis in patients with late clinical presentations. The severity of acute pancreatitis is not related to the degree of elevation of the serum amylase and lipase levels. Urine and serum trypsinogen-2 levels are elevated in patients with acute pancreatitis and, unlike serum amylase and lipase levels, do correlate with disease severity. However, trypsinogen-2 assays are not widely available. Initial radiologic studies usually include a plain film of the abdomen and chest radiograph.Ultrasonography of the abdomen and pelvis is especially useful for assessing gallstone-induced pancreatitis.

Hedstrom J, Kemppainen E, Andersen J, Jokela H, Puolakkainen P, Stenman UH. A comparison of serum trypsinogen-2 and trypsin-2-alpha1-antitrypsin complex with lipase and amylase in the diagnosis and assessment of severity in the early phase of acute pancreatitis. Am J Gastroenterol. 2001;96:424-30. PMID: 11232685 Smotkin J, Tenner S. Laboratory diagnostic tests in acute pancreatitis. J Clin Gastroenterol. 2002;34:459-62. PMID: 11907364 Lankisch PG, Burchard-Reckert S, Lehnick D. Underestimation of acute pancreatitis: patients with only a small increase in amylase/lipase levels can also have or develop acute pancreatitis. Gut. 1999;44:542-4. PMID: 10075962

Prognostic Factors in Acute Pancreatitis


Outcomes in patients with acute pancreatitis are related to whether the disease is interstitial (80% of patients) or necrotizing (20% of patients). Patients with interstitial pancreatitis have a mortality rate of less than 1%, whereas those with necrotizing pancreatitis have a 10% to 30% risk of dying and a 70% risk of developing complications. Failure of other organ systems is also associated with a poor prognosis. Younger age and disease due to biliary causes are more favorable prognostic indicators. Because management of high-risk patients differs from that of patients who are less severely ill, initial evaluation should include assessing the risks of complications and determining the possibility of death (Banks; Triester and Kowdley). These prognostic factors are usually determined by noninvasive means, such as scoring systems, serologic studies, and CT scanning. Many scoring systems are available (for example, Ranson, Glasgow, and Acute Physiology and Chronic Health Evaluation II [APACHE II]) (Robert et al.). These scoring systems are helpful because they have negative predictive values of 86% to 97%. Other factors that identify patients at high risk are a body mass index greater than 25, hematocrit greater than 50%, and an elevated C-reactive protein level. However, no single score or test is completely accurate and should not supplant clinical findings such as third space fluid loss or remote organ failure in determining risk. CT scanning with intravenous contrast should be considered for high-risk patients to detect the presence and extent of necrotizing pancreatitis. Necrosis is identified as a hypodense area of the pancreas that does not enhance with contrast. CT scanning usually can be deferred until the fourth hospital day. If intravenous contrast is contraindicated in a high-risk patient, a non-contrastenhanced CT scan may be obtained to diagnose the presence of single or multiple fluid collections.

Banks PA. Practice guidelines in acute pancreatitis. Am J Gastroenterol. 1997;92:37786. PMID: 9068455 Triester SL, Kowdley KV. Prognostic factors in acute pancreatitis. J Clin Gastroenterol. 2002;34:167-76. PMID: 11788614 Robert JH, Frossard JL, Mermillod B, Soravia C, Mensi N, Roth M, et al. Early prediction of acute pancreatitis: prospective study comparing computered tomography scans, Ranson, Glasgow, Acute Physiology and Chronic Health Evaluation II scores, and various serum markers. World J Surg. 2002;26:612-9. PMID: 12098056

21

Chronic Pancreatitis

In patients with severe acute pancreatitis, the most likely cause of death during the first week is remote organ failure, primarily due to circulatory shock or acute respiratory distress syndrome. Organ failure in patients with pancreatitis has been linked to cytokine activation. After the first week, death is most often due to pancreatic and/or systemic infections.

Treatment of Acute Pancreatitis


Toouli J, Brooke-Smith M, Bassi C, Carr-Locke D, Telford J, Freeny P, et al. Guidelines for the management of acute pancreatitis. J Gastroenterol Hepatol. 2002;17:S15-39. PMID: 12000591 Bassi C, Falconi M, Talamini G, Uomo G, Papaccio G, Dervenis C, et al. Controlled clinical trial of perfloxacin versus imipenem in severe acute pancreatitis. Gastroenterology. 1998;115:1513-7. PMID: 9834279 Nordback I, Sand J, Saaristo R, Paajanen H. Early treatment with antibiotics reduces the need for surgery in acute necrotizing pancreatitis. J Gastrointest Surg. 2001;5:113-20. PMID: 11331472 Baron TH, Morgan DE. Acute necrotizing pancreatitis. N Engl J Med. 1999;340:1412-7. PMID: 10228193

Patients with severe acute pancreatitis should be admitted to an intensive care unit for effective management (Toouli et al.). Initial therapy is supportive and involves use of rapid, aggressive intravenous hydration. Enteral nutrition is preferred to parenteral feedings because of the increased complications and higher costs associated with parenteral alimentation. Patients with gallstone-induced pancreatitis who develop cholangitis require urgent endoscopic retrograde cholangiography with endoscopic stone removal. Prospective randomized controlled clinical trials have shown that administering imipenem prophylactically reduces infectious complications in patients with necrotizing pancreatitis (Bassi et al.; Nordback et al.). If infected necrosis is suspected in a patient with remote organ failure or systemic toxicity, fine-needle aspiration should be performed, followed by surgical drainage if infection is confirmed. Some centers have successfully treated pancreatic infections by percutaneous and endoscopic approaches rather than by surgery.

Complications of Acute Pancreatitis


Fluid collections attributed to acute pancreatitis develop in approximately 30% of patients and resolve spontaneously in 50% of these patients. The fluid collections tend to occur early in the course of the disease, develop within the periphery of the pancreas, lack a definitive wall, and evolve into pancreatic abscesses, pseudocysts, or necrosis. Patients with excess fluid collections are more likely to have persistent symptoms and require surgical, endoscopic, or percutaneous drainage. Endoscopic drainage is used only for lesions adjacent to the stomach or duodenum. Transgastric drainage by either endoscopic or surgical means is effective, but cyst-jejunostomy is preferred because of the lower rate of recurrence. The presence of cystic pancreatic neoplasms should also be considered in patients with a fluid collection but without a history of acute pancreatitis. A pancreatic pseudoaneurysm can cause bleeding into the retroperitoneum, peritoneum, and bowel and is treated by arteriographic embolization. Fistulae may occur spontaneously but generally develop after pancreatic resection or percutaneous drainage. Pancreatic inflammation can result in splenic vein thrombosis with the formation of isolated gastric varices, which may later bleed. Diabetes mellitus is a frequent complication of pancreatitis in patients who require pancreatic resection or in those with alcohol-induced pancreatitis. Although the costs and complications of acute necrotizing pancreatitis are significant, quality-of-life outcomes at 2-year follow-up are similar to outcomes after coronary artery bypass graft surgery (Baron and Morgan). In addition, approximately 65% of patients who recover are actively employed at 6-year follow-up.

KEYPOINTS

The two most common causes of acute pancreatitis are gallstones and excessive alcohol consumption. The mortality rate for patients with interstitial pancreatitis is approximately 1%, whereas the mortality rate for those with necrotizing pancreatitis is 10% to 30%. The initial evaluation of patients with acute pancreatitis involves radiologic studies (including ultrasonography) and pancreatic enzyme determinations. Fluid collections, pseudoaneurysms, fistulae, and diabetes mellitus are common complications in patients with acute pancreatitis.

Chronic Pancreatitis
What are the most common causes of chronic pancreatitis? What features are characteristic of the newly described entity of autoimmune pancreatitis? What is the role of endoscopic ultrasonography in diagnosing chronic pancreatitis? How should patients with refractory pain due to chronic pancreatitis be managed?

22

Chronic Pancreatitis

Chronic pancreatitis is an inflammatory disorder characterized by morphologic changes, including strictures, calculi, and dilatation of the pancreatic duct. These changes are irreversible and are typically associated with pain and loss of pancreatic endocrine and exocrine function. Causes of chronic pancreatitis are listed in Table 10. The most common cause is alcohol abuse, which accounts for 60% or more of all cases in Western industrialized countries. Alcohol-induced chronic pancreatitis occurs primarily in men 35 to 45 years of age, and a history of drinking 150 g or more of ethanol daily for 6 or more years almost always predates the clinical onset of disease. Except for some cases of tropical pancreatitis, which is especially prevalent in India, most of the remaining cases of chronic pancreatitis are due to idiopathic causes. Recently, genetic mutations in the pancreatic secretory trypsin inhibitor gene (PSTI/SPINK1), the cationic trypsinogen gene, and the cystic fibrosis transmembrane conductance regulator (CFTR) have been identified in subgroups of patients with chronic pancreatitis who were previously classified as having idiopathic disease. Autoimmune pancreatitis is a newly described type of chronic pancreatitis (Okazaki and Chiba). It occurs most often in the Far East and is associated with hypergammaglobulinemia, diffuse enlargement of the pancreas, an irregular main pancreatic duct, and the presence of autoantibodies such as antinuclear antibody. Patients are usually asymptomatic or have only mild symptoms. Autoimmune pancreatitis is occasionally associated with other autoimmune diseases (most commonly Sjgrens syndrome) as well as with ulcerative colitis, primary sclerosing cholangitis, and systemic lupus erythematosus. Histologic findings include fibrotic changes with lymphocytic infiltrates. The intrapancreatic segment of the common bile duct is frequently strictured, and changes similar to sclerosing cholangitis have been reported.

Okazaki K, Chiba T. Autoimmune related pancreatitis. Gut. 2002;51:1-4. PMID: 12077078

Diagnosis of Chronic Pancreatitis


Repeated severe attacks of acute pancreatitis may result in morphologic changes or loss of pancreatic function associated with chronic pancreatitis. However, establishing the diagnosis of chronic pancreatitis in a patient with early disease can be difficult. Imaging studies such as CT scanning and endoscopic retrograde cholangiopancreatography are not very sensitive because early morphologic changes may be equivocal. Endoscopic ultrasonography has therefore been advocated as an alternative diagnostic means. In a prospective study of patients with suspected chronic pancreatitis, the diagnostic sensitivity of endoscopic ultrasonography was 88% compared with 75% for CT scanning, 74% for endoscopic retrograde cholangiopancreatography, and 58% for non-endoscopic ultrasonography (Buscail et al.). The specificity of endoscopic ultrasonography was 100% versus 95% for CT scanning. However, because echoendoscopic features of lobular architecture and hypoechoic changes of the pancreatic parenchyma may overlap with those of pancreatic cancer, differentiating the two entities is only 75% accurate. Fine-needle aspiration will increase the diagnostic accuracy but is associated with false-negative results for pancreatic cancer.

Buscail L, Escourrou J, Moreau J, Delvaux M, Louvel D, Lapeyre F, et al. Endoscopic ultrasonography in chronic pancreatitis: a comparative prospective study with conventional ultrasonography, computerized tomography, and ERCP. Pancreas. 1995;10:257-7. PMID: 7624302 Warshaw AL, Banks PA, FernandezDel Castillo C. AGA technical review: treatment of pain in chronic pancreatitis. Gastroenterology. 1998;115:765-76. PMID: 9721175

Treatment of Chronic Pancreatitis


Treatment focuses on relief of chronic pain and includes use of pancreatic enzyme supplementation, octreotide, endoscopic therapy, and nerve block (Warshaw et al.). When these are ineffective, surgery (pancreatectomy, pseudocyst drainage, or pancreaticojejunostomy) has resulted in relief of refractory pain in 52% to 94% of patients. Chronic pain may also subside spontaneously in patients with uncomplicated advanced chronic pancreatitis. Recurrence of pain had been linked to continued alcohol use and smoking, and patients should be

23

Pancreatic Cancer

Ammann RW, Muellhaupt B. The natural history of pain in alcoholic chronic pancreatitis. Gastroenterology. 1999;116:1132-40. PMID: 10220505 Talamini G, Bassi C, Falconi M, Sartori N, Salvia R, Francesco VD, et al. Pain relapses in the first 10 years of chronic pancreatitis. Am J Surg. 1996;171:565-9. PMID: 8678201

advised to reduce or abstain from these practices (Ammann and Muellhaupt; Talamini et al.). Corticosteroids are beneficial for treating biliary strictures in patients with autoimmune pancreatitis.

Complications of Chronic Pancreatitis


In addition to chronic pain, the most common complication of chronic pancreatitis is the development of pseudocysts. Larger pseudocysts (>4 cm) and extrapancreatic lesions are independent predictors of persisting symptoms and complications.

KEYPOINTS

Excessive alcohol consumption is the most common cause of chronic pancreatitis. Endoscopic ultrasonography is both sensitive and specific for diagnosing chronic pancreatitis. Surgery may be necessary to control refractory pain in patients with chronic pancreatitis.

Pancreatic Cancer
What are the risk factors for the development of pancreatic cancer? Which genetic alterations have been identified in patients with pancreatic cancer? How is a pancreatic mass evaluated? What are the treatment options for a patient with pancreatic cancer complicated by jaundice and lymphadenopathy?

Pancreatic cancer occurs primarily in Western industrialized countries. The incidence is 10.1/100,000 for men and 7.5/100,000 for women. Black persons have a 30% to 40% increased risk compared with white persons. The incidence of pancreatic cancer increased by more than 300% from 1920 to 1970 and has remained unchanged since then. Approximately 90% of pancreatic cancers are adenocarcinomas, and about 5% are neuroendocrine tumors. Survival rates for patients with pancreatic adenocarcinoma are the lowest for all types of cancer, with a 5-year survival rate of 3% to 5%. Patients with nonmetastatic, locally advanced disease have a 6- to 10month median survival, whereas those with metastatic disease have a median survival of 3 to 6 months. Survival is linked to a patients performance status and extent of disease at the time of diagnosis.

Risk Factors for the Development of Pancreatic Cancer Environmental and Medical Risk Factors
The environmental and medical risk factors for pancreatic cancer are listed in Table 12. Smokers have a 1.6 to 3.1 increased risk compared with nonsmokers. Obesity and high caloric intake have been associated with an increased risk, whereas consumption of fresh fruits and vegetables may be protective. Whether coffee and alcohol consumption are risk factors is unclear. Exposure to industrial carcinogens and radiation increases the risk, as does administration of cytotoxic agents. Undergoing partial gastrectomy increases the risk of pancreatic cancer by 3- to 7-fold. Hereditary pancreatitis and perhaps chronic pancreatitis have been associated with an increased risk, although additional studies are needed to confirm whether chronic pancreatitis is indeed a risk factor. Patients with pancreatic cancer are 5.2- to 13-fold more likely to have a first-degree relative with pancreatitis. Other medical disorders associated with the development of pancreatic cancer are listed in Table 12.

Genetic Risk Factors


Hruban RH, Iacobuzio-Donahue C, Wilentz RE, Goggins M, Kern SE. Molecular pathology of pancreatic cancer. Cancer J. 2001;7:251-8. PMID: 11561601

Alterations of four major classes of genes (oncogenes, tumor suppressor genes, mismatch repair genes, and mitochondrial genes) have been identified in association with pancreatic cancer (Hruban et al.). The most common alterations occur on the K-ras oncogene and various tumor suppressor genes. The K-ras oncogene is activated by a point mutation in 90% of pancreatic cancers, and

24

Pancreatic Cancer

TA B L E 1 2 Risk Factors for the Development of Pancreatic Cancer

Environmental Smoking Industrial carcinogens DDT 2-Naphthylamine Gasoline derivatives Radiation Dietary* Obesity and high-caloric intake Associated Medical Conditions Hereditary pancreatitis Tropical pancreatitis Chronic pancreatitis Gastrectomy Diabetes mellitus Administration of cytotoxic agents Hereditary disorders Multiple endocrine neoplasia type 1 Glucagonoma syndrome Gardners syndrome (hereditary) Breast cancer associated with BRCA2 mutation Peutz-Jeghers syndrome Neurofibromatosis Ataxia-telangiectasia Lynch II syndrome
*Consumption of fresh fruits and vegetables may be protective.

three tumor suppressor genes are inactivated in the majority of pancreatic neoplasms (p16 in 95% of cancers, p53 in 50% to 75%, and DPC4 in 55%). Other tumor suppressor genes may be inactivated to lesser degrees (Table 13).

Clinical Presentation and Diagnosis of Pancreatic Cancer


Signs and symptoms of pancreatic cancer include abdominal or back pain in approximately 80% of patients, weight loss in 60%, and jaundice in 47%. Newonset diabetes mellitus may also be found. Pain results from tumor infiltration of the nerves. Because the presentation of pancreatic cancer is similar to that of various other disorders, patients with unexplained pain, new-onset diabetes mellitus, and/or pancreatic exocrine failure require careful evaluation. The goal of the diagnostic evaluation is to demonstrate and stage a lesion in order to determine resectability (Hawes et al.). This is most often done by imaging studies such as CT scans, endoscopic ultrasonography, and endoscopic retrograde cholangiopancreatography (ERCP) (Tamm and Charnsangavej). Helical (spiral) CT scanning with thin cuts of the pancreas improves the accuracy of staging and can predict resectability in 90% of patients. Endoscopic ultrasonography is more accurate than either CT scanning or standard ultrasonography in staging the extent (T) and nodal status (N) of pancreatic neoplasms. However, endoscopic ultrasonography is less useful than CT scanning for staging metastatic disease because the quality of the images deteriorates beyond 5 cm from the endoscopic probe.

Hawes RH, Xiong Q, Waxman I, Chang KJ, Evans DB, Abbruzzese JL. A multispecialty approach to the diagnosis and management of pancreatic cancer. Am J Gastroenterol. 2000;95:17-31. PMID: 10638554 Tamm E, Charnsangavej C. Pancreatic cancer: current concepts in imaging for diagnosis and staging. Cancer J. 2001; 7:298-311. PMID: 11561606

25

Pancreatic Cancer

TA B L E 1 3 Genetic Mutations Associated with Pancreatic Cancer

Genes Tumor suppressor genes: p16 p53 DPC4 BRCA2 Oncogenes: K-ras AKT2 MYB DNA mismatch repair genes: hMLH1 hMLH2

Frequency

95% 65% 55% 7% 90% 10% <10% <5% <5%

Adapted from Hruban RH, Iacobuzio-Donahue C, Wilentz RE, Goggins M, Kern SE. Molecular pathology of pancreatic cancer. Cancer J. 2001;7:251-8. Copyright 2001. Jones and Bartlett Publishers, Sudbury, MA. www.jbpub.com. Reprinted with permission.

Because of the increasing use of other imaging modalities, the role of ERCP in identifying possible pancreatic malignancies is decreasing. ERCP-facilitated stenting is still indicated for preoperative management of cholangitis, but whether stents are beneficial for treating jaundice preoperatively is currently unclear. The need for histologic tissue confirmation of a mass-like pancreatic lesion depends on the clinical index of suspicion for pancreatic cancer. Patients with risk factors for pancreatic cancer who have a resectable lesion demonstrated by imaging studies are sometimes referred directly to surgery without biopsy confirmation. However, the primary lesion may not be an adenocarcinoma, and the management of non-adenocarcinoma tumors (neuroendocrine tumors, lymphoma, small cell cancer) is different. Therefore, an attempt at tissue diagnosis is prudent. This can be done using an endoscopic or a percutaneous approach, as described below. The sensitivity of CT-guided fine-needle aspiration for histologic confirmation has been reported to range from 45% to 100%. However, this technique is useful only if CT scans are first able to detect a lesion. In one multicenter trial, only 44% of pancreatic cancers were found by CT (see Hawes et al.). Endoscopic ultrasonography with fine-needle aspiration is superior to CT-guided fine-needle aspiration for detecting lesions and sampling lymph nodes. Levels of serum tumor markers such as CA 19-9 are frequently increased in patients with pancreatic adenocarcinoma. CA 19-9 lacks specificity, however, as CA 19-9 levels can also be highly elevated in patients with benign lesions. Although genetic alterations have been identified in patients with pancreatic cancer, the value of these findings in actually detecting cancer is unknown at this time.

Treatment of Pancreatic Cancer


Cancers of the tail of the pancreas are usually associated with advanced disease, as patients generally have arterial encasement and peritoneal metastases when first diagnosed. Although surgery for palliation or cure is the accepted choice for initial therapy, this is not evidence based, and one outcomes review noted no difference in 5-year survival for patients who were initially treated medically (chemotherapy and radiation therapy) or surgically (Gudjonsson). Surgical outcomes are enhanced when patients are treated at centers that perform this

Gudjonsson B. Carcinoma of the pancreas: critical analysis of costs, results of resections, and the need for standardized reporting. J Am Coll Surg. 1995;181:483-503. PMID: 7582222

26

Mucin-Secreting and Cystic Neoplasms of the Pancreas

procedure often. Such high-volume centers have 30-day mortality rates of 2% or less. Administration of 5-fluorouracil is the standard chemotherapy for pancreatic cancer. A randomized trial compared 5-fluorouracil with gemcitabine (a recently released deoxycytidine analog). Gemcitabine provided better symptom control in a greater percentage of patients (24% versus 5%) but resulted in only a small survival benefit (5.7 months versus 4.4 months) (Burris et al.). (Treatment of pancreatic cancer is discussed in greater detail in the Hematology/Oncology syllabus.)

Burris HA 3rd, Moore MJ, Anderson J, Green MR, Rothenberg ML, Modiano MR, et al. Improvements in survival and clinical benefits with gemcitabine as first-line therapy for patients with advanced pancreas cancer. J Clin Oncol. 1997;15:2403-13. PMID: 9196156
KEYPOINTS

Mucin-Secreting and Cystic Neoplasms of the Pancreas


What genetic alterations are associated with an intraductal papillary mucinous tumor? What findings are characteristic of intraductal papillary mucinous tumors on imaging studies? How are intraductal papillary mucinous tumors differentiated from mucin-producing cystic tumors?

Mucin-secreting and cystic neoplasms are relatively uncommon pancreatic disorders that include intraductal papillary mucinous tumors, mucin-producing cystic tumors, papillary tumors, and neuroendocrine tumors.

Pancreatic cancer is the fourth most common cause of cancer deaths in the United States. Alterations in four major classes of genes have been identified in patients with pancreatic cancer. Serum tumor markers such as CA 19-9 are frequently increased in patients with pancreatic cancer, but the test is not specific for this diagnosis. The goal of evaluation of a patient with possible pancreatic cancer is to identify and stage the tumor in order to determine resectability. The initial treatment of a patient with pancreatic cancer usually involves surgery for palliation or cure.

Intraductal Papillary Mucinous Tumors


Intraductal papillary mucinous tumors were previously referred to as mucinous ductal ectasia, mucin-hypersecreting tumors, mucin-producing tumors, ductectatic carcinoma of the pancreas, ductectatic cystadenomas, and cystadenocarcinomas. These tumors occur most often in men 60 years of age and older. Intraductal papillary mucinous tumors secrete mucin into the pancreatic ductal system, which causes intermittent obstruction of the pancreatic duct and leads to recurrent attacks of acute pancreatitis and eventually to chronic pancreatitis. A subset of patients may be asymptomatic (Farrell and Brugge; Zamora et al.). The tumor may originate in either the main or the branch pancreatic ducts. Main pancreatic duct tumors are more common and develop in the head of the pancreas in 46% of patients, the body of the pancreas in 40%, or throughout the pancreas in 14%. Invasive cancer or carcinoma in-situ is found in 70% of all main duct tumors. Branch duct tumors develop in the head of the pancreas in 80% of patients; 24% of these tumors are cancers. The 5-year survival rate is 47% for patients with main duct tumors and 90% for those with branch duct tumors. Once regional spread has occurred, the prognosis is the same as that for pancreatic adenocarcinoma (Maire et al.). Several genetic abnormalities have been found in patients with an intraductal papillary mucinous tumor. Expression of DPC4 occurs in 100% of patients, and MUC2 mucin and MUC5 mucin mRNA are expressed in more than 80% of patients (Nakamura et al.). K-ras and p53 mutations are less common. Intraductal papillary mucinous tumors are diagnosed by imaging studies, such as pancreatography, endoscopic ultrasonography, and endoscopic retrograde cholangiopancreatography. On endoscopic examination, the ampulla has a dilated fish mouth opening and contains mucinous material (Figure 7). On pancreatography, the main pancreatic duct is dilated, contains filling defects, and is associated with side branch ectasia. Treatment involves surgical resection with careful evaluation of the surgical margins to determine whether the resection needs to be extended.

Farrell JJ, Brugge WR. Intraductal papillary mucinous tumor of the pancreas. Gastrointest Endosc. 2002;55:701-14. PMID: 11979253 Zamora C, Sahel J, Cantu DG, Heyries L, Bernnard JP, Bastid C, et al. Intraductal papillary or mucinous tumors (IPMT) of the pancreas: report of a case series and review of the literature. Am J Gastroenterol. 2001;96:1441-7. PMID: 11374680 Maire F, Hammel P, Terris B, Paye F, Scoazec JY, Cellier C, et al. Prognosis of malignant intraductal papillary mucinous tumours of the pancreas after surgical resection. Comparison with pancreatic ductal adenocarcinoma. Gut. 2002;51:717-22. PMID: 12377813 Nakamura A, Horinouchi M, Goto M, Nagata K, Sakoda K, Takao S, et al. New classification of pancreatic intraductal papillary-mucinous tumour by mucin expression: its relationship with potential for malignancy. J Pathol. 2002;197:201-10. PMID: 12015744

27

Pancreatic Endocrine Tumors

Mucin-Producing Cystic Tumors


Although intraductal papillary mucinous tumors and mucin-producing cystic tumors of the pancreas are often confused, the World Health Organization and the Armed Forces Institute of Pathology have classified these as two distinct entities. Mucin-producing cystic tumors (mucinous cystadenomas) are large cystic tumors lined by mucus-secreting epithelial cells that most often occur in the body or tail of the pancreas. The tumors are the most common cystic lesions of the pancreas and almost always occur in middle-aged women. They are unilocular, have a thin wall, and do not communicate with the pancreatic duct. Mucus floats free in the cyst. Invasive cancer frequently develops, and the 5-year survival rate is 33%. Because of their malignant potential, surgical excision is required (Rattner et al.).

FIGURE 7. Endoscopic findings of intraductal papillary mucinous tumor. The ampulla shows the characteristic dilated fish mouth opening. (See Color Plate 4, at back of book.)

Other Cystic Neoplasms of the Pancreas


Serous cystadenomas are the second most common cystic neoplasm of the pancreas. They occur predominantly in women in their early 60s. Approximately 50% of patients are asymptomatic; the remainder usually present with pain, an abdominal mass, and/or weight loss. These lesions are microcystic, contain multiple small compartments separated by thin-walled septations, and do not contain debris or mucin. Larger lesions have central fibrosis or calcification with a characteristic sunburst appearance on CT scans. The presence of glycogenstaining cells on histologic examination is diagnostic. Serous cystadenomas have a low rate of malignancy (<1%). Therefore, elderly patients with radiologic features of serous cystadenoma and patients who are not good surgical candidates can be managed by observation alone. Patients with moderate to severe symptoms and/or an enlarging cystic neoplasm may require surgical resection. Other rare cystic neoplasms include papillary mucinous tumors and neuroendocrine tumors (for example, insulinoma, gastrinoma, glucagonoma). Both papillary and neuroendocrine tumors can become malignant and should be resected. Finally, approximately 75% of patients with von HippelLindau disease (an autosomal dominant disorder that affects various organs) have pancreatic lesions, including cysts, serous cystadenomas, neuroendocrine tumors, or some combination of these lesions.

Rattner DW, Fernandez-del Castillo C, Warshaw AL. Cystic pancreatic neoplasms. Ann Oncol. 1999;10:104-6. PMID: 10436797
KEYPOINTS

Although intraductal papillary mucinous tumors and mucin-producing cystic tumors have many similar features, they are classified as distinct disease entities. Surgical resection is generally required for patients with either intraductal papillary mucinous tumors or mucin-producing cystic tumors. Because serous cystadenomas have a less than 1% malignant potential, many patients with this tumor can be managed by observation alone.

Pancreatic Endocrine Tumors


How do functional and nonfunctional pancreatic endocrine tumors differ? What are the most common pancreatic endocrine tumors? How are pancreatic endocrine tumors diagnosed?

Case 3 A 67-year-old man comes to the emergency department because of severe abdominal pain. He has a history of dyspepsia that is effectively managed with proton pump inhibitors. One week ago, he stopped taking his proton pump inhibitor because of the expense. Abdominal examination discloses moderate epigastric tenderness. A plain radiograph shows no free air in the abdomen. A fasting serum gastrin level is 1876 pg/mL. An intravenous proton pump inhibitor is started, and the patients abdominal pain improves over the next 4 days. Although the term islet cell tumor is frequently used for pancreatic endocrine tumors, this is a misnomer, as many tumors arise from outside the pancreas, 28

Pancreatic Endocrine Tumors

such as in the duodenum. These tumors are either functional (characterized by a clinical syndrome) or nonfunctional. The various functional pancreatic and duodenal endocrine tumor syndromes are shown in Table 14. In order to diagnose a functional pancreatic endocrine tumor, the patient must have the associated clinical syndrome and elevated levels of the associated hormone. Scintigraphy using radiolabeled octreotide (a somatostatin analog) is the initial study for evaluating the location and extent of most primary pancreatic endocrine tumors (Alexander et al.; Jensen; Zimmer et al.). Positron emission tomography can be used in conjunction with scintigraphy. Endoscopic ultrasonography is a very sensitive study for detecting these lesions and is the preferred imaging modality for suspected insulinomas (Anderson et al.). Tumor immunohistochemical studies alone are insufficient for making a diagnosis. Gastrinomas occur almost as frequently in the duodenum as in the pancreas. Gastric acid hypersecretion in patients with gastrinomas can be controlled with proton pump inhibitors before staging and treating the tumor. Approximately 60% to 90% of gastrinomas are malignant. The presence of liver metastases is the single most important determinant of survival. Approximately 25% of patients have aggressive tumors with a 10-year survival rate of 30%. This compares with a 10-year survival rate of 96% for those with nonaggressive tumors. Multiple endocrine neoplasia type 1 (MEN 1) is present in almost 25% of patients with gastrinomas; in these patients the tumors are associated with a genetic mutation at chromosome 11q13. The primary therapy for localized gastrinomas is surgery. However, surgery in patients with MEN 1 and Zollinger-Ellison syndrome is controversial, as only 16% of patients are disease free in the immediate postoperative period. The treatment of functional symptomatic pancreatic endocrine tumors may include synthetic somatostatin analogs. Insulinomas measure 1 to 5 cm in 50% of patients, 0.5 to 1 cm in 33% of patients, and greater than 5 cm in 8% of patients. Whipples triad (symptoms of hypoglycemia, fasting plasma glucose values <50 mg/dL, and relief of symptoms following glucose administration) is a characteristic finding. Symptoms often occur during fasting. Serum insulin and C-peptide levels are usually elevated. Low serum C-peptide levels associated with high serum insulin levels suggest surreptitious insulin use.
TA B L E 1 4 Functional Endocrine Tumors of the Pancreas and Duodenum

Alexander HR, Fraker DL, Norton JA, Bartlett DL, Tio L, Benjamin SB, et al. Prospective study of somatostatin receptor scintigraphy and its effects on operative outcome in patients with Zollinger-Ellison syndrome. Ann Surg. 1998;228:228-38. PMID: 9712569 Jensen RT. Carcinoid and pancreatic endocrine tumors: recent advances in molecular pathogenesis an, localization, and treatment. Curr Opin Oncol. 2000;12:368-77. PMID: 10888424 Zimmer T, Stolzel U, Bader M, Koppenhagen K, Hamm B, Buhr H, et al. Endoscopic ultrasonography and somatostatin receptor scintigraphy in the preoperative localization of insulinomas and gastrinomas. Gut. 1996;39:562-8. PMID: 8944566 Anderson MA, Carpenter S, Thompson NW, Nostrant TT, Elta GH, Scheiman JM. Endoscopic ultrasound is highly accurate and directs management in patients with neuroendocrine tumors of the pancreas. Am J Gastroenterol. 2000;95:2271-7. PMID: 11007228

Syndrome Gastrinoma (ZollingerEllison syndrome)

Hormone Gastrin

Clinical Findings Abdominal pain, diarrhea, multiple endocrine neoplasia type 1 (MEN 1) in 25% of patients Hypoglycemia

Location and Size Pancreas and duodenum (small, difficult to localize)

Malignant Metastatic 60%90% Higher incidence of metastasis in patients with sporadic tumors than in those with MEN 1 Do not metastasize At diagnosis, most patients have metastases to the liver and lymph nodes Liver

Insulinoma Glucagonoma

Insulin Glucagon

Pancreas (solitary, <2 cm in diameter) 4D syndrome (diabetes Pancreas (usually >5 cm mellitus, dermatitis in diameter) [necrolytic migratory erythema], deep venous thrombosis, depression) Almost all arise from the pancreas (usually >3 cm in diameter) 75% arise from the pancreas (large mass); may arise from the duodenum, especially when associated with neurofibromatosis

<10% 60%80%

Vasoactive intestinal polypeptidesecreting tumor (VIPoma) Somatostatinoma

WDHA syndrome (watery diarrhea, hypokalemia, achlorhydria); fecal volume 68 L/day Somatostatin Abdominal pain, cholelithiasis, weight loss, diabetes mellitus, diarrhea, steatorrhea

Vasoactive intestinal polypeptide

50%

>70%

70% are metastatic at the time of diagnosis

29

Acute Diarrhea

KEYPOINTS

Pancreatic endocrine tumors can be either functional or nonfunctional. Gastrinomas are found with nearly equal frequency in the pancreas and the duodenum. Less than 10% of insulinomas are malignant. Surgery is the primary treatment for pancreatic endocrine tumors.

Endoscopic ultrasonography is the primary imaging modality for diagnosing insulinomas. In contrast to other pancreatic endocrine tumors, insulinomas are associated with negative octreotide scans in approximately 50% of patients. MRI only rarely identifies insulinomas. Less than 10% of insulinomas are malignant and even fewer are associated with metastatic hepatic lesions. Treatment consists of controlling hypoglycemic symptoms followed by surgical resection if the tumor is localized. Diazoxide has been used for control of hypoglycemia. Octreotide has also been shown to control hypoglycemia and other symptoms in approximately 50% of patients.

Intestines
Acute Diarrhea
How is acute diarrhea defined? Which enteric infectious agents are most commonly implicated in causing acute diarrhea? Are certain populations more susceptible to infectious diarrhea? Which foodborne agents are most often implicated in the development of acute diarrheal syndromes? What are the noninfectious causes of acute diarrhea?

Acute diarrhea is generally defined as three or more loose bowel movements in a 24-hour period or any loose stool that contains blood. Symptoms must be present for less than 14 days to be considered acute. Acute diarrhea is a very common diagnosis in general practice. Persons who are very young, elderly, or immunosuppressed are particularly vulnerable. Because acute infectious diarrhea is usually transmitted by the fecaloral route, ingestion of food or water contaminated by infectious organisms may cause a major outbreak and may be associated with significant morbidity and even mortality. The pathophysiologic findings are usually caused by colonization of a pathogen in the intestines. When diarrhea is due to ingestion of preformed toxins, colonization is not required. In most cases, however, adherence of the pathogen to the intestinal epithelial wall is a crucial causative step.

Evaluation of the Patient with Acute Diarrhea


Patients generally present with either noninflammatory watery diarrhea or inflammatory diarrhea. The former is usually an acute, self-limited process characterized by large-volume watery stools without blood or fecal leukocytes. Examples of causative agents include enterotoxigenic Escherichia coli, clostridial and staphylococcal food poisoning, Giardia, Cryptosporidium, Vibrio cholerae, rotavirus, and Norwalk virus. Patients with inflammatory diarrhea have frequent, smaller-volume stools that are often bloody and contain leukocytes. Findings are usually accompanied by fever, abdominal pain, and tenesmus. Causative agents include Salmonella, Campylobacter, Shigella, Yersinia, enterohemorrhagic and enteroinvasive E. coli, Clostridium difficile, and Entamoeba histolytica (Ilnyckyj). Other common causes of acute diarrhea include various medications, inflammatory bowel disease, and travelers diarrhea (travelers diarrhea is discussed in the Infectious Disease Medicine syllabus). Case 4 Four family members had lunch at a restaurant. The parents and daughter ate fried chicken, and the 5-year-old son had a hamburger. They each had cole slaw, potato chips, and cookies. Twelve hours later, the son develops bloody diarrhea and is taken to an emergency department. He is dehydrated and has elevated blood urea nitrogen and serum creatinine levels. 30

Ilnyckyj A. Clinical evaluation and management of acute infectious diarrhea in adults. Gastroenterol Clin North Am. 2001; 30:599-609. PMID: 11586547

Chronic Diarrhea

This patient has a foodborne illness, most likely from eating hamburger. Foodborne illnesses may cause significant morbidity. A rapid onset of symptoms, usually within 6 hours of eating, suggests ingestion of a preformed toxin, such as Staphylococcus or Bacillus cereus. If the onset occurs after 8 hours, Clostridium perfringens should be considered. Symptoms that occur after eating poorly cooked beef suggest enterohemorrhagic E. coli (E. coli O157:H7), which may be associated with the hemolytic uremic syndrome (see Ilnyckyj). Laboratory studies may include examination for fecal leukocytes and culture and sensitivity testing. A new latex agglutination test for lactoferrin (a neutrophil product) is also sensitive and specific for diagnosing acute diarrhea. Testing for ova and parasites is not cost effective in the evaluation of acute symptoms unless the patient has recently traveled to certain endemic regions, was exposed to infants, is a homosexual male, or has AIDS. Colonoscopy and sigmoidoscopy have a limited role in the evaluation of acute diarrhea. Although the endoscopic appearance of C. difficile colitis is quite characteristic, the diagnosis is usually suspected on a clinical basis because of a history of antibiotic use or the occurrence of acute diarrhea in the hospitalized patient. The diagnosis is confirmed by finding C. difficile toxin in the stool. Colonoscopy or flexible sigmoidoscopy with biopsy may have a role in differentiating inflammatory bowel disease from acute infectious diarrhea. The pathologic findings in colonic biopsy specimens from patients with acute selflimited infection are usually more superficial than the findings in patients with inflammatory disorders. More than 700 drugs have been implicated in causing acute or chronic diarrhea. The most frequently involved are antibiotics, laxatives (including sorbitol, which is used to dissolve other medications, and lactulose, which is used to treat chronic liver disease), magnesium-containing antacids, nonsteroidal anti-inflammatory drugs, colchicine, prostaglandins (misoprostol), antineoplastic agents, antiarrhythmic drugs, antihypertensive agents, cholesterol-lowering medications, and anticholinergic agents.

Treatment of Acute Diarrhea


Because most acute diarrheal illnesses are self-limited, no specific therapeutic intervention other than rehydration is usually required. Commercially available rehydration solutions contain sugar because glucose facilitates the absorption of sodium. This glucosesodium co-transport mechanism is maintained in the setting of infectious or toxin-induced diarrhea. Antibiotic therapy may be considered if the prevalence of a certain infectious agent is known to be high in a community or a setting. Antibiotics should be considered in more severely ill patients, pending the availability of definitive test results. Opiate compounds such as loperamide and diphenoxylate with atropine are often prescribed. Some clinicians have questioned whether opiates slow the clearance of pathogens and contribute to the development of toxic megacolon. However, these concerns are not evidence-based. Other agents such as bismuth subsalicylate and adsorbents may also be helpful in treating acute diarrhea.
KEYPOINTS

Foodborne illnesses are a serious health problem and may be a significant cause of morbidity. Most acute diarrheal illnesses are selflimited and require no specific treatment. Acute diarrheal syndromes may present as either a watery noninflammatory diarrhea or a smaller-volume, highfrequency (often bloody) diarrhea that is due to an inflammatory process. Noninfectious causes of acute diarrhea include inflammatory bowel disease and various medications.

Chronic Diarrhea
How is chronic diarrhea defined? What are the most common causes of chronic diarrhea? Which diagnostic studies are appropriate for evaluating patients with chronic diarrhea? How are patients with chronic diarrhea managed? What recent changes have occurred in the treatment of HIV-infected patients who have chronic diarrhea?

31

Chronic Diarrhea

Fine KD, Schiller LR. AGA technical review on the evaluation and management of chronic diarrhea. Gastroenterology. 1999;116:1464-86. PMID: 10348832

Chronic diarrhea is defined by many parameters, and, as yet, there is no consensus definition (Fine and Schiller). A diarrheal condition is generally considered chronic after a bout of acute diarrhea has continued beyond its usual self-limited course. Diarrhea of at least 4 weeks duration (or perhaps 6 to 8 weeks duration) is usually defined as chronic. Increased stool frequency and liquidity are also usually part of the definition. Fecal weight has been used as a criterion in many studies. However, some patients may have increased fecal weight (above 300 g) with normal stool consistency, whereas others may have normal fecal weight with loose or watery stools. Diarrhea should be distinguished from irritable bowel syndrome, which includes abdominal pain in association with bowel changes as part of its definition. Diarrhea should also be differentiated from fecal incontinence, which is the accidental loss of stool that may occur with or without diarrhea. Chronic diarrhea has various causes (Table 15). Most chronic diarrheal syndromes can be classified as either osmotic or secretory, which is determined by measuring the stool sodium, potassium, and osmolality.

TA B L E 1 5 Common Causes of Chronic Diarrhea

Primarily Osmotic Carbohydrate malabsorption (lactose) Osmotic laxatives (magnesium, polyethylene glycol) Nonabsorbable agents (sorbitol, lactulose) Primarily Secretory Stimulant laxatives Congenital syndromes (e.g., chloridorrhea) Bacterial toxins Drugs Disordered motility (e.g., post-vagotomy, post-sympathectomy, autonomic neuropathy) Hyperthyroidism Neuroendocrine tumors Colon cancer Lymphoma Villous adenoma Addisons disease Epidemic diarrhea (Brainerd-type diarrhea) Primarily Malabsorption (Steatorrhea) Pancreatic insufficiency Inadequate bile acids Mucosal diseases (e.g., celiac sprue) Short bowel syndrome Bacterial overgrowth Lymphatic obstruction Primarily Inflammatory Ulcerative colitis Crohns disease Microscopic colitis Collagenous colitis Infectious diseases Ischemia Radiation injury

32

Chronic Diarrhea

The osmotic gap can then be calculated by the following formula: 290 2([Na+] + [K+]) where 290 mosm/kg is the estimated osmolality of stool in the distal intestine, which is used to equilibrate with plasma osmolality. Noting the stool osmolality is important because if the fecal sample is excessively dilute, the patient may have added water or diluted urine to the sample (Topazian and Binder). If a stool sample is excessively concentrated, the sample may be contaminated with concentrated urine, or malabsorption with bacterial fermentation of unabsorbed nutrients may be present. An osmotic gap of >125 mosm/kg indicates a pure osmotic diarrhea. If the gap is <50 mosm/kg, a secretory diarrhea is present. Mixed processes also occur.

Topazian M, Binder HJ. Brief report: factitious diarrhea detected by measurement of stool osmolality. N Engl J Med. 1994;330:1418-9. PMID: 8159195

Evaluation of the Patient with Chronic Diarrhea


The history of the patient with chronic diarrhea may provide clues as to its cause. Conditions such as diabetes mellitus, inflammatory bowel disease, thyroid disorders, and the postoperative state following gastrointestinal surgery may contribute to the development of chronic diarrhea. A travel and medication history should also be obtained. Specific diagnostic tests using randomly collected fecal samples may also be helpful. These include Sudan stain for fat, examination for ova and parasites, measurement of pH (a pH <5.3 is characteristic of carbohydrate malabsorption), determination of electrolytes, and surveys for laxative abuse. Although most bacterial infections cause acute diarrhea, Plesiomonas and Aeromonas infections can cause a chronic disorder. The sensitivity and specificity of guaiac testing (fecal occult blood testing) for inflammatory and cancerous conditions that cause chronic diarrhea has not been determined (see Fine and Schiller). Testing for fecal leukocytes and a newer latex agglutination test for lactoferrin (a neutrophil product) are sensitive and specific for diagnosing the cause of acute diarrhea, but the usefulness of these tests for evaluating chronic diarrhea is currently unknown. Quantitative stool analyses for weight, osmolality, electrolytes, and fat may also be helpful in categorizing the diarrhea and determining the cause. Testing for protein-losing enteropathy may include a test for stool 1-antitrypsin concentration. Measurement of fecal clearance of 1-antitrypsin may provide a more accurate assessment. If a secretory process is suspected, other studies may include serum peptide hormone measurements; urine studies (for example, 5-hydroxyindoleacetic acid, vanillylmandelic acid); and serologic tests (for example, antiendomysial and antitransglutaminase antibodies, HIV). Flexible sigmoidoscopy or colonoscopy may be needed, and upper endoscopy may be useful for obtaining biopsy specimens from the small intestine when indicated. An upper gastrointestinal series with small bowel followthrough and barium enema examination may also be appropriate. Physiologic tests (for example, D-xylose test, Schilling test, hydrogen breath testing, and pancreatic function testing) may also be helpful when indicated.

Treatment of Chronic Diarrhea


Treatment of the patient with chronic diarrhea should be based on the underlying cause. Symptomatic therapy can be considered if a diagnosis is pending, if a diagnosis cannot be confirmed, or if the condition diagnosed does not have a specific treatment. Agents for symptomatic treatment include opiate-based medications, bile acidbinding agents, bismuth-containing medications, and fiber. A somatostatin analog such as octreotide may be beneficial in patients with secretory diarrhea.

33

Chronic Diarrhea

Treatment of the Patient with HIV Infection and Chronic Diarrhea


Chronic diarrhea is a common problem for patients with HIV infection. The lifetime incidence is approximately 50% for HIV-infected patients living in North America, and the development of diarrhea is closely correlated with a drop in the CD4 cell count below 50/L. The most common opportunistic pathogens in patients with CD4 cell counts below 200/L are Microsporidium, Cryptosporidium, Mycobacterium avium complex, and cytomegalovirus. Other less common pathogens include Isospora belli, Cyclospora, Histoplasma, and herpes simplex virus. The American Gastroenterological Association has published guidelines for the evaluation of patients with chronic HIV-related diarrhea (Wilcox et al.). In addition to a careful history and physical examination, the first step is to determine whether the onset was temporally related to starting a protease inhibitor because about 10% to 50% of patients who take these agents develop diarrhea (Oldfield). Multiple stool samples are then obtained for routine culture for pathogens, examination for ova and parasites, assay for Clostridium difficile toxin, smear for acid-fast bacilli (for detection of M. avium complex, Mycobacterium tuberculosis, and cryptosporidia), enzyme immunoassay for cryptosporidia, and Webers modified trichrome stain for microsporidia. If the patient is febrile, blood cultures, including cultures for M. avium complex, should be performed. If these tests do not establish the diagnosis, full colonoscopy with ileoscopy should be considered. Endoscopic examination is most useful for patients with fever, weight loss, and a CD4 cell count of less than 200/L (especially a count of less than 50/L). Although controversial, many clinicians recommend full colonoscopy rather than flexible sigmoidoscopy because the latter fails to detect cytomegalovirus in as many as 39% of patients (Dieterich and Rahmin). In addition, flexible sigmoidoscopy does not allow cannulation of the ileum to detect microsporidia, cryptosporidia, and other pathogens. If the diagnosis is still in doubt, upper endoscopy with aspiration and biopsy may be necessary. Treatment of chronic diarrhea in patients with HIV infection has changed significantly in recent years. The use of highly active antiretroviral therapy (HAART) has improved the prognosis of HIV-infected patients and has also improved the outcomes of those with chronic diarrhea. Diarrhea resolves in up to 87% of patients whose CD4 cell counts increase by at least 50/L after initiation of HAART (Bini and Cohen). Many patients who have diarrhea secondary to intestinal pathogens previously thought to be untreatable (for example, microsporidia, cryptosporidia) have marked improvement following immune reconstitution with HAART.

Wilcox CM, Rabeneck L, Friedman S. AGA technical review: malnutrition and cachexia, chronic diarrhea, and hepatobiliary disease in patients with human immunodeficiency virus infection. Gastroenterology. 1996;111:1724-52. PMID: 8942756 Oldfield EC 3rd. Evaluation of chronic diarrhea in patients with human immunodeficiency virus infection. Rev Gastroenterol Disord. 2002;2:176-88. Review. PMID: 12481169 Dieterich DT, Rahmin M. Cytomegalovirus colitis in AIDS: presentation in 44 patients and a review of the literature. J Acquir Immune Defic Syndr. 1991;4(Suppl 1): S29-35. PMID: 1848619 Bini EJ, Cohen J. Impact of protease inhibitors on the outcome of human immunodeficiency virus-infected patients with chronic diarrhea. Am J Gastroenterol. 1999;94:3553-9. PMID: 10606318

KEYPOINTS

Most chronic diarrhea syndromes can be classified as either secretory or osmotic by measuring the osmotic gap. Testing should be geared toward the most likely diagnosis and may include stool studies, serologic testing, physiologic studies, and endoscopic examination with mucosal biopsy. Treatment should target the underlying cause; treatment may also be considered for symptomatic relief. In many HIV-infected patients, chronic diarrhea resolves after initiation of highly active antiretroviral therapy.

Treatment of the Patient with Diabetes Mellitus and Chronic Diarrhea


Treatment of the diabetic patient with chronic diarrhea can be very complex. In addition to the more usual causes of diarrhea, several unique circumstances contribute to chronic diarrhea in patients with diabetes mellitus. These patients may have regular dietary intake of sorbitol, which often acts as an osmotic laxative. Stopping the sorbitol may remedy the diarrhea. Patients with diabetes mellitus may also have autonomic dysfunction or motility disorders that can result in bacterial overgrowth, which may respond to oral antibiotic agents.

34

Constipation

Constipation
What are the most common causes of constipation? What are the most appropriate studies for evaluating patients with constipation? What treatment options are currently available for patients with constipation?

Constipation is defined as the decreased frequency of bowel movements and usually refers to fewer than three bowel movements per week. Patients may also describe hard stool, difficulty defecating, or the sensation of incomplete evacuation. Constipation may be due to colorectal dysmotility (slow-transit constipation, pelvic floor dysfunction) or irritable bowel syndrome (constipation-predominant irritable bowel syndrome) or may be secondary to systemic or medication-induced disorders (Table 16) (Locke et al.; Wald).

Slow-Transit Constipation
The number of patients with slow-transit constipation (colonic inertia) depends on sample and population characteristics (Bharucha and Phillips). However, most patients with colonic inertia are young women. Self-reported stool frequency does not correlate well with actual transit times. Slow transit refers to ineffective or slow colonic propulsion of stool contents. Patients may have isolated colonic inertia or a motility disorder involving other proximal portions of the gastrointestinal tract that may be associated with a systemic disease such as progressive systemic sclerosis (scleroderma) or diabetes mellitus. High-amplitude propagated contractions (HAPCs) are thought to be responsible for the mass movement of stool contents in the colon. The cause of colonic inertia is unknown; however, patients with slow-transit constipation have fewer HAPCs than normal controls. How these reduced HAPCs contribute to the development of colonic inertia is unclear. It is also unclear whether this disorder is caused by a reduced HAPC response to a meal. Diagnostic studies for colonic inertia include radiopaque marker studies and scintigraphy. Several protocols are available for colon marker studies, but the basic principle is having the patient ingest a number of radiopaque markers over a period of time and following the presence or absence of these markers on an abdominal radiograph or series of radiographs. The treatment of slow-transit constipation may involve a trial of fiber therapy. Osmotic laxatives such as lactulose, sorbitol, milk of magnesia, and polyethylene glycol may also be used. Stimulant laxatives should be avoided except as a last resort. Colchicine and misoprostol are sometimes prescribed because these agents may cause diarrhea or frequent bowel movements when administered for their usual indications. Surgical procedures, such as colectomy, are reserved for patients who have refractory symptoms but have no other signs or symptoms of a motility disorder in other parts of the gastrointestinal tract (Knowles et al.).

Locke GR 3rd, Pemberton JH, Phillips SF. American Gastroenterological Association Medical Position Statement: guidelines on constipation. Gastroenterology. 2000;119:1761-6. PMID: 11113098 Wald A. Constipation. Med Clin North Am. 2000;84:1231-46. PMID: 11026926 Bharucha AE, Phillips SF. Slow transit constipation. Gastroenterol Clin North Am. 2001;30:77-95. PMID: 11394038

Pelvic Floor Dysfunction


Patients with pelvic floor dysfunction are unable to adequately evacuate stool from the rectum. The dysfunction may be functional or may be due to a structural disorder such as a rectocele. Anorectal dyssynergia is a functional disorder in which patients tend to tighten the anorectal angle and increase the pressure of both the puborectalis muscles and the external anal sphincter during defecation. This results in an obstruction to defecation. Patients may have a social or job-related history of deferred defecation. Some studies suggest that such patients may also have a history of physical and/or sexual abuse (Stark).

Knowles CH, Scott M, Lunniss PJ. Outcome of colectomy for slow transit constipation. Ann Surg. 1999;230:627-38. PMID: 10561086 Stark ME. Challenging problems presenting as constipation. Am J Gastroenterol. 1999;94:567-74. PMID: 10086634

35

Irritable Bowel Syndrome

TA B L E 1 6 Common Causes of Constipation

Irritable Bowel Syndrome (constipation-predominant) Colorectal Dysmotility Slow-transit constipation Pelvic floor dysfunction Disorders Limited to the Colon Neoplastic lesions Proctitis Anal fissures Strictures Systemic Disorders Metabolic Diabetes mellitus Hypothyroidism Hypercalcemia Neurologic Spinal cord lesions Parkinsons disease Multiple sclerosis Medication-induced Prescription medications Anticholinergic agents Anticonvulsant agents Antipsychotic agents Antihypertensive agents Calcium channel blockers Diuretics
KEYPOINTS

Constipation may be due to metabolic causes, colonic inertia (slow-transit constipation), pelvic floor dysfunction, or constipation-predominant irritable bowel syndrome or may be secondary to medications or a systemic illness. Slow-transit constipation primarily affects young women. Patients who fail to respond to carefully chosen medical therapy may be candidates for surgical management. Pelvic floor dysfunction may have a structural component (e.g., rectocele) or may be purely functional.

Dopaminergics Iron Opiates Sympathomimetic agents Tricyclic antidepressants Over-the-counter medications: Antacids (calcium, aluminum) Antihistamines Antidiarrheal agents

Evaluation may include anorectal manometry and balloon expulsion studies. Defecography may also be helpful. Treatment involves biofeedback and training to relax the pelvic floor muscles, which often restores normal defecatory coordination.

Irritable Bowel Syndrome


How is irritable bowel syndrome defined? Does the severity of symptoms affect the prognosis of patients with irritable bowel syndrome? What new medications are available for treatment of irritable bowel syndrome?

Irritable bowel syndrome is a functional gastrointestinal disorder characterized by abdominal pain associated with defecatory symptoms. The Rome II criteria 36

Irritable Bowel Syndrome

for diagnosing this disorder are listed in Table 17. Irritable bowel syndrome is one of the most common disorders for which patients consult primary care physicians. The pathophysiologic basis is not completely understood, but the three factors most often implicated are altered gastrointestinal motility, visceral hypersensitivity (heightened sensory awareness of the gastrointestinal tract), and psychosocial factors. There is a complex interrelationship of causative factors, symptoms, and outcomes. Three symptom subgroups have been described: constipation-predominant, diarrhea-predominant, and mixed or alternating constipation/diarrhea irritable bowel syndrome. This classification is useful when considering treatment because the three subgroups appear to respond differently to various therapeutic options (Whitehead). Patients have also been classified as having mild, moderate, or severe irritable bowel syndrome based on their clinical features at the time of presentation. Abdominal and psychological symptoms appear to be independently associated with impaired quality of life in patients identified as having severe irritable bowel syndrome (Creed et al.). The treatment of irritable bowel syndrome is challenging. Alosetron, a 5-HT3 receptor antagonist, was found to improve symptoms in women with diarrhea-predominant and alternating constipation/diarrhea irritable bowel syndrome (Camilleri). 5-HT3 receptors are nonselective cation channels whose activation affects visceral pain, colonic transit, and secretion. Alosetron was initially approved by the United States Food and Drug Administration in early 2000, but in November of that year the drug was withdrawn from the market because of its association with an increased incidence of ischemic colitis. Alosetron was re-released after an advisory panel restricted prescription of this drug to qualified experts. Alosetron should only be prescribed for female patients with severe diarrhea after ischemic bowel disease has been excluded. Tegaserod, a 5-HT4 receptor partial agonist, was approved by the United States Food and Drug Administration in 2002 for the treatment of constipation-predominant irritable bowel syndrome in women. Studies have shown that this drug is effective in relieving pain and constipation in these patients (Muller-Lissner et al.). Other medications, including anticholinergic and antispasmodic agents, have side effects and have not definitely been shown to be more effective than placebo for long-term treatment of irritable bowel syndrome. Tricyclic antidepressants are effective in some subgroups of patients with irritable bowel syndrome. A newer antidepressant class, selective serotonin reuptake inhibitors, may also be effective, particularly in patients with pain-predominant irritable bowel syndrome. Patients with predominant features of diarrhea or constipation may benefit from therapy for these specific symptoms. Although general dietary modification has usually been ineffective, a careful dietary history may identify foods that should be avoided. Fiber supplementation is frequently used and is most successful in patients with constipation-predominant irritable bowel syndrome. Psychotherapy, cognitive therapy, biofeedback, and hypnosis may also be helpful in subgroups of patients, especially those whose symptoms are refractory to medical treatment.
TA B L E 1 7 Rome II Diagnostic Criteria for Irritable Bowel Syndrome

Whitehead WE. Patient subgroups in irritable bowel syndrome that can be defined by symptom evaluation and physical examination. Am J Med. 1999;107(5A):33S-40S. PMID: 10588171 Creed F, Ratcliffe J, Fernandez L, Tomenson B, Palmer S, Rigby C, et al. Health-related quality of life and health care costs in severe, refractory irritable bowel syndrome. Ann Intern Med. 2001; 134(Pt 2):860-8. PMID: 11346322 Camilleri M. Management of the irritable bowel syndrome. Gastroenterology. 2001;120:652-68. PMID: 11179242 Muller-Lissner SA, Fumagalli I, Bardhan KD, Pace F, Pecher E, Nault B, Ruegg P. Tegaserod, a 5-HT(4) receptor partial agonist, relieves symptoms in irritable bowel syndrome patients with abdominal pain, bloating and constipation. Aliment Pharmacol Ther. 2001;15:1655-66. PMID: 11564007

KEYPOINTS

Irritable bowel syndrome is a common disorder characterized by abdominal pain and altered bowel habits. Disordered intestinal motility, visceral hypersensitivity, and psychosocial factors appear to play a role in the pathogenesis of irritable bowel syndrome. New treatment modalities targeting serotonin receptors that modulate motility and visceral sensation appear promising in the treatment of patients (especially women) with irritable bowel syndrome.

At least 12 weeks (which need not be consecutive) in the past 12 months of abdominal discomfort or pain associated with two of the following three features: Relieved with defecation and/or Onset associated with pain and a change in frequency of stool; and/or Onset associated with a change in form (appearance) of stool
Adapted from Thompson WG, Longstreth GF, Drossman DA, Heaton KW, Irvine EJ, Muller-Lissner SA. Functional bowel disorders and functional abdominal pain. Gut. 1999 Sep;45(Suppl 2):II43-7.

37

Malabsorption

Malabsorption
Which types of malabsorption can be diagnosed by small bowel biopsy? How are celiac sprue and tropical sprue treated?

Farrell JJ. Overview and diagnosis of malabsorption syndrome. Semin Gastrointest Dis. 2002;13:182-90. PMID: 12462704

The jejunum is the site of primary absorption of electrolytes and most nutritive substances (fats, proteins, carbohydrates), with the exception of iron, vitamin B12, and bile salts. Causes of fat malabsorption include, but are not limited to, pancreatic insufficiency, bile salt deficiency (inadequate micelle formation), impaired delivery of fat into the lymphatic circulation, and mucosal small bowel disease (Table 18). Various signs and symptoms are suggestive of malabsorption, including bulky, greasy, or foul-smelling stools, watery diarrhea, increased flatus, low -carotene levels, osteoporosis, and weight loss (Farrell). Routine laboratory tests should include studies to detect anemia (which may be the only abnormality found), hypocalcemia, and prolonged coagulation parameters. These studies should be supplemented by specific tests for malabsorption. Only a few tests that correspond to the specific presenting features are usually needed. A qualitative Sudan fat stain may suggest the diagnosis of fat malabsorption, but the gold standard remains a 72-hour stool collection showing a fat output of greater than 7% of intake (more than 7 g/d after ingestion of 100 g/d of fat). A low serum -carotene determination also correlates closely with fat malabsorption. Because upper endoscopy with mucosal biopsy establishes the specific diagnosis in most patients (Table 19), many tests previously used to differentiate the causes of malabsorption are no longer necessary. Other tests that may sometimes be helpful include the D-xylose test, the four-part Schilling test, and antibody tests for celiac disease.

TA B L E 1 8 Major Causes of Fat Malabsorption

Type Pancreatic insufficiency

Cause Chronic pancreatitis Pancreatic carcinoma Pancreatic resection Cystic fibrosis Intrahepatic or extrahepatic cholestasis Cholecystocolonic fistula Crohns disease Ileal resection See Table 19 Intestinal lymphangiectasia Lymphoma Zollinger-Ellison syndrome Bacterial overgrowth Systemic mastocytosis Carcinoid syndrome Diabetes mellitus Hypoparathyroidism Hyperthyroidism Adrenal insufficiency Collagen vascular disorders

Bile salt deficiency

Mucosal small intestinal disease Impaired delivery of fat from small intestine to lymphatic system Miscellaneous

38

Malabsorption

TA B L E 1 9 Biopsy Findings of Treatable Causes of Malabsorption

Cause Abetalipoproteinemia Acute radiation enteropathy Amyloidosis Bacterial overgrowth Celiac sprue Chronic radiation enteropathy Common variable immunodeficiency Crohns disease Eosinophilic enteritis Lymphangiectasia Lymphoma Parasites Systemic mastocytosis Tropical sprue Whipples disease

Biopsy Findings Normal villi Cytoplasmic droplets that stain for fat (acetic acid and Sudan III stain) Decreased mitoses in crypts; superficial ulceration Deposits of green birefringence with polarized light after staining with Congo red Mononuclear cell infiltrate that responds to antibiotics Flattening of villi Infiltration with lymphocytes and plasma cells Connective tissue and vascular proliferation Findings similar to those in celiac sprue, but without plasma cells Giardia often present on brush border Noncaseating granulomas, transmural inflammation, microscopic skip areas Eosinophils Dilated lymphatic vessels that resolve when dietary long-chain fatty acids are replaced with medium-chain fatty acids Malignant lymphocytes Giardia, Isospora, Strongyloides, Schistosoma Mast cells Villous atrophy with plasma cells Periodic acid-Schiffpositive macrophages Bacilli on electron microscopy

Tropical Sprue
Case 5 A 68-year-old woman has a 3-year history of watery diarrhea and weight loss of 13.5 kg (30 lb). She recently came to the United States from Santo Domingo. Diagnostic studies show steatorrhea and an abnormal D-xylose test. A small bowel biopsy shows flattened villi and a lymphocytic and plasma cell infiltrate in the lamina propria. Celiac sprue and tropical sprue are disorders of the small intestine that have similar presentations and intestinal biopsy findings. Tropical sprue is an infectious disease contracted by persons who reside in or travel to specific regions in the tropics. Causes are believed to be overgrowth of coliform bacteria and folic acid deficiency. Therapy includes broad-spectrum antibiotics and folic acid. The patient in Case 5 had tropical sprue, which resolved after treatment with tetracycline and folic acid.

Celiac Sprue
Celiac sprue is a T-cellmediated inflammatory reaction to gluten, a protein found in wheat, barley, rye, and oats (Trier). Small intestinal biopsy specimens show villous blunting and lymphocytic and plasma cell infiltrates. Patients have malabsorption of calcium, iron, and folic acid, but not all patients have diarrhea. The disorder may be associated with dermatitis herpetiformis, splenic atrophy without Howell-Jolly bodies, human leukocyte antigen (HLA)-DR3 and HLADQw2, and an increased risk of developing lymphoma and adenocarcinoma of the small intestine. The finding of serum IgA antiendomysial or antitransglutaminase antibodies is often diagnostic of celiac sprue in patients presenting with diarrhea, weight loss, and nutritional deficiencies. Although antibody testing is at least 85% sensitive and 99% specific for diagnosing celiac sprue (Sollid and Scott), a small
Trier JS. Diagnosis of celiac sprue. Gastroenterology. 1998;115:211-6. PMID: 9649477 Sollid LM, Scott H. New tool to predict celiac disease on its way to the clinics. Gastroenterology. 1998;115:1584-6. PMID: 9834289

39

Inflammatory Bowel Disease

bowel biopsy is still needed for confirmation before initiating a strict gluten-free diet because false-negative antibody tests may occur in patients who are already on a low-gluten diet. IgA antibodies may also be low in patients with concurrent IgA deficiency. However, IgA-deficient individuals are at risk of developing celiac sprue, and IgG antibodies are typically positive in these patients. A gluten-free diet is helpful for most patients, although some patients may tolerate oats. Intestinal biopsy findings and serum antibody levels generally return to normal in patients who adhere to the diet. Patients who have severe refractory celiac sprue may require treatment with corticosteroids or other immunosuppressive agents.

Bacterial Overgrowth
Bacterial overgrowth may occur in patients with surgically created blind loops of bowel, large diverticula, strictures with stasis, fistulae, and motility disturbances, such as progressive systemic sclerosis (scleroderma), chronic intestinal pseudo-obstruction, and diabetes mellitus. It may also occur following ileectomy. Fat malabsorption is believed to occur from the bacterial deconjugation of bile salts, the concentration of which falls below the levels needed for micelle formation. Carbohydrate and vitamin B12 absorption is also abnormal, and the serum folic acid level may be high. Symptoms of bacterial overgrowth are related to the underlying disease and to associated malabsorption. The diagnosis is confirmed by a glucose hydrogen breath test in which orally administered glucose causes a rapid rise in breath hydrogen as a result of bacterial fermentation. Intermittent antibiotic therapy provides temporary relief.

Short Bowel Syndrome


Vanderhoof JA, Langnas AN. Short-bowel syndrome in children and adults. Gastroenterology. 1997;113:1767-78. PMID: 9352883
KEYPOINTS

A 72-hour fecal fat collection is the gold standard for detecting fat malabsorption. Celiac sprue is treated with a glutenfree diet, whereas tropical sprue responds to antibiotics and folic acid. Bacterial overgrowth and short bowel syndrome are other common causes of malabsorption.

Short bowel syndrome may develop after resection of the small intestine for extensive Crohns disease or after jejunoileal bypass for morbid obesity. The degree of resulting malabsorption depends on the extent and site of resection and the adaptive function of the remaining bowel. Adaptation of villous lengthening and epithelial hyperplasia can take up to 2 years after resection or bypass. Some nutritional deficiencies may be ameliorated following adaptation, but continued replacement of electrolytes, zinc, magnesium, calcium, and vitamins A, B12, D, E, and K is generally required (Vanderhoof and Langnas). Total parenteral nutrition is often needed if less than 60 to 90 cm of small intestine is anastomosed to the colon or less than 150 cm of small intestine ends in a stoma. Enteral tube feedings may avoid the many complications and expense of total parenteral nutrition. Administration of cholestyramine should be avoided because it exacerbates fat malabsorption by disrupting the enterohepatic circulation of bile salts and depleting the bile salt pool.

Inflammatory Bowel Disease


How are ulcerative colitis and Crohns disease differentiated? How does smoking affect disease activity in patients with ulcerative colitis and Crohns disease? What is the initial therapy for patients with mildly or moderately active ulcerative colitis or Crohns disease? What is the role of corticosteroids in treating patients with inflammatory bowel disease? What new medications are available for treating patients with inflammatory bowel disease? How should the increased cancer risk in patients with ulcerative colitis and Crohns colitis be managed?

40

Inflammatory Bowel Disease

Inflammatory bowel diseases are associated with an exaggerated mucosal immune response in the gastrointestinal tract. The two most common inflammatory bowel disorders are ulcerative colitis and Crohns disease. Ulcerative colitis is characterized by chronic inflammation of the colonic mucosa and submucosa with occasional backwash ileitis, whereas Crohns disease is manifested by inflammation of the four intestinal mucosal layers (most commonly in the ileum and/or colon but also in the more proximal intestine). In genetically predisposed individuals, an inflammatory response is initiated by putative environmental triggers that are not controlled by normal homeostatic mechanisms (Fiocchi). The genetic susceptibility to inflammatory bowel disease is complex, which suggests involvement of multiple genes. The first major Crohns disease susceptibility gene, NOD2/CARD15, was recently identified in a small subset of patients with Crohns ileitis (Cuthbert et al.). Inflammation in inflammatory bowel disease is characterized by an imbalance in activity between pro-inflammatory mediators (for example, interleukin-1, tumor necrosis factor-, thromboxane A2) and anti-inflammatory mediators (for example, interleukin-1 receptor antagonist, interleukin-10, prostaglandin E2). Immunoregulatory cytokines such as interleukin-2 regulate the process of lymphocyte differentiation and function, which also appear to be imbalanced in patients with inflammatory bowel disease. T-helper null (Th0) cells differentiate into Th1 cells, which regulate cellular immunity and are especially important in the development of Crohns disease, and into Th2 cells, which regulate humoral immunity and are thought to be important in the development of ulcerative colitis. Cyclosporine, which is given only to patients with severe ulcerative colitis, reversibly inhibits expression of interleukin-2 and may minimize both Th1 and Th2 cellular responses. Case 6 A 24-year-old man comes to the emergency department because of a 3-week history of bloody diarrhea, abdominal cramping, and tenesmus. He has not traveled recently and stopped smoking 2 months ago. Physical examination discloses diffuse leftsided abdominal tenderness, and initial routine laboratory studies show leukocytosis. Flexible sigmoidoscopy is performed, which shows continuous inflammation from the rectum to the splenic flexure. An assay for Clostridium difficile toxin is negative, and all stool cultures subsequently show no growth. Both ulcerative colitis and Crohns disease have a bimodal incidence curve, with the larger peak occurring at about 20 years of age and the smaller peak at about 50 years of age, but either may occur at any age. Both diseases affect men and women equally and are recurring and remitting disorders. Aside from these similarities, ulcerative colitis and Crohns disease can be differentiated clinically, endoscopically, pathologically, and serologically in more than 90% of patients. The remaining patients are considered to have an indeterminate inflammatory bowel disease. Patients with Crohns disease often present with diarrhea, right lower quadrant abdominal pain with an inflammatory mass, fever, and weight loss. Acute appendicitis is sometimes suspected. Patients often have aphthous or discrete linear ulcerations with discontinuous or skip areas of inflammation. Crohns disease affects the terminal ileum, the colon (especially the right side), or both the ileum and the colon. Noncaseating granulomas are found in about 30% of patients and more than 60% have anti-Saccharomyces cerevisiae antibodies (ASCA). Less than 10% of patients with Crohns disease have perinuclear antineutrophil cytoplasmic antibodies (p-ANCA), a percentage similar to that

Fiocchi C. Inflammatory bowel disease: etiology and pathogenesis. Gastroenterology. 1998;115:182-205. PMID: 9649475 Cuthbert AP, Fisher SA, Mirza MM, King K, Hampe J, Croucher PJ, et al. The contribution of NOD2 gene mutations to the risk and site of disease in inflammatory bowel disease. Gastroenterology. 2002;122:867-74. PMID: 11910337

41

Inflammatory Bowel Disease

Rubin DT, Hanauer SB. Smoking and inflammatory bowel disease. Eur J Gastroenterol Hepatol. 2000;12:855-62. PMID: 10958212 Robinson M. Optimizing therapy for inflammatory bowel disease. Am J Gastroenterol. 1997;92(Suppl):12S-17S. PMID: 9395347

of unaffected populations. Cigarette smoking is a strong risk factor for the onset and recurrence of Crohns disease. Patients with ulcerative colitis present with diarrhea and hematochezia. Endoscopic examination shows crypt abscesses and superficial inflammation extending from the rectum proximally through the colon. Up to 75% of patients with ulcerative colitis have positive p-ANCA assays. Smoking is inversely correlated with disease severity in patients with ulcerative colitis (as illustrated by the patient in Case 6), and nicotine may be therapeutic (Rubin and Hanauer).

Treatment of Patients with Inflammatory Bowel Disease


Treatment of patients with ulcerative colitis is determined by the activity of the disease (mild, moderate, or severe), and therapy for those with Crohns disease is decided by the behavior of the disease (fibrostenotic, inflammatory, or fistulizing) (Table 20) (Robinson).

Medical Treatment
Drugs used to treat inflammatory bowel disease include aminosalicylates, corticosteroids, immunomodulating agents, antibiotics, and new biologic agents such as infliximab. Although sulfasalazine was introduced more than 50 years ago to treat rheumatoid arthritis, its usefulness in treating inflammatory bowel disease was soon discovered. Sulfasalazine is composed of 5-aminosalicylic acid (5-ASA, mesalamine) and sulfapyridine joined by a diazo bond. The active antiinflammatory moiety is 5-ASA, which is released when the diazo bond is cleaved by reductases in colonic bacteria. Therefore, sulfasalazine is expected to be effective only for treating ulcerative colitis or Crohns colitis and is not useful for treating small bowel disease. Because sulfasalazine competitively inhibits the absorption of folic acid, patients taking this agent require folic acid supplementation. Most of the adverse effects of sulfasalazine are due to the sulfapyridine component. Sulfa-free 5-ASA agents have therefore been developed to deliver 5-ASA to specific regions of the gastrointestinal tract. These include 5-ASA enemas and suppositories (for the rectosigmoid colon), 5-ASA in a pH-sensitive coating for release in the colon, olsalazine (two 5-ASA molecules linked with a diazo bond for release in the colon), and 5-ASA in a time-release preparation for release in both the colon and the small intestine. Both mesalamine and sulfaTA B L E 2 0 Treatment Options for Ulcerative Colitis (Based on Disease Activity) and Crohns Disease

(Based on Disease Behavior) Ulcerative Colitis Disease Activity* Mild disease (<4 bowel movements daily; occasional blood in stool; normal vital signs, hemoglobin, and erythrocyte sedimentation rate) Moderate disease (intermediate between mild and severe disease) Severe disease (>6 bowel movements daily; bleeding; fever; pulse >90/min; anemia) Crohns Disease Disease Behavior Fibrostenotic disease Inflammatory disease Fistulizing disease
5-ASA = 5-aminosalicylic acid; 6-MP = 6-mercaptopurine *Disease activity criteria for ulcerative colitis adapted from Truelove SC, Witts LJ. Cortisone in ulcerative colitis; final report on a therapeutic trial. Br Med J. 1955;(4947):1041-8.

Principal Therapeutic Option(s) 5-ASA, nicotine 5-ASA, oral corticosteroids, 6-MP Intravenous corticosteroids; intravenous cyclosporine followed by oral cyclosporine; surgery

Principal Therapeutic Option(s) Surgery 5-ASA, intravenous or oral corticosteroids, 6-MP, methotrexate, infliximab, surgery 6-MP, metronidazole, fistulas only, infliximab, surgery

42

Inflammatory Bowel Disease

salazine can induce and maintain remissions in patients with inflammatory bowel disease. Among other activities, corticosteroids inhibit phospholipase A2 activity and reduce thromboxane A2 levels. They also inhibit the nuclear factor Bcytokine activating system. Corticosteroids quickly reduce inflammation in most patients with inflammatory bowel disease, which is generally followed by disease remission. The patient described in Case 6 responded dramatically to corticosteroid enemas. Corticosteroids have no proven role in maintaining remission, and long-term treatment is not advised because of the serious adverse effects associated with these agents. Budesonide, a corticosteroid available in both enema and ileal release forms, undergoes rapid first-pass metabolism and may have fewer side effects than traditional corticosteroids. Azathioprine and its metabolite 6-mercaptopurine (6-MP) are purine analogs that inhibit the synthesis of rapidly dividing inflammatory cells. Both are steroid-sparing and are effective in inducing and maintaining remission in patients with inflammatory bowel disease. The slow onset of action (average of 3 months) limits their usefulness in inducing remission, but long-term remission can be maintained at a dosage of 1.5 to 2.5. mg/kg daily. Important side effects include pancreatitis, allergic reactions, and dose-dependent leukopenia. Adjusting the dose to find the narrow window between therapeutic and toxic effects of 6-MP can be determined by measuring concentrations of the active 6-thioguanine nucleotide metabolites and a toxic 6-MP metabolite (Dubinsky et al.). Methotrexate, an antimetabolite, is effective for treating patients with active Crohns disease and for maintaining remission. The most significant side effects are interstitial pneumonitis and hepatic fibrosis. Cyclosporine may be effective for patients with severe ulcerative colitis who would otherwise require proctocolectomy. Antibiotics, particularly metronidazole, can be effective in subgroups of patients with Crohns disease, particularly those with fistula formation. Probiotics are currently being evaluated for treatment of inflammatory bowel disease. Ever-expanding knowledge of the pathophysiology of inflammatory bowel disease has allowed the development of novel biologic therapeutic agents such as infliximab, interleukin-1ra, interleukin-10 (an inhibitor of Th1 cell activity), and fish oil (eicosapentaenoic acid, an inhibitor of thromboxane A2 synthesis). Infliximab, a mousehuman monoclonal chimeric antibody to tumor necrosis factor-, has recently been approved for treatment of patients with fistulous Crohns disease (Present et al.). In addition to maintaining improvement in patients with the fistulous form of the disease, infliximab acts as a steroid-sparing agent and also results in rapid improvement in severe inflammatory Crohns disease and its extraintestinal manifestations (Targan et al.). Induction therapy is administered intravenously at 0, 2, and 6 weeks. Maintenance therapy is usually given every 8 weeks. Because infliximab has been associated with dissemination of tuberculosis, all patients should be screened for this infection prior to initiation of treatment. In addition, infliximab is contraindicated in patients with stricturing Crohns disease and in those with active infections (Sandborn and Hanauer). Other promising immunomodulator agents are being developed for treating patients with inflammatory bowel disease.

Dubinsky MC, Lamothe S, Yang HY, Targan SR, Sinnett D, Theoret Y, Seidman EG. Pharmacogenomics and metabolite measurement for 6-mercaptopurine therapy in inflammatory bowel disease. Gastroenterology. 2000;118:705-13. PMID: 10734022 Present DH, Rutgeerts P, Targan S, Hanauer SB, Mayer L, van Hogezand RA, et al. Infliximab for the treatment of fistulas in patients with Crohns disease. N Engl J Med. 1999;340:1398-405. PMID: 10228190 Targan SR, Hanauer SB, van Deventer SJ, Mayer L, Present DH, Braakman T, et al. A short-term study of chimeric monoclonal antibody cA2 to tumor necrosis factor alpha for Crohns disease. Crohns Disease cA2 Study Group. N Engl J Med. 1997; 337:1029-35. PMID: 9321530 Sandborn WJ, Hanauer SB. Infliximab in the treatment of Crohns disease: a users guide for clinicians. Am J Gastroenterol. 2002;97:2962-72. PMID: 12492177

Surgical Treatment
Patients with inflammatory bowel disease have an increased risk of developing colorectal cancer. This is especially true for those who have had extensive ulcerative colitis or Crohns colitis (involving most, if not all, of the colon) for at least

43

Ischemic Intestinal Disease

Provenzale D, Wong JB, Onken JE, Lipscomb J. Performing a cost-effectiveness analysis: surveillance of patients with ulcerative colitis. Am J Gastroenterol. 1998;93:872-80. PMID: 9647011
KEYPOINTS

Inflammatory bowel disease likely results from one or more environmental agents that initiate an inflammatory response in a genetically susceptible individual. The inflammation in inflammatory bowel disease is perpetuated by abnormal homeostatic mechanisms that create an imbalance between pro- and anti-inflammatory mediators. Smoking is a risk factor for the onset and recurrence of Crohns disease but is inversely correlated with disease activity in ulcerative colitis. Corticosteroids and 5-aminosalicylic acid are the mainstay of therapy for patients with inflammatory bowel disease; however, corticosteroids should not be used to maintain remission. Patients with extensive ulcerative colitis for at least 8 years require surveillance colonoscopy to detect possible colorectal cancer. Administration of infliximab has significantly improved the management of patients with Crohns disease, especially those with fistula formation.

8 years. To minimize cancer mortality, periodic surveillance colonoscopy every 1 to 2 years with extensive biopsies is recommended (Provenzale et al.). The finding of dysplasia, a premalignant neoplastic lesion, or cancer should prompt a recommendation for proctocolectomy. Other indications for surgery include severe active disease that is refractory to medical therapy, toxic megacolon, corticosteroid dependence, abscesses, fibrotic strictures, perforation, and fistula formation. In patients with ulcerative colitis, the anal sphincter can be spared during proctocolectomy by forming an ileal pouch that is double-stapled to the anal ring. Pouchitis (inflammation of the ileal pouch that develops in approximately 50% of patients by 5 years postoperatively) is usually treated successfully with metronidazole or with probiotics.

Extraintestinal Manifestations of Inflammatory Bowel Disease


Some extraintestinal manifestations of inflammatory bowel disease, such as nondeforming asymmetric polyarthritis, erythema nodosum, iritis, and uveitis, may be related to disease activity. These disorders often respond to treatment of the bowel disease. In contrast, primary sclerosing cholangitis, pyoderma gangrenosum, and central arthritis are not related to the activity of bowel disease, even in patients with ulcerative colitis who have had a colectomy. The presence of HLA-B27 in patients with inflammatory bowel disease is associated with uveitis and a spondyloarthropathy similar to either ankylosing spondylitis or sacroiliitis.

Ischemic Intestinal Disease


What are the most frequent causes of ischemic intestinal disease? How are ischemic intestinal diseases treated?

Kumar S, Sarr MG, Kamath PS. Mesenteric venous thrombosis. N Engl J Med. 2001;345:1683-8. PMID: 11759648

The clinical hallmark of intestinal ischemia is abdominal pain that is not explained by physical findings. The underlying cause is insufficient blood supply to the intestinal mucosa. The ischemia may be either nonocclusive or occlusive. Nonocclusive intestinal ischemia may be further defined as chronic or acute. Chronic mesenteric ischemia (intestinal angina) is usually caused by decreased blood flow as a result of atherosclerosis of the proximal mesenteric arteries. Typical features are postprandial abdominal pain, fear of eating because of anticipated pain, and weight loss. The diagnosis is suggested by magnetic resonance angiography, duplex Doppler ultrasonography, or contrast-enhanced CT scanning. Angioplasty or surgical revascularization is the most effective treatment. Mesenteric venous thrombosis causes 5% to 15% of all cases of intestinal ischemia (Kumar et al.). The disorder may be acute but may also cause subacute or chronic symptoms, leading to misdiagnosis. The superior mesenteric vein is most often involved. Predisposing factors include oral contraceptive use (8% to 18% of cases), acquired or inherited hypercoagulable states, recent abdominal surgery, and cirrhosis with or without portal hypertension. Although Doppler ultrasonography or CT scanning may establish the diagnosis, the most sensitive study is selective mesenteric angiography. If there is no evidence of bowel infarction, anticoagulation is the mainstay of medical management. Acute mesenteric ischemia is often caused by an embolism in the celiac artery or superior mesenteric artery as a result of valvular heart disease or atrial fibrillation. It may also be caused by arterial thromboses in a proximal vessel resulting from a low-flow state in a patient with a hypercoagulation disorder. Typical features are sudden and severe periumbilical pain, followed by nausea, vomiting, diffuse abdominal tenderness, leukocytosis, and ileus. Acute mesenteric ischemia is a lethal disease. Prompt recognition and immediate treatment

44

Diverticular Diseases

are critical. Angiography followed by thrombolysis or immediate surgery is the only option for preventing or treating bowel infarction (Klempnauer et al.). Ischemic colitis is also caused by a low-flow state in the mesentery but is usually not due to proximal vessel atherosclerosis. Predisposing factors include hypotension, dehydration, hypercoagulable states, and vasculitis. The relatively low blood flow to the colon, which is further decreased during motor activity, may be a predisposing factor in the development of colonic ischemia. The left side of the colon (the watershed area between the distribution of the middle colic and inferior mesenteric arteries) is most often affected. Patients usually present with hematochezia, diarrhea, and abdominal pain. Edematous haustral folds (thumbprinting) can be seen on abdominal radiographs. Colonoscopy may show a paucity of vessels, aphthoid ulcerations, and sharply demarcated areas of inflammation. Endoscopic differentiation from Crohns disease, infectious colitis, or even colon cancer may be difficult. Therapy usually requires correcting the predisposing factor, which is most often dehydration. The role of antibiotics is uncertain. Surgery or anticoagulation therapy is rarely needed.

Klempnauer J, Grothues F, Bektas H, Pichlmayr R. Long-term results after surgery for acute mesenteric ischemia. Surgery. 1997;121:239-43. PMID: 9068664

KEYPOINTS

Patients with chronic mesenteric ischemia (intestinal angina) with severe symptoms usually require angioplasty or surgical revascularization. Patients with acute mesenteric ischemia require emergency medical or surgical treatment. Patients with ischemic colitis can usually be managed with supportive care.

Diverticular Diseases
What factors correlate with the prevalence of diverticular disease? What are the different complications of diverticulosis? What diagnostic tests are available to localize diverticular bleeding?

Diverticular disease most commonly affects the colon. Small intestinal diverticula are less common and may be associated with underlying connective tissue disorders. Colonic diverticula are false or pulsion diverticula, meaning that they do not contain all four colonic layers as would be seen in congenital diverticula. The mucosa and submucosa herniate at points of the colonic wall where the circular muscle layer is interrupted by the vasa recta vessels. The proximity to the colonic wall vasculature explains the high incidence of bleeding from colonic diverticula. On autopsy series, the prevalence of diverticulosis in Western industrialized countries increased from 5% to 10% in 1918 to 35% to 50% in 1969; more recent prevalence data are not available. Prevalence increases with age (from about 5% at age 40 years to about 65% at age 85 years). Persons in Western industrialized countries have the highest rates of diverticulosis, and the disease predominantly affects the left side of the colon. In Asia and Africa, the prevalence is less than 0.2%, symptomatic diverticular disease is usually right-sided, and most patients are 40 years of age or younger (Morris et al.). In geographic areas such as Japan, Singapore, and Hong Kong, which have a more Westernized lifestyle, the prevalence of diverticulosis has markedly increased, although a much greater number of patients from these areas have right-sided disease. Although low dietary fiber has long been recognized to be associated with diverticular disease, complex interactions between colonic structure, motility, and diet are likely to be important in the pathogenesis of diverticulosis (Simpson et al.). For example, localized increases in intraluminal pressure may be due to abnormal motility, which is likely exacerbated by a low-fiber diet.

Morris CR, Harvey IM, Stebbings WS, Speakman CT, Kennedy HJ, Hart AR. Epidemiology of perforated colonic diverticular disease. Postgrad Med J. 2002;78:654-8. PMID: 12496319 Simpson J, Scholefield JH, Spiller RC. Pathogenesis of colonic diverticula. Br J Surg. 2002;89:546-54. PMID: 11972543

Complications of Diverticulosis
Approximately 70% of patients with diverticula will remain asymptomatic, 15% to 25% will develop acute diverticulitis (inflammation of the wall of the diverticulum ranging from a subclinical disorder to frank peritonitis), and 5% to 15% will develop diverticular bleeding. Only use of nonsteroidal anti-inflammatory

45

Diverticular Diseases

Goh H, Bourne R. Non-steroidal antiinflammatory drugs and perforated diverticular disease: a case-control study. Ann R Coll Surg Engl. 2002;84:93-6. PMID: 11995772

Ambrosetti P, Robert J, Witzig JA, Mirescu D, de Gautard R, Borst F, et al. Prognostic factors from computed tomography in acute left colonic diverticulitis. Br J Surg. 1992;79:117-9. PMID: 1555056

drugs has been definitely associated with an increased risk of complications from diverticulosis (Goh and Bourne). Diverticulitis may be classified as either simple or complicated. Simple diverticulitis accounts for approximately 75% of the complications of diverticulosis. Patients most often present with constant left lower quadrant abdominal pain of several days duration that is sometimes accompanied by nausea, vomiting, recent obstipation, and diarrhea. Physical examination generally discloses a tender left lower quadrant mass and low-grade fever, and leukocytosis may be present. A CT scan usually establishes the diagnosis. Characteristic CT findings include pericolic fat density (stranding), air-filled diverticula, bowel wall thickening, and soft-tissue masses, suggesting a phlegmon or abscess. Barium enema examination and colonoscopy are contraindicated because of the risk of rupturing previously contained diverticula. Most patients with simple diverticulitis will quickly respond to treatment with a liquid diet and antibiotics such as ciprofloxacin and/or metronidazole. Patients with moderate to severe symptoms often respond to complete bowel rest, intravenous fluids, and broad-spectrum antibiotics. Clinical improvement should be noted by 24 to 48 hours; if not, complicated diverticulitis may be present. About 15% of patients with simple diverticulitis require surgery. Approximately 25% of patients experiencing a first episode of diverticulitis develop complicated diverticulitis, defined as the presence of free or localized perforation, colonic obstruction, pericolonic abscess, or fistula formation. Free perforation with peritonitis should be treated with fluid resuscitation, broadspectrum antibiotics, and rapid surgical intervention. On barium enema examination, obstruction from diverticular disease often has a picket fence appearance or is seen as a constricting lesion resembling carcinoma. Resection with primary anastomosis is often possible for patients with colonic obstruction. A pericolonic abscess occurs in approximately 16% of patients with acute diverticulitis (Ambrosetti et al.). Broad-spectrum antibiotics and percutaneous catheter drainage may relieve symptoms and provide definitive treatment. More often, percutaneous catheter drainage is done prior to later elective single-stage surgery with resection and reanastomosis. Approximately 20% of all surgical procedures for complicated diverticulitis are done to repair fistulae. The most common locations for fistula formation are from the sigmoid colon to the bladder (65%) or from the sigmoid colon to the vagina (25%). Colo-enteric and colo-uterine fistulae are much less common. Surgical resection of the portion of the colon from which the fistula arises is usually necessary. Patients with complicated diverticulitis who undergo resection of the involved site have approximately a 15% rate of progression of diverticular disease in the remaining colon. However, only about 10% of these patients require a second surgical procedure. Diverticular bleeding occurs in about 15% of patients with extensive diverticulosis and is massive in about one third of these patients. Bleeding stops spontaneously in 75% of episodes, but the risk of rebleeding is 14% to 38%. Following a second episode, the risk of rebleeding increases to 50%. Most diverticular bleeding occurs from the right colon, even though 75% of diverticula are left-sided. This increased risk of bleeding from right-sided diverticula may be due to their wider ostia (which exposes the vasa recta for longer lengths of vessel) or to the thinner wall of the right colon. After ruling out an upper gastrointestinal source for the blood loss, diverticular bleeding is managed by fluid resuscitation and/or blood transfusion. Localization of the bleeding should be attempted for all patients except those who have such massive bleeding that prompt hemodynamic resuscitation is not possible. Appropriate preoperative diagnostic studies will identify the approxi-

46

Incidence Rates and Risk Factors for Colorectal Cancer

mate source of bleeding in 90% of patients. Radionuclide scans detect bleeding noninvasively, even when the rate of bleeding is as low as 0.1 mL/min. However, these scans only indicate a general site of bleeding and may not be able to distinguish small intestinal from colonic hemorrhage. Angiography is a more sensitive study for localizing the source of hemorrhage, even when bleeding rates are as low as 0.5 mL/min, and also allows therapeutic intervention with intra-arterial vasopressin or embolization. However, complication rates are much higher with angiography than with radionuclide scans. Colonoscopy also allows therapy by injection of epinephrine near the bleeding diverticulum or by coagulation of a visible vessel, although colonoscopic examination is less accurate than angiography in localizing the source of bleeding. Use of intraoperative colonoscopy increases accuracy. When the cause of bleeding has been localized, segmental colonic resection is performed. Postoperatively, long-term rebleeding is less than 15%. Because blind segmental resection is associated with a rebleeding rate of almost 50%, some surgeons perform subtotal colectomy when the source of bleeding has not been localized preoperatively.

KEYPOINTS

The development of diverticulosis is associated with a low-fiber diet; other causative factors may also be important. In Western industrialized countries, diverticula primarily develop in the left colon, but diverticular bleeding more frequently occurs from the right colon. Preoperative localization of the bleeding site markedly reduces rebleeding rates in patients undergoing colonic resection for diverticular bleeding.

Colorectal Cancer
Incidence Rates and Risk Factors for Colorectal Cancer
What is the incidence of colorectal cancer in the United States? What are the major environmental and hereditary risk factors for the development of colorectal cancer? Which two inherited syndromes are associated with a very high risk for colorectal cancer?

In the United States, colorectal cancer is more common than all other gastrointestinal cancers combined, and approximately 147,000 new cases are expected to be diagnosed in 2003. Incidence rates are slightly higher in men than in women and are higher in black persons than in white persons (Jemal et al.). Colorectal cancer is uncommon before 50 years of age. The incidence increases thereafter, with a lifetime risk of about 6%. Mortality rates have declined somewhat over the past 2 decades, so that the overall 5-year survival rate has risen from about 50% to 62%. Survival is related to the stage of the neoplasm (Table 21). Mortality rates from colorectal cancer are also higher among black persons, in part because the disease is often diagnosed at later stages in this population. Risk factors for colorectal cancer can be environmental or hereditary. Environmental risk factors include birth in North America, Western Europe, Australia, or New Zealand; a diet rich in animal fats, red meat, and sucrose; excessive alcohol consumption; tobacco use; obesity; and physical inactivity (Giovannucci). Advanced age and longstanding inflammatory bowel disease (both ulcerative colitis and Crohns colitis) are potent risk factors. Protective
TA B L E 2 1 Stage, Distribution, and 5-Year Survival of Persons

Jemal A, Murray T, Samuels A, Ghafoor A, Ward E, Thun MJ. Cancer statistics, 2003. CA Cancer J Clin. 2003;53:5-26. PMID: 12568441 Giovannucci E. Modifiable risk factors for colon cancer. Gastroenterol Clin North Am. 2002;31:925-43. PMID: 12489270

with Colorectal Cancer Stage Localized Regional (nodal) Distant metastases Distribution 37% 37% 20% 5-Year Survival 90% 65% 9%

Adapted from Jemal A, Murray T, Samuels A, Ghafoor A, Ward E, Thun MJ. Cancer statistics, 2003. CA Cancer J Clin. 2003;53:5-26.

47

Pathogenesis of Colorectal Cancer

Winawer S, Fletcher R, Rex D, Bond J, Burt R, Ferrucci J, et al. Colorectal cancer screening and surveillance: clinical guidelines and rationale-Update based on new evidence. Gastroenterology. 2003;124:544-60. PMID: 12557158

KEYPOINTS

Colorectal cancer is the most common gastrointestinal cancer in the United States. The lifetime risk of developing colorectal cancer is 6%. The overall 5-year survival rate for colorectal cancer is 62%. Risk factors for colorectal cancer include birth in North America, Western Europe, Australia, or New Zealand; a diet rich in fats, red meat, and sucrose; longstanding inflammatory bowel disease; and at least one first-degree relative with colorectal cancer. Familial adenomatous polyposis and hereditary nonpolyposis colorectal cancer are the two inherited syndromes associated with a very high risk of colorectal cancer.

factors include birth in Africa or Asia; a diet rich in fruits, vegetables, and fiber; high intake of methionine, folate, and calcium; use of postmenopausal hormone replacement therapy; long-term use of aspirin and nonsteroidal anti-inflammatory drugs; and vigorous physical activity. There is a 2- to 3-fold increased risk of colorectal cancer in persons with one first-degree relative who was diagnosed with this neoplasm when older than 50 years of age. The risk increases to 3- to 4-fold if two first-degree relatives were affected or if one first-degree relative was diagnosed with colorectal cancer when younger than 50 years of age (Winawer et al.). Inheritance probably determines a persons susceptibility for this cancer. Polymorphisms in genes involved in folate metabolism or mild mutations of genes that cause cancer syndromes account for some of this susceptibility. For example, about 6% of Ashkenazi Jews have a mild mutation of the adenomatous polyposis coli (APC) gene, and this mutation doubles an individuals risk of developing colorectal cancer. Inherited syndromes associated with a very high risk of colorectal cancer include familial adenomatous polyposis (FAP) and hereditary nonpolyposis colorectal cancer (HNPCC). Both syndromes have autosomal dominant inheritance. Persons with FAP usually have a mutation of the APC gene with truncation of the protein product. Affected persons develop hundreds to thousands of adenomatous colorectal polyps, and cancer is inevitable if the colon is not removed. Polyps occur early in adolescence, and colorectal cancer develops at a mean age of 39 years. Persons with FAP may also have numerous benign gastroduodenal polyps and connective tissue tumors. Individuals with HNPCC have only a few colonic polyps. However, their risk of developing colorectal cancer is quite high, with a mean age at diagnosis of 44 years. Other neoplasms associated with HNPCC include cancer of the endometrium, genitourinary tract, and stomach. The criteria for diagnosing HNPCC are shown in Table 22. The genes for FAP and HNPCC have been identified. Genetic tests for these conditions are available and should be used in conjunction with appropriate counseling (Calvert and Frucht).

Pathogenesis of Colorectal Cancer


Calvert PM, Frucht H. The genetics of colorectal cancer. Ann Intern Med. 2002;137:603-12. PMID: 12353948

How do colorectal cancers develop? Do all adenomatous polyps progress to colorectal cancer? Which genetic defects are associated with colon carcinogenesis?

Virtually all colorectal cancers arise from adenomatous polyps. Thirty percent to 50% of Americans older than 50 years of age will develop adenomatous polyps. However, only 1 in 20 adenomas will progress to cancer, and this progression usually takes 5 to 10 years. The cancer risk associated with adenomas is related to the size, number, villous architecture, and degree of cytologic dysplasia. Removal of adenomas by colonoscopic polypectomy reduces the expected incidence of colorectal cancer (see Winawer et al.).
TA B L E 2 2 Amsterdam Criteria for the Diagnosis of Hereditary

Nonpolyposis Colorectal Cancer Each of the following criteria must be met for diagnosing hereditary nonpolyposis colorectal cancer (HNPCC): Three first-degree relatives affected with an HNPCC-associated cancer Two generations affected with an HNPCC-associated cancer One individual affected with colorectal cancer before 50 years of age

48

Prevention of Colorectal Cancer

Genetics of Colon Carcinogenesis


The genetics of colon carcinogenesis involves sequential mutations that ultimately result in a cancer. An accumulation of genetic mutations leads to the progression from normal colonic epithelium to an adenoma and then to cancer (Figure 8). Mutations in oncogenes, tumor suppressor genes, and DNA repair genes are involved. Oncogenes, such as the K-ras oncogene, stimulate cell growth and, when mutated, give off unabated signals for continuous growth. Tumor suppressor genes, such as APC, p53, DCC (deleted in colorectal cancer), and DPC4 (deleted in pancreatic cancer) genes, may result in unregulated cell growth when mutated or lost. Mismatch repair (MMR) genes are a family of genes that are responsible for correcting mismatched nucleotide sequences during DNA replication. Mutation or loss of any of the MMR genes leads to an accumulation of mutations, including those that promote polyp enlargement and carcinogenesis. Epigenetic decreases of gene expression by promoter hypermethylation can mimic the effects of actual gene mutations by impairing the production of proteins important for colonocyte growth regulation. Mutation of the APC tumor suppressor gene is considered the earliest step in the developmental sequence of colon cancer. Mutation of this gene inhibits apoptosis (programmed cell death) and leads to the accumulation of an aberrant clone of cells. This clone develops into a polyp, and subsequent mutations of other genes (p53, DCC, K-ras, and MMR genes) result in progression to cancer. Germline mutations of the APC gene cause FAP with its dramatic phenotype. Acquired mutations of the APC gene cause a polyp in an average-risk individual only when both copies of the gene have a mutation or allelic loss (loss of heterozygosity). This unlikely event requires many years, which explains why sporadic adenomas occur at a later age. About 80% of sporadic adenomatous polyps have an APC gene mutation as the initial or very early step in development, but only 5% to 10% of such adenomas progress to cancer. Germline mutations of MMR genes result in HNPCC. About 15% of sporadic colorectal cancers also demonstrate loss of MMR gene function, which is usually due to promoter methylation rather than to gene mutation.

KEYPOINTS

Colorectal cancers develop from adenomatous polyps. Malignant degeneration of adenomas requires 5 or more years, and only 5% to 10% of sporadic adenomas progress to colorectal cancer. Removal of adenomas will markedly reduce the development of colorectal cancer. Carcinogenesis involves an accumulation of genetic mutations and epigenetic alterations.

Janne PA, Mayer RJ. Chemoprevention of colorectal cancer. N Engl J Med. 2000;342:1960-8. PMID: 10874065 Shaheen NJ, Straus WL, Sandler RS. Chemoprevention of gastrointestinal malignancies with nonsteroidal antiinflammatory drugs. Cancer. 2002;94:950-63. PMID: 11920463

Prevention of Colorectal Cancer


The National Cancer Institute has made recommendations for prevention of colorectal cancer. These recommendations are based on case-control studies and include consuming a diet high in fruits, vegetables, and fiber and low in fats; moderate intake of alcoholic beverages; and avoidance of tobacco products. Chemoprevention of colorectal cancer holds promise (Janne and Mayer). Sulindac and celecoxib cause regression of polyps in persons with FAP, but these drugs should not be considered as a substitute for colectomy. Cohort and case-control studies have shown that use of long-term aspirin and nonsteroidal anti-inflammatory drugs may decrease the incidence of and mortality from colorectal cancer (Shaheen et al.). The mechanism of action appears to be stimulation of apoptosis. Calcium and folic acid intake may also help prevent colorectal cancers.
DNA MMR genes Methylation
KEYPOINTS

Recommendations to prevent colorectal cancer include increased dietary fiber, fruits, and vegetables; moderate alcohol consumption; and avoidance of tobacco products. The incidence of and mortality from colorectal cancer may be decreased by use of long-term aspirin or nonsteroidal anti-inflammatory drugs, folic acid, and calcium.

APC

K-ras

DCC/DPC4

p53

FIGURE 8. Genetic mutations and colon carcinogenesis.


APC = adenomatous polyposis coli gene; DDC = deleted in colorectal cancer gene; DPC4 = deleted in pancreatic cancer 4 gene; K-ras = K-ras oncogene; MMR = mismatch repair genes; p53 = p53 tumor suppressor gene

Normal mucosa

Early adenoma

Intermediate adenoma

Late adenoma

Carcinoma

49

Screening and Surveillance for Colorectal Cancer

Screening and Surveillance for Colorectal Cancer


How effective is screening for colorectal cancer? What are the recommended approaches for colorectal cancer screening and surveillance? What new tests are being developed for colorectal cancer screening?

Screening refers to testing persons who have not been diagnosed with adenomas or colorectal cancer. Surveillance refers to follow-up of patients who have an increased risk for colorectal cancer, usually because of the presence of hyperplastic or adenomatous polyps.

Screening for Colorectal Cancer


Case 7 An asymptomatic 53-year-old female college professor comes for a routine evaluation. Medical history is noncontributory. The patient does not smoke, has a fairly sedentary lifestyle, and prefers a meat-and-potatoes diet with two glasses of wine each evening. She takes one aspirin tablet daily to protect her heart. There is no family history of colon cancer or polyps. General physical examination is normal. Rectal examination is also normal, and a stool specimen is negative for occult blood. Colorectal cancer screening with fecal occult blood testing and sigmoidoscopy detects adenomas and early-stage cancers and helps prevent deaths from cancer. Data based on several large prospective studies show that screening fecal occult blood testing is associated with a 33% decrease in colorectal cancer mortality if performed annually and an 18% decrease if performed biennially (Burt). Screening sigmoidoscopy with removal of observed lesions is associated with a 60% to 70% decreased mortality from cancer in the examined area of the bowel (see Winawer et al.). Colonoscopy is generally used for surveillance of the entire colon. Screening colonoscopy will detect colorectal cancer in up to 1% of otherwise healthy individuals over 50 years of age (Lieberman et al.; Imperiale et al.). Detection and removal of adenomas by colonoscopic polypectomy will prevent the development of approximately 75% to 90% of colorectal cancers.

Burt RW. Colon cancer screening. Gastroenterology. 2000;119:837-53. PMID: 10982778 Lieberman DA, Weiss DG, Bond JH, Ahnen DJ, Garewal H, Chejfec G. Use of colonoscopy to screen asymptomatic adults for colorectal cancer. Veterans Affairs Cooperative Study Group 380. N Engl J Med. 2000;343:162-8. PMID: 10900274 Imperiale TF, Wagner DR, Lin CY, Larkin GN, Rogge JD, Ransohoff DF. Risk of advanced proximal neoplasms in asymptomatic adults according to the distal colorectal findings. N Engl J Med. 2000;343:169-74. PMID: 10900275 Smith RA, Cokkinides V, Eyre HJ; American Cancer Society. American Cancer Society guidelines for the early detection of cancer, 2003. CA Cancer J Clin. 2003;53:27-43. PMID: 12568442 Pignone M, Rich M, Teutsch SM, Berg AO, Lohr KN. Screening for colorectal cancer in adults at average risk: a summary of the evidence for the U.S. Preventive Services Task Force. Ann Intern Med. 2002;137:132-41. Summary for patients in: Ann Intern Med. 2002;137:I38. PMID: 12118972

Screening the Average-Risk Patient


The American Cancer Society, the United States Preventive Services Task Force, and the GI Consortium Panel strongly endorse screening for colorectal cancer (Smith et al.; Pignone et al.; see alsoWinawer et al.). Screening should begin at 50 years of age. Several approaches may be used, including fecal occult blood testing every year, sigmoidoscopy every 5 years, annual fecal occult blood testing plus sigmoidoscopy every 5 years, colonoscopy every 10 years, or double-contrast barium enema examination every 5 years (Table 23). Fecal occult blood testing should be performed on three spontaneously passed stools, and samples should be obtained from two parts of each stool specimen (total of six samples). The patient should not take aspirin, nonsteroidal anti-inflammatory drugs, vitamin C, or iron prior to testing and should avoid rare red meat during the test period. The test cards should not be rehydrated prior to applying the developing solution because rehydration will increase the rate of false-positive tests and therefore decrease the specificity. Screening colonoscopy every 10 years or barium enema examination every 5 years has been recommended, and prospective studies regarding the effectiveness of screening colonoscopy are under way.

50

Screening and Surveillance for Colorectal Cancer

TA B L E 2 3 Screening Strategies for Colorectal Cancer

Risk Category Average risk

Age to Begin Screening 50 years of age

Recommended Test(s) Fecal occult blood testing annually or Sigmoidoscopy every 5 years or Fecal occult blood testing annually plus sigmoidoscopy every 5 years or Colonoscopy every 10 years or Double-contrast barium enema examination every 5 years

One first-degree relative with colorectal cancer or an adenomatous polyp at 60 years of age or Two second-degree relatives with colorectal cancer Two or more first-degree relatives with colorectal cancer or One first-degree relative with colorectal cancer or adenomatous polyps diagnosed at <40 years of age Patient at risk for familial adenomatous polyposis Patient at risk for hereditary nonpolyposis colorectal cancer

40 years of age

Same as average risk

40 years of age or 10 years younger than the age of the earliest patient in the family (whichever comes first) 1012 years old 2025 years of age or 10 years younger than the age of the earliest patient in the family (whichever comes first)

Colonoscopy every 5 years

Sigmoidoscopy annually Genetic counseling and testing Colonoscopy every 12 years Genetic counseling and testing

Patient with inflammatory bowel disease

Cancer risk increases after 8 years of pancolitis or after 1215 years of left-sided colitis

Colonoscopy every 12 years with multiple biopsies Referral to specialty center

The patient in Case 7 is at average risk for developing colorectal cancer and should undergo standard screening. She should be told that colorectal cancer screening can be done in several ways but that the preferred options are yearly fecal occult blood testing plus sigmoidoscopy every 5 years or colonoscopy every 10 years. A negative fecal occult blood test as part of a digital rectal examination is not a substitute for properly performed testing of a series of fecal test cards submitted by the patient.

Screening the High-Risk Patient


Persons at high risk for developing colorectal cancer are those with a family history of this neoplasm or of adenomatous polyps or a personal history of colorectal cancer, adenomatous polyps, or inflammatory bowel disease, as well as individuals who are known carriers of genes for FAP or HNPCC or who are otherwise at increased risk for these disorders. Screening recommendations are shown in Table 23.

New Colorectal Cancer Screening Approaches


New tests are being developed to screen for colorectal cancer, including CT colonography, immunochemical tests, and tests to detect fecal DNA mutations

51

Diagnosis of Colorectal Cancer

Levin B, Brooks D, Smith RA, Stone A. Emerging technologies in screening for colorectal cancer: CT colonography, immunochemical fecal occult blood tests, and stool screening using molecular markers. CA Cancer J Clin. 2003;53:44-55. PMID: 12568443 Pickhardt PJ, Choi JR, Hwang I, Butler JA, Puckett ML, Hildebrandt HA, et al. Computed tomographic virtual colonoscopy to screen for colorectal neoplasia in asymptomatic adults. N Engl J Med. 2003:349:2191-200. PMID: 14657426 Ahlquist DA, Skoletsky JE, Boynton KA, Harrington JJ, Mahoney DW, Pierceall WE, et al. Colorectal cancer screening by detection of altered human DNA in stool: feasibility of a multitarget assay panel. Gastroenterology. 2000;119:1219-27. PMID: 11054379

KEYPOINTS

Properly performed annual fecal occult blood testing may reduce deaths from colorectal cancer by approximately one third. Sigmoidoscopy may reduce deaths from distal colorectal cancers within reach of the sigmoidoscope by approximately two thirds. CT colonography (virtual colonoscopy) and noninvasive stool tests based on immunochemical detection of occult blood or DNA mutations appear promising as screening tests for colorectal cancer. The need for surveillance colonoscopy after colonoscopic removal of adenomas depends upon the findings at the initial colonoscopic examination.

(Levin et al.). Thin-section helical CT colonography, known popularly as virtual colonoscopy, is a developing technology that provides two- and threedimensional computer-generated images that resemble the views seen on conventional colonoscopy. This essentially noninvasive technique requires air insufflation of the colon using a rectal tube and still requires complete bowel cleansing. However, it avoids the need for sedation and is less time-consuming for the patient. When done by experienced radiologists, CT colonography is at least 90% sensitive for detecting large polyps and cancers. Sensitivity decreases for polyps measuring less than 5 to 6 mm. CT colonography is useful for visualizing the proximal colon after an incomplete colonoscopy. In a recent study that involved screening 1233 average-risk individuals, three-dimensional CT colonography and colonoscopy performed equally well in detecting clinically relevant lesions (Pickhardt et al.). Immunochemical tests for fecal occult blood testing use antibodies to detect human hemoglobin, whereas conventional guaiac-based tests rely on the peroxidase activity of hemoglobin. These new tests avoid the false-positive guaiac reactions caused by the ingestion of non-human hemoglobin or vegetable peroxidase. The more accurate results of the new tests should theoretically improve patient compliance. The sensitivity of immunochemical tests is 87% for colon cancer and 47% for large polyps, with a specificity of 98%. Immunochemical fecal occult blood testing appears to be equal to or better than guaiac-based tests, and largescale screening studies involving average-risk persons are in progress. Novel noninvasive screening tests have been designed that are based on detecting DNA mutations in the stool. In pilot studies using a multi-target stool assay panel that analyzed stool for several of the common genes altered in colon cancer, the sensitivity was 91% for detecting colorectal cancer and 82% for detecting large adenomas (Ahlquist et al.). A clinical trial comparing stool DNA testing with guaiac-based fecal occult blood testing in average-risk individuals will soon be completed. Capsule endoscopy involves swallowing a tiny camera (the size of a pill) that transmits images. It is useful for identifying enigmatic bleeding from the stomach and small intestine, but so far this technology has not been applied to colorectal cancer screening.

Surveillance of Adenomatous Polyps


If an adenomatous or hyperplastic polyp is found at screening sigmoidoscopy, full colonoscopy is warranted. Colonoscopic surveillance should be repeated in 3 years in patients with three or more adenomas or one advanced adenoma (a lesion that is larger than 1 cm, has villous histologic features, or shows highgrade dysplasia) (see Winawer et al.). If only one or two small adenomas are found at screening sigmoidoscopy or if the examination is negative or reveals only a hyperplastic polyp, colonoscopy may be extended to 5-year intervals. The intervals should be reduced for patients with numerous polyps, incomplete polypectomy, poor-quality examinations, or a malignant adenoma with invasive cancer. Surveillance should be discontinued when the patients longevity is expected to be less than 5 to 10 years or when the risks of the procedure outweigh the potential benefits.

Diagnosis of Colorectal Cancer


Case 8 A 67-year-old woman has a 3-month history of hematochezia. Medical history is significant for hemorrhoids and for a possible polyp detected by sigmoidoscopy 15 years ago. Physical exami52

Treatment of Colorectal Cancer

nation is normal. On rectal examination, the stool is brown and is negative for occult blood. Hematocrit is 36%, mean corpuscular volume is 71 fL, serum ferritin is 3 ng/mL, and the transferrin saturation is 10% (normal: 15% to 55%). At colonoscopy, the patient has a 5-cm nonobstructing ulcerating adenocarcinoma of the sigmoid colon. Two small adenomas are also removed. No hemorrhoids are noted. Adenomatous polyps rarely cause symptoms, and colorectal cancers are usually asymptomatic until they become unresectable. However, hematochezia, altered bowel habits, diarrhea, abdominal pain, and obstructive symptoms may occur. Diagnostic studies are indicated for all patients with symptoms or with positive screening tests. Men or postmenopausal women with iron deficiency anemia also require evaluation for colorectal cancer. Colonoscopy is the standard diagnostic study for colorectal cancer because it allows visual diagnosis, biopsy, and polypectomy and can detect at least 90% of large adenomas and cancers. Complications such as sedation problems, bleeding, and perforation are rare (0.03% to 0.6%) for colonoscopic diagnostic procedures and are only slightly higher (0.07% to 2.7%) for therapeutic procedures (see Pignone et al.). Barium enema examination is less effective because biopsy cannot be done, and the sensitivity of this study is only 50% for large polyps.

Treatment of Colorectal Cancer


What are the roles of surgery, chemotherapy, radiation therapy, and endoscopy for treatment of patients with colorectal cancer? What follow-up is appropriate for patients who have undergone resection of colorectal cancers?

Surgery is the principal therapeutic option for patients with colorectal cancer. Chemotherapy has an adjuvant role, especially for stage III colon cancer (nodal involvement) and may also be beneficial for rectal cancers and stage II tumors. Radiation therapy is useful for certain rectal cancers, and endoscopically directed therapies are also available. (Staging and treatment of colorectal cancer are discussed in greater detail in the Hematology/Oncology syllabus.) Most patients with colorectal cancer undergo surgical resection for cure or for palliation to debulk the lesion and to prevent or alleviate obstruction. Preoperative evaluation should include colonoscopy to detect synchronous polyps or cancers. A preoperative carcinoembryonic antigen determination should be obtained as a baseline for postoperative surveillance. CT scans and liver chemistry tests to screen for metastases usually do not alter the operative approach and are probably unnecessary. Endoscopic ultrasonography is the best available test for staging of rectal cancers. It provides an excellent view of the depth of tumor penetration into the rectal wall, local extension of the tumor, and involvement of local lymph nodes. In selected patients, endoscopic resection of large or high-grade adenomas may be curative. Palliative therapies include laser ablation of bleeding or obstructing tumors and placement of expandable metal stents for obstructing lesions. Relief of obstruction may allow an adequate bowel preparation so that a one-stage surgical procedure can be performed and a temporary colostomy can be avoided.

Management After Resection of Colorectal Cancer


The aim of management after resection of primary colorectal cancer is to detect recurrences and metachronous adenomas or cancers. Approximately 80% of recurrent colorectal cancers occur within 2 years of resection, and virtually all 53

Abnormal Liver Chemistry Studies in Asymptomatic Patients

KEYPOINTS

Surgery is the principal therapy for cure or palliation of colorectal cancer. Resection of operable liver metastases due to colorectal cancer can improve survival. Adjuvant chemotherapy and/or radiation therapy may be useful for treating invasion and nodal involvement in patients with colorectal cancer.

occur within 5 years of resection. There is no consensus about the best approach for detecting recurrent neoplasms, the majority of which occur outside of the bowel lumen. Survival may be prolonged by resection of operable liver metastases. Colonoscopy is performed primarily to detect metachronous adenomas and cancers and should be done within 3 years after resection and then, if normal, every 5 years thereafter.

Liver
Abnormal Liver Chemistry Studies in Asymptomatic Patients
What are the common hepatic causes of elevated serum aminotransferase values? Which pattern of liver chemistry abnormalities occurs in patients with hepatocellular damage? Which pattern of liver chemistry abnormalities occurs in patients with cholestatic liver injury? Which liver chemistry studies are used to assess hepatic synthetic function?

Pratt DS, Kaplan MM. Evaluation of abnormal liver enzyme results in asymptomatic patients. N Engl J Med. 2000;342:1266-71. PMID: 10781624

Bardella MT, Vecchi M, Conte D, Del Ninno E, Fraquelli M, Pacchetti S, et al. Chronic unexplained hypertransaminasemia may be caused by occult celiac disease. Hepatology. 1999;29:654-7. PMID: 10051464

The liver has a central role in many physiologic processes. Since the clinical manifestations of liver disease are variable, evaluation of liver chemistry studies is important for detecting liver damage in patients who are asymptomatic (Pratt and Kaplan) (Figure 9). Aspartate aminotransferase (AST) and alanine aminotransferase (ALT) are intracellular transaminating enzymes found in hepatocytes. After hepatocyte injury or death, these enzymes are released into the circulation, and their measurement in serum becomes a sensitive, although nonspecific, test of liver damage. Common causes of aminotransferase elevation include nonalcoholic fatty liver disease, alcoholic liver disease, viral hepatitis, autoimmune hepatitis, druginduced hepatoxicity, and metabolic disorders (Table 24). AST and ALT values greater than 10 to 15 times the upper limit of normal generally indicate acute hepatocellular necrosis caused by viral, drug-induced, or toxic injury or by hepatic ischemia. Viral, drug-induced, and toxic hepatic injuries generally cause prolonged aminotransferase elevations, whereas hepatic ischemia is suggested by recent hypotension and rapid normalization of elevated aminotransferase values over several days. Alcoholic hepatitis causes a modest aminotransferase increase (<300 U/L), with AST values being greater than ALT values, a finding that does not occur in most other types of hepatocellular injury. Cirrhosis is suggested by mild aminotransferase elevations (with AST values greater than ALT values) accompanied by an elevated serum bilirubin level, thrombocytopenia, and a prolonged prothrombin time. In one study, celiac sprue was found in 10% of asymptomatic patients who had elevated aminotransferase values (Bardella et al.).Therefore, when more common causes of aminotransferase elevations have been excluded in asymptomatic patients, testing for antiendomysial and antigliadin antibodies should be done to diagnose possible celiac sprue. Serum alkaline phosphatase activity is derived from liver, bone, intestine, and placenta. Elevated serum alkaline phosphatase levels may be caused by primary biliary cirrhosis, bile duct disease or obstruction, and infiltrative liver diseases (for example, metastatic disease and granulomatous disorders such as sarcoidosis). Cholestasis, a condition in which bile formation and/or flow is diminished, causes elevated serum alkaline phosphatase levels, in part because retained bile acids in the liver solubilize alkaline phosphatase from the hepato-

54

Abnormal Liver Chemistry Studies in Asymptomatic Patients

Evaluation of Patient with Elevated Liver Tests

AP/GGT Predominance

ALT/AST Predominance

Ultrasound

Initial Tests hepatitis A, B, C serology Ultrasound for fatty liver ANA, SMA, Fe/TIBC

(+) dilated ducts ERCP

normal ultrasound AMA Secondary Tests AAT phenotype ceruloplasmin antiendomysial antibodies

(-) AMA MRCP/ERCP if normal ANA + liver biopsy

(+) AMA primary biliary cirrhosis consider liver biopsy

FIGURE 9. Algorithm of evaluation of a patient with elevated liver test results.


AAT = 1-antitrypsin; ALT = alanine aminotransferase; AMA = antimitochondrial antibody; ANA = antinuclear antibody; AP = alkaline phosphatase; AST = aspartate aminotransferase; ERCP = endoscopic retrograde cholangiopancreatography; Fe/TIBC = serum iron and total iron-binding capacity; GGT = -glutamyltransferase; MRCP = magnetic resonance cholangiopancreatography; SMA = smooth muscle antibody

cyte plasma membrane. The enzymes 5'-nucleotidase and -glutamyltransferase are also released into the circulation of patients with cholestatic diseases. Elevations of these enzymes are used to confirm that an increased alkaline phosphatase level is of hepatic origin. Once confirmed, abdominal ultrasonography or abdominal CT scans should be obtained to evaluate the biliary system and liver parenchyma. A normal ultrasound examination or CT scan should be followed by measurement of antimitochondrial antibody, which, if positive, is highly suggestive of primary biliary cirrhosis. Patients with persistent alkaline phosphatase elevation despite a negative antimitochondrial antibody assay and normal ultrasonography or CT scans require endoscopic retrograde cholangiopancreatography or magnetic resonance cholangiopancreatography to evaluate possible primary sclerosing cholangitis. Liver biopsy should also be considered. In some patients, structural hepatobiliary disease may not be documented even though the increased alkaline phosphatase is of hepatic origin. The serum bilirubin concentration is determined by the balance between bilirubin production, conjugation, and excretion into bile. Hyperbilirubinemia has many causes, including hematologic disorders and hemolysis, congenital abnormalities of bilirubin conjugation, liver disease, and biliary obstruction. Elevated serum bilirubin levels do not fall immediately after biliary obstruction or hepatitis resolves because bilirubin binds covalently to albumin, which is slowly removed from the circulation. 55

Nonalcoholic Fatty Liver Disease and Nonalcoholic Steatohepatitis

TA B L E 2 4 Evaluation of Patients with Abnormal Liver Tests

Hepatocellular Injury ( Serum AST/ALT Serum Bilirubin) Obesity (nonalcoholic steatohepatitis) Drug-induced toxicity Hepatitis A, B, and C viral infections Autoimmune hepatitis Wilsons disease Hemochromatosis 1-Antitrypsin deficiency Celiac sprue Cholestatic Injury ( Serum Alkaline Phosphatase/-Glutamyltransferase Serum Bilirubin) Drug-induced toxicity

Evaluation Exclude other causes; evaluate fatty liver by abdominal ultrasonography Check use of medications, illicit drugs, herbs Obtain IgM anti-HAV, HBsAg, IgM HBc, anti-HCV Obtain antinuclear antibody titer, smooth muscle antibody titer, serum protein electrophoresis Obtain serum ceruloplasmin Obtain serum iron and total iron-binding capacity, serum ferritin Obtain 1-antitrypsin phenotype and level Obtain antiendomysial antibody titer, antigliadin antibody titer

Evaluation Check use of medications (phenothiazines, tricyclic antidepressants, erythromycin, trimethoprimsulfamethoxazole, amoxicillinclavulanate, androgens, estrogens) Obtain antimitochondrial antibody titer liver biopsy Obtain endoscopic retrograde cholangiopancreatography; magnetic resonance cholangiopancreatography Obtain antinuclear antibody titer + liver biopsy Obtain serum angiotensin converting enzyme + liver biopsy Obtain abdominal ultrasonography or CT scan + liver biopsy

Primary biliary cirrhosis Primary sclerosing cholangitis Autoimmune cholangitis Sarcoidosis Infiltrative/neoplastic disorders

AST = aspartate aminotransferase; ALT = alanine aminotransferase; IgM anti-HAV = IgM antibody to hepatitis A virus, HBsAg hepatitis B surface antigen; IgM HBc = IgM antibody to hepatitis B core antigen; anti-HCV = antibody to hepatitis C virus

KEYPOINTS

The most common causes of elevated serum aminotransferase levels in asymptomatic patients are fatty liver disease, hepatitis, and alcoholism. Elevated serum aminotransferase levels reflect hepatocellular injury or necrosis, whereas elevated serum alkaline phosphatase and -glutamyltransferase levels reflect cholestatic liver disease. The prothrombin time and the serum albumin level are used to assess hepatic synthetic function.

Gilberts syndrome is a common cause of unconjugated hyperbilirubinemia. Serum total bilirubin levels can reach 5 mg/dL. Patients with Gilberts syndrome have a normal life expectancy. Hepatic protein synthesis is assessed by the prothrombin time and serum albumin level. The prothrombin time is quickly affected by hepatic synthetic dysfunction because of the short half-life of coagulation factors. Determining the prothrombin time is therefore useful as a daily marker of hepatic synthetic function after vitamin K deficiency has been excluded. In comparison, albumin has a long half-life (20 days), and serum albumin levels change more slowly. Renal and gastrointestinal losses of albumin must be considered when evaluating a patient with hypoalbuminemia but with a normal prothrombin time.

Nonalcoholic Fatty Liver Disease and Nonalcoholic Steatohepatitis


What are the risk factors for nonalcoholic fatty liver disease (NAFLD) and nonalcoholic steatohepatitis (NASH)? What is the natural history of NAFLD and NASH? What are the characteristic hepatic pathologic findings in patients with NASH? What are the therapeutic approaches for patients with NASH?

Nonalcoholic fatty liver disease (NAFLD) includes a spectrum of clinicopathologic entities (excluding alcoholism as the name implies) that are characterized by fat droplets (macrovesicular steatosis) in hepatocytes. When hepatic steatosis is associated with polymorphonuclear leukocytes, lymphocytes, and hepatocellular injury surrounding central veins, the term nonalcoholic steatohepatitis (NASH) is used (Figure 10). Mallory bodies and pericentral and perisinusoidal 56

Nonalcoholic Fatty Liver Disease and Nonalcoholic Steatohepatitis

fibrosis may be present, similar to the pathologic findings in patients with alcoholic hepatitis (Brunt). The prevalence of adult obesity in the United States is currently about 30%. NAFLD and NASH are estimated to affect 20% to 40% of obese persons, although some patients with NAFLD may not be obese. In addition to visceral obesity, risk factors for NAFLD and NASH include other features of the metabolic syndrome, namely insulin resistance and diabetes mellitus, hypertriglyceridemia, and hypertension. Therefore, NAFLD can be considered the hepatic component of the metabolic syndrome. NAFLD is also associated with some drugs (for example, amiodarone, tamoxifen, diltiazem), lipoatrophy caused by antiretroviral agents, jejunoileal bypass for morbid obesity, and total parenteral nutrition. The molecular pathogenesis of NASH includes insulin resistance, genetic polymorphisms and susceptibility to hepatic inflammation, free fatty acid toxicity, and mitochondrial dysfunction (Sanyal et al.). However, NAFLD and NASH may occur in the absence of these risk factors. Although most patients with NASH are asymptomatic, some may develop fatigue and right upper quadrant abdominal discomfort. Diabetes mellitus or glucose intolerance is noted in 30% to 50% of patients, and hypertriglyceridemia is found in 20% to 80%. Serum aminotransferase levels are moderately elevated, with ALT levels being greater than AST levels. NAFLD is diagnosed by demonstrating hepatic steatosis radiographically. A definitive diagnosis of NASH can only be made by liver biopsy. The natural history of NAFLD is poorly defined, but patients with hepatic steatosis alone rarely develop advanced liver disease. However, 30% to 40% of patients with NASH have significant fibrosis, and 0.5% to 1% of individuals undergoing gastric bypass surgery for morbid obesity have cirrhosis. NASH is the underlying cause of cryptogenic cirrhosis in a significant number of patients (Bugianesi et al.). Treatment of NASH and NAFLD includes weight loss and pharmacologic management (Angulo). Gradual weight loss of approximately 1.3 kg (3 lb) per week improves hepatic histologic findings. Rapid weight reduction, such as weight loss due to fasting, results in increased portal inflammation and fibrosis despite resolution of hepatic steatosis. The optimal rate and amount of weight loss are unknown. Preliminary studies suggest that several pharmacologic agents may improve liver chemistry values and help repair hepatic injury. These include agents that target insulin resistance (for example, metformin and glitazones), antioxidants and cytoprotective compounds (for example, vitamin E, N-acetylcysteine, betaine, ursodeoxycholic acid), and gemfibrozil. Liver transplantation may be necessary for patients with advanced liver disease, but recurrence of NASH in the allograft has been described.

Brunt EM. Nonalcoholic steatohepatitis: definition and pathology. Semin Liver Dis. 2001;21:3-16. PMID: 11296695 Sanyal AJ, Campbell-Sargent C, Mirshahi F, Rizzo WB, Contos MJ, Sterling RK, et al. Nonalcoholic steatohepatitis: association of insulin resistance and mitochondrial abnormalities. Gastroenterology. 2001;120:1183-92. PMID: 11266382 Bugianesi E, Leone N, Vanni E, Marchesini G, Brunello F, Carucci P, et al. Expanding the natural history of nonalcoholic steatohepatitis: from cryptogenic cirrhosis to hepatocellular carcinoma. Gastroenterology. 2002;123:134-40. PMID: 12105842 Angulo P. Nonalcoholic fatty liver disease. N Engl J Med. 2002;346:1221-31. PMID: 11961152

KEYPOINTS

Nonalcoholic steatohepatitis (NASH) is defined as hepatic steatosis accompanied by hepatocellular injury and inflammation in a patient without chronic alcoholism. The risk factors for nonalcoholic fatty liver disease (NAFLD) and NASH include obesity, insulin resistance, hypertriglyceridemia, total parenteral nutrition, and certain drugs. The natural history of NAFLD is poorly defined, but a small percentage of patients develop cirrhosis. Approximately 30% to 40% of patients with NASH have significant fibrosis at the time of diagnosis. Treatment of NASH is directed toward slow but gradual weight loss and treatment of underlying medical conditions.

FIGURE 10. Nonalcoholic steatohepatitis. Liver biopsy specimen showing characteristic features of steatohepatitis: macro- and microvesicular fat droplets, ballooning hepatocyte injury, lymphocytic infiltrate, and a large Mallory body (arrow). (See Color Plate 5, at back of book.)

57

Alcoholic Liver Disease

Alcoholic Liver Disease


What are the hepatic manifestations of alcoholic liver disease? What are the risk factors for alcoholic liver disease? How is a patient with alcoholic hepatitis managed?

Akriviadis E, Botla R, Briggs W, Han S, Reynolds T, Shakil O. Pentoxifylline improves short-term survival in severe acute alcoholic hepatitis: a double-blind, placebocontrolled trial. Gastroenterology. 2000;119:1637-48. PMID: 11113085

KEYPOINTS

Alcoholic liver disease includes reversible fatty liver; fatty liver with necroinflammatory injury (alcoholic hepatitis), which is a precursor of cirrhosis; and micronodular cirrhosis. Risk factors for alcoholic liver disease include the daily amount (but not type) of alcohol consumed, duration of consumption, sex of the patient, and genetic susceptibility. Alcoholic hepatitis is treated with supportive care; pentoxifylline appears promising in treating patients with severe liver injury. Corticosteroids are used for patients with severe liver injury and encephalopathy who have no evidence of infection.

Alcoholic liver disease involves a spectrum of conditions that include fatty liver, alcoholic hepatitis, and micronodular cirrhosis. Alcoholic hepatitis, which is characterized histologically by hepatocellular necrosis, inflammation (polymorphonuclear leukocytes), and Mallory bodies (see Figure 10), is the precursor of cirrhosis. Fatty liver is reversible, and early alcoholic hepatitis may be reversible if the patient abstains from alcohol. Because only 10% to 15% of persons with chronic alcoholism develop liver disease, genetic and acquired factors other than those for alcoholism alone probably influence the susceptibility for liver injury. The risk for liver disease is determined in part by the amount of alcohol consumed daily, by the sex of the patient (women have a greater risk than men), and by the duration of alcohol ingestion. The threshold of alcohol consumption that results in liver injury is generally considered to be 60 to 80 g/d in men and 20 to 40 g/d in women. Alcoholic liver disease should be considered in any individual with relevant clinical and biochemical abnormalities who consumes alcohol on a regular basis. Patients usually have fever, leukocytosis, right upper quadrant abdominal pain, modest elevations of serum aminotransferases (with AST less than 300 U/L and AST values greater than ALT values), and increased -glutamyltransferase values (Table 25). Alcoholic liver disease may be life threatening. Corticosteroids reduce short-term mortality in patients with encephalopathy who have a discriminant function greater than 32 [serum total bilirubin + 4.6 (prothrombin time of patient minus control prothrombin time)]. A recent study reported that pentoxifylline reduced mortality from 46.1% to 24.5% and progression to hepatorenal syndrome from 34.6% to 8.2% compared with placebo in patients who had a high discriminant function irrespective of encephalopathy (Akriviadis et al.). Liver transplantation may be considered for patients with severe alcoholic liver disease if they have abstained from alcohol for at least 6 months and have an adequate support system to prevent relapse.

TA B L E 2 5 Features of Alcoholic Hepatitis

Risk Factors Sex of patient (male predominance) Alcohol consumption 6080 g/d in men 2040 g/d in women Duration of alcohol ingestion Genetic susceptibility Polymorphisms Tumor necrosis factorAlcohol metabolizing enzymes
DF = discriminant function (see text)

Histologic Findings Fat deposition Inflammation Cellular necrosis Mallory bodies

Therapy (Evidence for Therapy) Prednisone, 40 mg/d 28 days (meta analysis 12 randomized trials showed mortality reduction of 25% in patients with encephalopathy and DF >32) Pentoxifylline, 400 mg 3 times daily 28 days (one randomized, placebo-controlled trial of 101 patients with DF >32 showed 40% reduction in mortality and 65% reduction in progression to hepatorenal syndrome)

58

Acute and Chronic Hepatitis

Acute and Chronic Hepatitis


What are the common causes of acute hepatitis? What are the current management options for patients with acute viral hepatitis and acute drug-induced hepatitis? What is the natural history of chronic hepatitis B and C? What are the current management options for patients with chronic hepatitis B and C? Which drugs may cause chronic hepatitis?

Hepatitis is characterized histologically by hepatocellular necrosis and inflammatory cell infiltration of the liver. It can be either acute or chronic. Acute hepatitis implies a disorder of less than 6 months duration, and chronic hepatitis indicates a disease that has persisted for 6 or more months.

Acute Hepatitis
Acute hepatitis may be caused by viruses, drugs, and toxins. Regardless of the cause, symptoms and signs of acute hepatitis vary in frequency and intensity. The laboratory hallmark is elevation of serum aminotransferase levels. The clinical course ranges from asymptomatic disease to fulminant hepatic failure.

Acute Viral Hepatitis


Five hepatotropic viruses cause acute viral hepatitis: hepatitis A virus (HAV), hepatitis B virus (HBV), hepatitis C virus (HCV), hepatitis D virus (HDV, which only occurs together with HBV infection), and hepatitis E virus (HEV) (Table 26). Cytomegalovirus and EpsteinBarr virus also cause acute hepatitis but account for only a small proportion of cases. Data from the United States Centers for Disease Control and Prevention indicate that HAV infection accounts for 47% of all cases of acute viral hepatitis, HBV for 34%, and HCV for 16%. HAV and HEV are excreted in the feces prior to onset of symptoms and are transmitted by the fecaloral route. These viruses are implicated in most instances of waterborne and foodborne infections and in epidemics of viral hepatitis. HAV occurs worldwide. HEV is most often associated with outbreaks in East Asia, Central Africa, the Middle East, and Mexico. In endemic areas, HEV
TA B L E 2 6 Types of Acute Viral Hepatitis

Type Hepatitis A virus (HAV)

Nucleic Acid RNA

Route Fecaloral

Incubation Period 26 wk

Fulminant Hepatitis <1%

Case-Fatality Rate 0.3%

Chronic Disease No

Treatment Supportive

Prevention Universal precautions IG HAV vaccine Universal precautions HBIG HBV vaccine Universal precautions Universal precautions Universal precautions

Hepatitis B virus (HBV)

DNA

Parenteral

424 wk

<1%

0.8%

5%10% (in adults)

Supportive

Hepatitis C virus (HCV) Hepatitis D virus (HDV) Hepatitis E virus (HEV)

RNA

Parenteral

210 wk

Rare

70%80%

Supportive Interferonalfa? Supportive Supportive

RNA RNA

Parenteral Fecaloral

Similar to HBV 29 wk

Possible* <1%

? Overall? 10%20% in pregnant women

70% No

HBIG = hepatitis B immune globulin; IG = immunoglobulin *With HBV co-infection and superinfection.

59

Acute and Chronic Hepatitis

is particularly severe in pregnant women, in whom case-fatality rates of 10% to 20% have been reported. HBV, HCV, and HDV are primarily transmitted parenterally. HBV is present in virtually all body fluids and excreta of carriers. High-risk groups for acquisition of HBV infection include sexual partners of acutely and chronically infected persons, injection drug users, infants of infected mothers (vertical transmission), and health-care professionals. In the United States, most HBV infections are attributed to heterosexual contact. In other areas of the world, the major route of infection is vertical transmission. Prior to 1992, HCV was the main cause of post-transfusion hepatitis. It is currently the most common cause of hepatitis in injection drug users and accounts for a substantial number of cases of sporadic, community-acquired hepatitis. The risk of vertical or sexual transmission of HCV is much lower than that of HBV. Clinical and Laboratory Manifestations of Acute Viral Hepatitis Acute viral hepatitis typically begins with a prodromal phase that lasts several days and is characterized by constitutional and gastrointestinal symptoms, including malaise, fatigue, anorexia, nausea, vomiting, myalgias, and headache. Mild fever may also be present, especially in patients with HAV infection. Arthritis and urticaria, which are attributed to immune complex deposition, develop in 5% to 10% of patients with acute HBV and HCV infection. Patients with acute viral hepatitis usually have an enlarged and tender liver. Many patients, especially children, are asymptomatic or do not have jaundice (anicteric hepatitis), and medical attention is often not sought. Because aminotransferases are released from damaged hepatocytes, serum levels of AST and ALT are often greater than 20- to 100-fold normal. Serum total bilirubin levels are elevated; values above 2.5 to 3.0 mg/dL cause jaundice. Serum alkaline phosphatase levels are usually increased to less than three times normal. Mild leukopenia with atypical lymphocytes may also develop. Anemia and thrombocytopenia are rare. Serodiagnosis of Acute Viral Hepatitis The serologic studies for diagnosing acute viral hepatitis are shown in Table 27. An etiologic diagnosis is important for determining the prognosis of infected patients and for instituting preventive and public health measures for close contacts of patients. Acute HAV infection is diagnosed by the presence of IgM antibody to hepatitis A virus (IgM anti-HAV), which persists for 3 to 6 months after onset of infection. IgG anti-HAV appears during the acute illness and persists thereafter, granting long-term immunity. In patients with acute HBV infection, hepatitis B surface antigen (HBsAg) appears 2 to 10 weeks after exposure and usually clears in 4 to 6 months. Clearance of HBsAg is followed by a window period, after which antibody to hepatitis B surface antigen (anti-HBs) appears. The

TA B L E 2 7 Serodiagnosis of Acute Viral Hepatitis

Type Hepatitis A Hepatitis B Hepatitis C Hepatitis D Hepatitis E

IgM anti-HAV +

HBsAg + +

IgM anti-HBc +

IgG anti-HCV +*

IgM anti-HDV +

IgM anti-HEV +

IgM anti-HAV = IgM antibody to hepatitis A virus; HBsAg = hepatitis B surface antigen; IgM anti-HBc = IgM antibody to hepatitis B core antigen; IgG anti-HCV = IgG antibody to hepatitis C virus; IgM anti-HDV = IgM antibody to hepatitis D virus; IgM anti-HEV = IgM antibody to hepatitis E virus *A positive IgG anti-HCV is observed in only 50%60% of patients at the onset of symptoms. HCV RNA becomes detectable within 2 weeks after exposure. In HDV/HBV co-infection, IgM anti-HBc may be positive.

60

Acute and Chronic Hepatitis

presence of this antibody confers lifelong immunity. During the window period, the presence of IgM antibody to hepatitis B core antigen (IgM anti-HBc) may be the only evidence of HBV infection. Acute HDV infection superimposed on HBV infection is detected by the presence of IgM antibody to hepatitis D virus (IgM anti-HDV). Acute HCV infection can be diagnosed by a positive assay for HCV RNA within 2 weeks of exposure. Antibody to hepatitis C virus (anti-HCV) develops within 12 weeks of exposure. If only the antibody test is performed at the onset of symptoms, acute HCV infection may not be detected in 30% to 50% of patients. Therefore, HCV RNA is the most sensitive test for persons with suspected acute HCV infection. Acute HEV infection is detected by IgM and IgG antibodies to HEV. However, available assays are limited by their poor sensitivity and specificity and are not currently approved by the United States Food and Drug Administration. Treatment of Acute Viral Hepatitis and Prevention in Contacts Therapy for acute viral hepatitis is primarily supportive and includes rest, symptomatic care (for example, administration of antiemetics), and adequate hydration and dietary intake. Hospitalization is indicated for patients with dehydration or deteriorating liver function. Prevention of all types of acute viral hepatitis infection in contacts relies on universal precautions. In addition, persons exposed to HAV or HBV may require administration of immunoglobulin and/or vaccination, as appropriate. In patients with acute HCV infection, therapy with interferon-alfa may decrease the risk of developing chronic hepatitis (Jaeckel et al.; Poynard et al.).

Drug- and Toxin-Induced Acute Hepatitis


Drugs and toxins cause a broad spectrum of hepatic diseases. Many reactions are predictable and dose-dependent (for example, toxicity due to acetaminophen and carbon tetrachloride), whereas others are unpredictable (idiosyncratic). Some drugs, such as halothane and isoniazid, have genetically determined metabolic pathways that produce toxic metabolites. Immune-mediated hepatotoxicity may occur if drugs or their metabolites act as haptens on the surface of hepatocytes. Herbal supplements are increasingly being used worldwide. Although often considered safe because they are natural, many are hepatotoxic. Senecio, Heliotropium, Crotalaria, and comfrey contain pyrrolizidine alkaloids that cause hepatic veno-occlusive disease. Hepatotoxicity ranging from mild hepatitis to fulminant hepatic failure has been associated with the use of chaparral, germander, pennyroyal oil, mistletoe, valerian root, comfrey, and Mahuang. Other than supportive care, the mainstay of therapy for patients with acute drug- or toxin-related hepatic injury is the withdrawal of the causative agent. However, patients with acetaminophen-induced hepatic injury should be treated with N-acetylcysteine.

Jaeckel E. Cornberg M, Wedemeyer H, Santantonio T, Mayer J, Zankel M, et al. Treatment of acute hepatitis C with interferon alfa- 2b. N Eng J Med. 2001;345:1452-7. PMID: 11794193 Poynard T, Regimbeau C, Myers RP, Thevenot T, Leroy V, Mathurin P, et al. Interferon for acute hepatitis C. Cochrane Database of Syst Rev. 2002; Review. PMID: 11869573

Chronic Hepatitis
Chronic hepatitis is defined as an inflammatory hepatic process that has persisted for longer than 6 months. Histologically, inflammatory cells may extend beyond the limits of the portal tracts to surround hepatocytes (interface hepatitis). Bridging necrosis between adjacent portalportal and portalcentral areas may occur. Significant fibrosis, which indicates an increased risk for progression to cirrhosis, may also develop. Acute HBV and HCV infections may become chronic. Acute HAV and HEV infections do not progress to a chronic form. Nonalcoholic steatohepatitis (NASH) is a frequent cause of chronic hepatitis in the United States and 61

Acute and Chronic Hepatitis

TA B L E 2 8 Causes of Chronic

Hepatitis Viral Disorders Hepatitis C Hepatitis B Hepatitis B with superimposed hepatitis D Nonalcoholic Steatohepatitis Drugs Methyldopa Methotrexate Nitrofurantoin Amiodarone Captopril Propylthiouracil Autoimmune Disorders Autoimmune hepatitis Primary biliary cirrhosis Primary sclerosing cholangitis Autoimmune cholangiopathy Genetic and Metabolic Disorders Wilsons disease
1-Antitrypsin

Western Europe. Although several drugs may cause chronic hepatitis, an etiologic agent is sometimes difficult to identify. These idiopathic forms may be caused by quiescent autoimmune hepatitis, undetected drug toxicity, or antibody-negative viral infections.

Classification of Chronic Hepatitis


The classification of chronic hepatitis was first developed in the mid-1970s. Chronic persistent hepatitis (inflammatory activity confined to portal areas) and chronic lobular hepatitis (inflammatory activity and necrosis scattered throughout the lobule) were thought to have a good prognosis. Chronic active hepatitis (inflammation that extended into the lobule [periportal hepatitis, interface hepatitis] in association with necrosis and fibrosis) was thought to have an increased risk for progression to cirrhosis and liver failure. Several causes of chronic hepatitis have subsequently been determined (Table 28). This has prompted reclassification of chronic hepatitis based on the etiologic agent together with the grade of injury (determined by the number and location of inflammatory cells) and the stage of disease (determined by the degree, location, and distortion of normal hepatic architecture by fibrosis). The new classification allows integration of specific causes of chronic liver disease with histologic features of hepatic damage to assess the severity and prognosis of the disorder. Chronic Viral Hepatitis Chronic viral hepatitis may follow acute infection with either HBV or HCV. Chronic HBV infection follows an acute infection in approximately 5% of adults and 90% of neonates born to mothers who are positive for hepatitis B surface antigen (HBsAg) and hepatitis B e antigen (HBeAg). Although patients with early-stage chronic HBV infection are frequently asymptomatic, some patients have malaise, fatigue, and anorexia. Chronic HBV infection causes cirrhosis and end-stage liver disease in approximately 25% to 30% of individuals. Patients who are positive for HBsAg and HBeAg and have high serum levels of HBV DNA (>5 million copies/mL) are considered to be in a high viral replicative phase and frequently have abnormal serum aminotransferase levels and histologic evidence of inflammatory activity. These patients are at greater risk for having progressive liver injury and developing cirrhosis and hepatocellular carcinoma than are those who are HBsAg positive but HBeAg negative and who have low serum levels of HBV DNA (<5 million copies/mL). This latter group was previously referred to as asymptomatic carriers. A subgroup of patients with chronic HBV infection may be HBeAg negative but still be in a high viral replicative phase, as evidenced by high serum HBV DNA levels. These patients have a mutation of the precore region of the HBV genome. Therapy for chronic HBV infection is currently reserved for patients in a high viral replicative phase. Available treatment options include interferon-alfa or the oral nucleoside analog lamivudine (Lok and McMahon). Administration of interferon-alfa, 5 MU daily or 10 MU three times weekly subcutaneously for 16 to 24 weeks, results in decreased inflammatory activity and suppressed viral replication in patients with well-compensated liver disease and no contraindications to interferon therapy. Loss of HBeAg positivity and development of anti-HBe (seroconversion) occur in 15% to 20% of patients. Loss of HBsAg positivity occurs in 7.8% of treated patients compared with 1.8% of controls. Side effects of interferon-alfa require dose modification in up to 35% of patients and discontinuation of therapy in some patients. For patients who are unable to tolerate, do not respond to, or have contraindications to interferon-alfa, treatment with lamivudine is a safe and welltolerated alternative. Lamivudine, 100 mg orally daily for 52 weeks, results in a

deficiency

Lok AS, McMahon BJ. Chronic hepatitis B. Hepatology. 2001;34:1225-41. PMID: 11732013

62

Acute and Chronic Hepatitis

16% to 18% seroconversion rate of HBeAg to anti-HBe. The major complication of long-term lamivudine therapy is the emergence of resistant viral strains (the most common being the YMDD mutant) signaled by the reappearance of HBV DNA. This complication occurs in 14% to 32% of patients after 1 year of treatment. Administration of lamivudine for more than 1 year should be considered for patients with advanced liver disease, for those in whom seroconversion has not occurred, and for those with mutant strains provided they have no evidence of worsening liver disease associated with the mutant virus. Clinical trials are currently under way to evaluate the efficacy of other nucleoside analogs. More than 70% to 80% of patients with acute HCV infection develop chronic infection. According to the National Health and Nutrition Examination Survey conducted from 1988 to 1995, the prevalence of antiHCV in the United States is 1.8% (equal to an estimated 3.9 million persons nationwide, 74% of whom are positive for HCV RNA). HCV can be further classified into distinct genotypes on the basis of RNA sequence variability. In the United States, approximately 70% to 75% of patients are infected with genotype 1; the remainder are primarily infected with genotypes 2 and 3. Patients with chronic HCV infection are usually asymptomatic, but some experience nonspecific constitutional symptoms such as malaise, fatigue, anorexia, nausea and flu-like symptoms. In addition, several extrahepatic manifestations have been described, including cryoglobulinemia, glomerulonephritis, porphyria cutanea tarda, Sjgrens syndrome, and seronegative arthritis. About 20% of patients develop cirrhosis approximately 20 to 30 years after infection. This is currently the most common indication for liver transplantation. Epidemiologic studies have found that the risk for progression of fibrosis in patients with HCV infection correlates with male sex, increased patient age, and a history of alcohol intake, particularly consumption greater than 50 g/d. Antiviral treatment has evolved from interferon monotherapy to the current combination of pegylated interferon and ribavirin (Poynard et al. 2000; Poynard et al. 2002). A recent study found that pegylated interferon alfa-2b, 1.5 g/kg weekly, plus ribavirin, 800 mg/d, produced a sustained virologic response (SVR) of 54% (Manns et al.). In patients with genotype 1 infection, the SVR was 42% after 48 weeks of therapy, whereas the SVR for patients with genotype 2 and 3 infections was approximately 80% after 24 weeks of therapy. Other predictors of response include baseline serum HCV RNA levels less than 3.5 million copies/mL, no or only mild fibrosis, female sex, and age younger than 40 years. In addition to viral clearance, benefits of antiviral therapy include improvement in the grade of necrosis and inflammation and reduced progression of fibrosis, indicating a decreased risk for developing advanced liver disease and hepatocellular carcinoma. Major complications of interferon-based regimens include the development of flu-like symptoms, fatigue, malaise, bone marrow suppression, neuropsychiatric symptoms, induction of autoimmune disorders, and worsening of existing autoimmune disorders. Ribavirin may cause a hemolytic anemia, which may worsen preexisting coronary and peripheral arterial diseases and precipitate heart failure. Contraindications to combination interferon and ribavirin therapy include poorly controlled psychiatric illness, pregnancy or the inability to practice contraception, decompensated cirrhosis, renal failure, autoimmune disorders, pancreatitis, and active substance abuse. Relative contraindications include cardiovascular disease, pulmonary disease, seizure disorders, diabetes mellitus, hemoglobin less than 12 g/dL in women and less than 13 g/dL in men, a leukocyte count less than 1500/L, a platelet count less than 75,000/L, and age greater than 65 years.

Poynard T, McHutchison J, Goodman Z, Ling MH, Albrecht J. Is an a la carte combination interferon alfa-2b plus ribavirin regimen possible for the first line treatment in patients with chronic hepatitis C? The ALGOVIRC Project Group. Hepatology. 2000;31:211-8. PMID: 10613748 Poynard T, McHutchison J, Manns M, Trepo C, Lindsay K, Goodman Z, et al. Impact of pegylated interferon alfa-2b and ribavirin on liver fibrosis in patients with chronic hepatitis C. Gastroenterology. 2002;122:1303-13. PMID: 11984517 Manns MP, McHutchison JG, Gordon SC, Rustgi VK, Shiffman M, Reindollar R, et al. Peginterferon alfa-2b plus ribavirin compared with interferon alfa-2b plus ribavirin for initial treatment of chronic hepatitis C: a randomised trial. Lancet. 2001;358:958-65. PMID: 11583749

63

Autoimmune and Chronic Cholestatic Liver Diseases

KEYPOINTS

Hepatitis A, B, and C viruses are the major causes of acute viral hepatitis in the United States. HBV and HCV infections can be either acute or chronic. Acute HAV and HBV infections are diagnosed by antibody/antigen tests. The preferred test for diagnosis of acute HCV infection is measurement of HCV RNA rather than determination of anti-HCV. A detailed medication and toxin exposure history and a high index of suspicion are the mainstays for the diagnosis of acute drug-induced hepatitis. Management of acute viral hepatitis consists primarily of symptomatic relief and supportive care. Management of. acute drug-induced hepatitis requires removal of the causative agent. Chronic HBV infection results in cirrhosis in approximately 25% to 30% of patients, and chronic HCV infection results in cirrhosis in approximately 20% of patients. Current treatment options for chronic HBV infection include lamivudine or interferon-alfa. Pegylated interferon combined with ribavirin is the current treatment of choice for chronic HCV infection. Many drugs may cause chronic hepatitis, including methotrexate, nitrofurantoin, oral sulfonylureas, oral hypoglycemic agents, and antihypertensive agents.

Chronic Drug-Induced Hepatitis Chronic drug-induced hepatitis is defined as a drug-related injury that causes elevated liver chemistry test results for more than 6 months and that usually results from continued exposure to the drug. The folate antagonist methotrexate is commonly associated with elevated serum aminotransferase levels in 20% to 50% of patients. However, this does not necessarily imply significant toxicity. Risk factors for the development of significant liver injury and fibrosis during methotrexate therapy include concomitant alcohol intake, preexisting liver disease, daily administration, treatment for more than 2 years, and a cumulative methotrexate dose of greater than 1.5 g. Other drugs that have been associated with chronic liver injury include methyldopa and nitrofurantoin, which cause a syndrome resembling autoimmune hepatitis. Captopril, chlorpromazine, haloperidol, imipramine, and tolbutamide have been associated with a chronic cholestatic syndrome resembling primary biliary cirrhosis. Granulomatous hepatitis may occur with use of diltiazem, quinidine, procainamide, and allopurinol. Other Causes of Chronic Hepatitis Autoimmune, genetic, and metabolic causes of chronic hepatitis are discussed later in this syllabus.

Autoimmune and Chronic Cholestatic Liver Diseases


What are the principal biochemical, serologic, and histologic features of autoimmune hepatitis and primary biliary cirrhosis? Which therapy is effective for most patients with autoimmune hepatitis? Which therapy is currently recommended for patients with primary biliary cirrhosis? How is the diagnosis of primary sclerosing cholangitis usually established? What is the role of liver transplantation in the management of patients with autoimmune hepatitis, primary biliary cirrhosis, and sclerosing cholangitis?

Patients with autoimmune hepatitis present with features of hepatocellular damage and elevated serum aminotransferase levels. Other liver chemistry studies are variably abnormal, depending on the severity of disease. The disease onset is indistinguishable from that of acute viral hepatitis in approximately 25% of patients. Women are affected more often than men (ratio of 3.6 to 1). All ethnic groups and age groups are susceptible, although the two peak incidences occur between 10 and 20 years of age and around menopause. Liver biopsy specimens classically show a dense mononuclear infiltrate in the portal triads. The infiltrate is composed of lymphocytes and numerous plasma cells and extends into the surrounding liver parenchyma (interface hepatitis) (Figure 11). The presence of hypergammaglobulinemia and characteristic autoantibodies helps establish the diagnosis. Type 1 autoimmune hepatitis, which accounts for 70% to 80% of cases, is associated with positive antinuclear antibody and/or antismooth muscle antibody at a titer of 1:80 or greater. Type 2 autoimmune hepatitis, which is a more severe form of the disease, is associated with a positive titer for antiliver/kidney microsomal type 1 antibody. Up to 40% of patients with untreated autoimmune hepatitis die within 6 months of diagnosis, and most survivors develop cirrhosis over time. Prednisone, either alone or in combination with azathioprine, improves the prognosis. Patients with or without cirrhosis respond equally well to this regimen, and the 20-year life expectancy for treated patients exceeds 80%. Patients can usually be maintained in long-term remission with a combination of prednisone, 10 mg/d, and azathioprine, 50 mg/d. Nevertheless, drug withdrawal should be attempted for all patients at least once. Liver transplantation is asso-

64

Autoimmune and Chronic Cholestatic Liver Diseases

FIGURE 11A AND 11B. Pathologic features of autoimmune hepatitis and primary biliary cirrhosis. A, Liver biopsy specimen from a patient with autoimmune hepatitis showing a mononuclear infiltrate in a portal triad that consists of lymphocytes and plasma cells (arrows) and extends into the liver parenchyma (interface hepatitis). B, Liver biopsy specimen from a patient with primary biliary cirrhosis showing a portal triad that contains a degenerating bile duct (double arrow) that is surrounded by lymphoid follicles and a granuloma containing a giant cell (single arrow). (See Color Plates 6A and 6B at back of book.)

11A

11B

ciated with excellent survival rates for patients whose condition deteriorates despite corticosteroid treatment. However, autoimmune hepatitis recurs in the graft in approximately 25% of transplant recipients (Czaja and Freese; Manns and Strassburg). Primary biliary cirrhosis is a cholestatic disease that mainly affects women (90% of cases) between 40 and 60 years of age. The most common symptoms at onset are pruritus and fatigue, and the major biochemical abnormality is an elevated serum alkaline phosphatase level. The histologic abnormalities in early (stages 1 and 2) disease include degeneration of interlobular bile ducts, often with lymphocytes infiltrating the bile duct epithelium, along with portal lymphoid follicles and occasional granulomas (see Figure 11). Stage 3 disease is characterized by bridging fibrosis between portal triads, and stage 4 disease indicates the presence of cirrhosis. In patients with typical clinical and histologic features, the diagnosis is established by a positive test for antimitochondrial antibody (titer of 1:40 or greater). More than 25% of patients develop hepatic

Czaja AJ, Freese DK. Diagnosis and treatment of autoimmune hepatitis. Hepatology. 2002;36:479-97. PMID: 12143059 Manns MP, Strassburg CP. Autoimmune hepatitis: clinical challenges. Gastroenterology. 2001;120:1502-17. PMID: 11313321

65

Complications of Cirrhosis

Prince M, Chetwynd A, Newman W, Metcalf JV, James OF. Survival and symptom progression in a geographically based cohort of patients with primary biliary cirrhosis: follow-up for up to 28 years. Gastroenterology. 2002;123:1044-51. PMID: 12360466 Pasha T, Heathcote J, Gabriel S, CauchDudek K, Jorgensen R, Therneau T, et al. Cost-effectiveness of ursodeoxycholic acid therapy in primary biliary cirrhosis. Hepatology. 1999;29:21-6. PMID: 9862844 Angulo P, Lindor KD. Primary sclerosing cholangitis. Hepatology. 1999;30:325-32. PMID: 10385674
KEYPOINTS

Most patients with autoimmune hepatitis have elevated serum aminotransferase levels; 70% to 80% of patients also have a positive antinuclear antibody and/or antismooth muscle antibody titer. Prednisone, either alone or combined with azathioprine, significantly improves the prognosis for patients with autoimmune hepatitis. Most patients with primary biliary cirrhosis have an elevated serum alkaline phosphatase level; a positive antimitochondrial antibody titer helps establish the diagnosis. The diagnosis of primary sclerosing cholangitis is usually established by endoscopic retrograde cholangiopancreatography, which shows diffuse strictures and beading of the intrahepatic and extrahepatic bile ducts. Ursodeoxycholic acid (ursodiol) is currently recommended for treating patients with primary sclerosing cholangitis. Liver transplantation has proved to be effective for patients with end-stage autoimmune hepatitis, primary biliary cirrhosis, and primary sclerosing cholangitis.

failure within 10 years after diagnosis, and the median survival time is 9.3 years (Prince et al.). The currently recommended therapy for primary biliary cirrhosis is ursodeoxycholic acid (ursodiol), 12 to 15 mg/kg daily. Several studies have shown that this drug improves biochemical parameters, slows progression of disease, and is cost effective (Pasha et al.). Liver transplantation is effective for patients with end-stage disease, but features of primary biliary cirrhosis may recur in the graft. Some patients have features that are indistinguishable from primary biliary cirrhosis but do not have a positive antimitochondrial antibody titer. This disorder has traditionally been termed antimitochondrial antibodynegative primary biliary cirrhosis and more recently has been called autoimmune cholangitis if the antinuclear antibody titer is positive. Autoimmune cholangitis does not appear to differ significantly from primary biliary cirrhosis in terms of outcome. Primary sclerosing cholangitis is characterized by progressive inflammation and fibrosis of the large intrahepatic and extrahepatic bile ducts. Approximately 70% to 80% of patients have inflammatory bowel disease, usually chronic ulcerative colitis. The diagnosis is established by findings on endoscopic retrograde cholangiopancreatography that show diffuse strictures and beading of the bile ducts. Primary sclerosing cholangitis causes biliary cirrhosis and its complications, and the median survival from the time of diagnosis is 9 to 11 years. Approximately 10% to 15% of patients develop bile duct carcinoma, which is difficult to diagnose and treat. Therapy for primary sclerosing cholangitis includes supportive care, endoscopic management of dominant strictures, and prompt treatment of superimposed episodes of bacterial cholangitis. Liver transplantation may be effective for patients with end-stage disease (Angulo and Lindor).

Complications of Cirrhosis
What is the pathophysiology of portal hypertension? What is the progression of therapy for ascites, and how is refractory ascites managed? How is the diagnosis of spontaneous bacterial peritonitis established? Which therapy is used for primary prophylaxis of variceal hemorrhage? What are the precipitating causes of hepatic encephalopathy? What are the diagnostic screening tests for hepatocellular carcinoma?

The primary complications of cirrhosis are portal hypertension with associated ascites, spontaneous bacterial peritonitis, hepatorenal syndrome, gastroesophageal varices, hepatic encephalopathy, and hepatocellular carcinoma.

Portal Hypertension and Ascites


Portal hypertension is a complication of cirrhosis that is a major cause of ascites formation and variceal hemorrhage (Garcia-Tsao). Elevated portal pressure results from a combination of increased portal blood flow due to splanchnic vasodilatation and intrahepatic resistance to portal blood flow. A diminished effective circulating blood volume causes enhanced sodium and water retention. The normal pressure gradient between the portal and hepatic veins is 3 to 6 mm Hg. Ascites develops when the gradient exceeds 10 mm Hg, and variceal hemorrhage occurs when it exceeds 12 mm Hg. Ascites occurs in approximately 30% of patients with cirrhosis. Initial therapy is restriction of dietary sodium to 2 g/d. Diuretics can be considered if sodium restriction is ineffective. Spironolactone, which has greater efficacy than loop diuretics, is administered first. Patients with moderate to severe ascites

Garcia-Tsao, G. Current management of the complications of cirrhosis and portal hypertension: variceal hemorrhage, ascites and spontaneous bacterial peritonitis. Gastroenterology. 2001;120:726-48. PMID: 11179247

66

Complications of Cirrhosis

require both spironolactone and furosemide, in 50-mg and 20-mg increments, respectively. Complications of diuretic therapy include prerenal azotemia, hyponatremia, and hepatic encephalopathy. When spironolactone, 400 mg/d, and furosemide, 160 mg/d, are ineffective, the ascites is considered to be refractory to diuretics and is managed by large-volume paracentesis. Colloid replacement is needed when more than 5 liters of ascitic fluid is removed (6 to 8 g of albumin is given for each liter of fluid that is removed). If this is ineffective, peritoneovenous shunt and transjugular intrahepatic portosystemic shunt (TIPS) may be considered while the patient is listed for liver transplantation.

Spontaneous Bacterial Peritonitis


Spontaneous bacterial peritonitis occurs in patients with cirrhosis who have ascites. The main mechanism of infection is translocation of enteric bacteria. Ascitic fluid with a polymorphonuclear leukocyte count greater than 250/L establishes the diagnosis. Cefotaxime, given intravenously for 5 days, is usually effective for treating spontaneous bacterial peritonitis. Gentamicin and other aminoglycosides should be avoided because of renal toxicity. Communityacquired spontaneous bacterial peritonitis without complications can be treated with oral ofloxacin. Norfloxacin, 400 mg/d indefinitely, is recommended for the prevention of recurrent infection. Antibiotic prophylaxis with a fluoroquinolone for 1 week is also recommended for patients with cirrhosis who have variceal bleeding.

Hepatorenal Syndrome
Hepatorenal syndrome (functional renal failure caused by the severe vasoconstriction of arterial vessels in the renal cortex) may occur after spontaneous bacterial peritonitis or after large-volume paracentesis. The only definitive therapy is liver transplantation. Continuous venovenous hemodialysis is preferred for patients with liver and renal failure who are awaiting liver transplantation. Limited studies of patients with hepatorenal syndrome suggest that renal function may improve following use of specific splanchnic vasoconstrictors (currently not available in the United States) in combination with intravenous volume expansion or transjugular intrahepatic portosystemic shunt.

Gastroesophageal Varices
Gastroesophageal varices develop in 40% of patients with well-compensated cirrhosis and in 85% of those with decompensated cirrhosis (Sharara and Rockey). Patients who have large varices with red or blue markings are at high risk for hemorrhage. Screening for esophageal varices is recommended for all patients with cirrhosis, but the cost effectiveness has not been determined. Options for primary and secondary treatment and prevention of variceal hemorrhage are shown in Table 29. Nonselective -blockers (propranolol, nadolol) are used for primary and secondary treatment of acute bleeding, with a goal of a 25% reduction in resting heart rate. Isosorbide mononitrate (10 to 20 mg twice daily), in conjunction with -blockers, reduces the incidence of initial variceal bleeding by 50%, but patients with advanced liver disease usually have difficulty tolerating this combination. Because endoscopic band ligation has fewer complications than sclerotherapy, banding is now preferred for the primary and secondary prevention of variceal bleeding. Transjugular intrahepatic portosystemic shunt may be used for patients with decompensated cirrhosis who are waiting for liver transplantation. Patients with well-compensated cirrhosis may require shunt surgery for prevention of recurrent variceal bleeding.
Sharara, AL, Rockey DC. Gastroesophageal variceal hemorrhage. N Engl J Med. 2001;345:669-81. PMID: 11547722

67

Complications of Cirrhosis

TA B L E 2 9 Management of Esophageal Varices

Management Active bleeding Primary prevention Secondary prevention

First Choice Intravenous octreotide for 25 days and endoscopic band ligation Nonselective -blockers with or without isosorbide mononitrate Endoscopic band ligation and -blockers with or without isosorbide mononitrate

Second Choice TIPS Blakemore tube until TIPS is performed Endoscopic band ligation Shunt surgery for patients with varices classified as ChildTurcotte-Pugh, grade A TIPS if variceal obliteration fails in patients with varices classified as Child-Turcotte-Pugh, grades B and C (see Table 33)

TIPS = transjugular intrahepatic portosystemic shunt

Blei AT and Cordoba J. Hepatic encephalopathy. Am J Gastroenterol. 2001;96:1968-76. PMID: 11467622 Baron RL, Peterson MS. From the RSNA refresher courses: screening the cirrhotic liver for hepatocellular carcinoma with CT and MR imaging: opportunities and pitfalls. Radiographics. 2001;21:5117-32. PMID: 11598252 Befeler AS, DiBisceglie AM. Hepatocellular carcinoma: diagnosis and treatment. Gastroenterology. 2002;122:1609-19. PMID: 12016426
KEYPOINTS

Hepatic Encephalopathy
Hepatic encephalopathy is caused by a reversible decrease in neurologic function associated with liver failure and portosystemic venous shunting. Initial manifestations are subclinical and require psychometric testing for diagnosis. These are followed by four progressive stages of impairment: Stage 1: Poor attention, irritability, tremor Stage 2: Drowsiness, poor memory, sleep disorder, asterixis Stage 3: Obtundation (but patient is arousable), hyperactive reflexes, clonus Stage 4: Coma, decerebrate posturing, dilated pupils. The pathogenesis of hepatic encephalopathy is attributed to global central nervous system depression from nitrogenous compounds that result in excitation of -aminobutyric acid (GABA) and decreased neurotransmission of glutamate. Precipitating factors include constipation, medications, infections, dehydration, electrolyte imbalance, gastrointestinal bleeding, and renal failure. Management of patients with chronic hepatic encephalopathy includes administration of oral lactulose titrated to produce two to three bowel movements per day. Protein intake from dairy products and vegetables is preferred. Neomycin and/or metronidazole and zinc are added in difficult-to-control cases. Patients with stage 3 and stage 4 hepatic encephalopathy may require lactulose enemas, possible intubation for airway protection, and rapid identification and correction of the precipitating factors (Blei and Cordoba).

Portal hypertension is caused by intrahepatic resistance to portal blood flow and increased portal blood flow due to splanchnic vasodilatation. The initial therapy for ascites is sodium restriction and spironolactone. Treatment options for refractory ascites include large-volume paracentesis with colloid replacement, transjugular intrahepatic portosystemic shunts, and peritoneovenous shunts. The diagnosis of spontaneous bacterial peritonitis is confirmed by the presence of ascitic fluid that contains polymorphonuclear leukocytes with a count greater than 250/L. Screening for esophageal varices is recommended for all patients with cirrhosis, although the cost effectiveness of screening has not been determined. Nonselective -blockers are the primary prophylactic agents for variceal bleeding. Isosorbide mononitrate can be added if tolerated. Precipitating factors for development of hepatic encephalopathy are constipation, infections, medications, dehydration, electrolyte imbalances, gastrointestinal bleeding, and azotemia. Triphasic abdominal CT scanning, ultrasonography, and measurement of serum -fetoprotein are used to screen for hepatocellular carcinoma.

Hepatocellular Carcinoma
Hepatocellular carcinoma is the most frequently occurring primary liver cancer worldwide and is becoming increasingly prevalent in Western industrialized countries. Symptoms include right upper quadrant abdominal pain, weight loss, and ascites. Many patients are diagnosed incidentally during evaluation for liver transplantation. A serial triphasic CT study showing a lesion that has increased in size and enhances during the arterial phase is considered sufficient evidence for hepatocellular carcinoma (Baron and Peterson), especially if the serum fetoprotein level exceeds 400 ng/mL. Either this imaging study or ultrasonography is frequently used for screening (Befeler and DiBisceglie). Measurement of serum -fetoprotein levels has a poor sensitivity (39% to 64%), fair specificity (76% to 91%), and poor positive predictive value (9% to 32%) for detecting hepatocellular carcinoma. The detection of small lesions allows for curative treatment (liver transplantation, hepatic resection, radiofrequency ablation). Chemoembolization and chemotherapy are used for palliation (Table 30).

68

Fulminant Hepatic Failure

TA B L E 3 0 Treatment of Hepatocellular Carcinoma

Treatment Liver transplantation Hepatic resection Radiofrequency ablation Chemoembolization Chemotherapy

Indications Used for solitary lesion <5 cm or three or fewer lesions, each <3 cm Used if no significant cirrhosis or vascular invasion is present Used for patients awaiting liver transplantation Used as adjuvant therapy before liver transplantation Used when above treatment options are ineffective or in setting of clinical trials

Efficacy 80%90% survival (1 year post-transplant) and 75% survival (5 years post-transplant) 33% to 44% (5-year survival) 3% to 4% 1-year recurrence after ablation; long-term survival is unknown No survival benefit when used alone; decreased tumor recurrence post-transplant Palliation

Fulminant Hepatic Failure


When is the diagnosis of fulminant hepatic failure made? Which drug is the most common cause of fulminant hepatic failure? What are the major complications that occur in patients with fulminant hepatic failure? How are patients with fulminant hepatic failure managed?

Ostapowicz G, Fontana RJ, Schiodt FV, Larson A, Davern TJ, Han SH, et al. Results of a prospective study of acute liver failure at 17 tertiary care centers in the United States. Ann Intern Med. 2002;137:947-54. PMID 12484709 Lee WM. Acute liver failure in the United States. Semin Liver Dis. 2003;23:217-26. PMID: 14523675 Schidt FV, Atillasoy E, Shakil AO, Schiff ER, Caldwell C, Kowdley KV, et al. Etiology and outcome for 295 patients with acute liver failure in the United States. Liver Transpl Surg. 1999;5:29-34. PMID: 9873089 Shakil AO, Kramer, D, Mazariegos GV, Fung JJ, Rakela J. Acute liver failure: clinical features, outcome analysis and applicability of prognostic criteria. Liver Transpl. 2000;6:163-9. PMID: 10719014

Fulminant hepatic failure is defined as hepatic encephalopathy that develops within 8 weeks after the onset of acute severe liver injury. The encephalopathy is usually accompanied by significant jaundice and prolongation of the prothrombin time. Late-onset hepatic failure is a variant in which encephalopathy occurs between 8 and 24 weeks after liver injury. Both disorders are associated with a high mortality rate if the patient develops coma. Fulminant hepatic failure has many causes (Table 31). Data from the United States Acute Liver Failure Study Group indicate that drugs and toxins account for more than 50% of cases and that acetaminophen poisoning is the most common cause (Ostapowicz et al.; Lee). Ingestion of more than 10 g of acetaminophen is usually required to induce fulminant hepatic failure in a patient who does not drink alcoholic beverages. Lesser amounts are required in a patient who does drink because alcohol enhances hepatic metabolism of acetaminophen to its toxic metabolite. Malnourished patients also have an increased risk because of low hepatic glutathione levels. The treatment of choice for patients with acetaminophen-induced fulminant hepatic failure is prompt administration of N-acetylcysteine. Complications of fulminant hepatic failure are numerous and include hepatic encephalopathy that progresses to coma, cerebral edema that reduces cerebral blood flow, pulmonary disease that resembles the acute respiratory distress syndrome, renal failure, hypoglycemia, bacterial and fungal infections, and bleeding diatheses due to platelet decrease or dysfunction and reduced levels of clotting factors. Development of cerebral edema is the most severe complication and is the cause of death in 30% to 50% of patients. Management of patients with fulminant hepatic failure requires prompt determination of the cause, institution of specific therapy (if possible), and careful monitoring. Transfer to a liver transplant center should be considered for patients with grade 2 hepatic encephalopathy (lethargy and asterixis) and should be done for those with grade 3 hepatic encephalopathy (obtunded but arousable). Criteria developed in the United Kingdom that predict the need for transplantation in acetaminophen-induced fulminant hepatic failure are shown in Table 32. Oneyear survival rates for patients with fulminant hepatic failure who undergo liver transplantation have varied from 60% to 90% in different series (Schidt et al.; Shakil et al.).

KEYPOINTS

The diagnosis of fulminant hepatic failure is made when a patient develops hepatic encephalopathy within 8 weeks after the onset of acute liver injury. Acetaminophen toxicity is the most common cause of fulminant hepatic failure in the United States. Patients with fulminant hepatic failure may develop multiple complications, including progressive hepatic encephalopathy, renal failure, clotting disorders, infections, metabolic disturbances, and cerebral edema. Referral to a liver transplant center should be considered for patients with fulminant hepatic failure who develop grade 2 hepatic encephalopathy and should be done for patients with grade 3 hepatic encephalopathy.

69

Liver Transplantation

Kamath PS, Wiesner RH, Malinchoc M, Kremers W, Therneau TM, Kosberg CL, et al. A model to predict survival in patients with end-stage liver disease. Hepatology. 2001;33:464-70. PMID: 11172350 Gonwa TA. Hypertension and renal dysfunction in long-term liver transplant recipients. Liver Transpl. 2001;7:522-6. PMID: 11689773 McCashland TM. Posttransplantation care: role of the primary care physician versus transplant center. Liver Transpl. 2001;7:2-12. PMID: 11689771 Reuben A. Long-term management of the liver transplant patient: diabetes, hyperlipidemia, and obesity. Liver Transpl. 2001;7:13-21. PMID: 11689772

Liver Transplantation
What are the general indications for liver transplantation? When should patients be referred to a liver transplant center? How can the primary care physician assist in post-transplant management?

KEYPOINTS

The general indications for liver transplantation are fulminant hepatic failure, chronic irreversible liver disease of any type, genetic disorders with hepatic involvement, and hepatocellular carcinoma. Patients with chronic liver disease should be referred to a liver transplant center if they have ascites and/or encephalopathy that is poorly controlled by medical therapy or if they have a Child-Turcotte-Pugh score 7. Primary care physicians may provide preventive care for the post-transplant patient and assist in the management of medical conditions such as hypertension, diabetes mellitus, and hyperlipidemia.
TA B L E 3 1 Causes of Fulminant Hepatic Failure

The general indications for liver transplantation are for management of patients with fulminant hepatic failure, chronic irreversible liver disease of any type, and genetic disorders with hepatic involvement. Transplantation is also used for patients with hepatocellular carcinoma who cannot undergo resection; who have a single lesion <5 cm in diameter or no more than three lesions, each of which is <3 cm in diameter; and who have no vascular invasion or extrahepatic spread of tumor. The Child-Turcotte-Pugh classification is used to assess the severity of chronic liver disease (Table 33). Any patient with a score 7 should be considered for referral to a transplant center. In February 2002, the United Network for Organ Sharing changed its criteria for prioritizing patients for liver transplantation to the following: Status 1. Fulminant hepatic failure with stage 3 or 4 encephalopathy Status 2. Model for End-Stage Liver Disease (MELD) score ranging from 6 to 40 points Status 7. Inactive. The MELD score provides a reliable estimate of short-term survival for patients with a wide range of liver diseases (Kamath et al.). It was derived from data from four transplant centers and is based on a patients INR, serum creatinine, and serum total bilirubin levels. More information about using the MELD score can be found at http://www.mayoclinic.org/gi-rst/mayomodel5.html. Contraindications to liver transplantation include HIV disease, extrahepatic malignancy in which recurrence is possible, unresolved sepsis, other severe medical illnesses such as cardiopulmonary compromise, and active use of alcohol or illicit drugs.

Long-term Management of the Post-Transplant Patient


The most common areas in which primary care physicians assist in post-transplant management are preventive care and treatment of hypertension, hyperlipidemia, diabetes mellitus, obesity, and bone disease (Gonwa; McCashland; Reuben). Hypertension occurs in more than 50% of liver transplant recipients as a result of corticosteroid therapy and the effect of calcineurin inhibitors on renal blood flow. Intervention is recommended to maintain blood pressure below 140/90 mm Hg. Diuretics are given first and are followed by calcium channel blockers and then by angiotensin-converting enzyme inhibitors if the blood pressure does not decrease to the recommended value. Hyperlipidemia is a risk factor for cardiovascular disease. Therapy consists of dietary manipulation, weight reduction, exercise, and usually one of the statin lipid-lowering drugs. Management of diabetes mellitus is not significantly different from that for non-transplant patients, except that reduction or withdrawal of corticosteroids is helpful for diabetic transplant recipients. Most patients require insulin as part of their initial therapy.

Drugs Acetaminophen Amiodarone Dapsone Disulfiram Halothane Isoniazid Phenytoin Propylthiouracil Troglitazone* Valproic acid Toxins Amanita mushrooms Bacillus cereus toxin Carbon tetrachloride Yellow phosphorus (cont'd)

70

Liver Disease in Pregnancy

Liver Disease in Pregnancy


What changes occur in liver chemistry values during a normal pregnancy? Which liver diseases may have increased severity in pregnant patients? Which liver diseases are specifically related to pregnancy? What treatment options are available for liver disorders that are specifically related to pregnancy?

TA B L E 3 1 Causes of Fulminant

Hepatic Failure (cont'd) Viral Infections Classic hepatitis viruses Hepatitis A Hepatitis B Hepatitis B and D Hepatitis C Hepatitis E Other viruses Adenovirus Cytomegalovirus Epstein-Barr virus Hemorrhagic fever virus Herpes simplex virus Lassa virus Paramyxovirus Varicella-zoster virus Vascular Disorders Budd-Chiari syndrome Heat stroke Ischemia Primary nonfunction of graft after liver transplantation Veno-occlusive disease Metabolic Disorders Wilsons disease Fructose intolerance Galactosemia Neonatal iron storage disease Tyrosinemia Miscellaneous Acute fatty liver of pregnancy Autoimmune hepatitis Malignant infiltration Lymphoma Breast cancer Lung cancer Colon cancer Melanoma
*No longer on the market.

Liver size, histology, and blood flow do not change during normal pregnancy. However, serum albumin levels progressively decrease as a result of hemodilution and reach a mean of 3.1 g/dL near the end of gestation. Serum alkaline phosphatase levels increase during the third trimester because of leakage of placental alkaline phosphatase into maternal blood. Serum aminotransferase levels are not altered during normal pregnancy. Pregnant women have the same risks as nonpregnant women for contracting most liver diseases. Hepatitis caused by herpes simplex, herpes zoster, or hepatitis E viruses may be more severe during pregnancy. Approximately 20% of pregnant women who contract hepatitis E develop fulminant hepatic failure. The three liver disorders that are specifically related to pregnancy are intrahepatic cholestasis of pregnancy, the HELLP syndrome, and acute fatty liver of pregnancy (Knox and Olans). Intrahepatic cholestasis of pregnancy usually occurs during the second or third trimester and is manifested by pruritus, which is sometimes severe. Jaundice develops in 20% to 60% of women from 1 to 4 weeks after the onset of pruritus. Both findings usually persist until delivery and then resolve rapidly. Intrahepatic cholestasis recurs in 40% to 60% of subsequent pregnancies. Although the maternal prognosis is good, the fetus is at increased risk for premature birth or stillbirth (Heinonen and Kirkinen). Management is therefore directed towards improving the status of the fetus. Pruritus in the mother is managed with ursodeoxycholic acid (ursodiol), 15 mg/kg orally once daily. The HELLP syndrome (hemolysis, elevated liver enzymes, low platelets) and acute fatty liver of pregnancy occur in the third trimester and increase the risk of both maternal and fetal mortality. They are associated with toxemia of pregnancy and also with disorders of fatty acid oxidation in the fetus. As its name implies, the HELLP syndrome is characterized by microangiopathic hemolytic anemia, serum aminotransferase levels 2 to 10 times normal, and a platelet count of less than 100,000/L. Delivery of the fetus provides definitive therapy (Barton and Sibai). Acute fatty liver of pregnancy is characterized by accumulation of microvesicular fat within hepatocytes. The clinical presentation is variable but may progress to hepatic and renal failure, disseminated intravascular coagulation, coma, and death (Reyes). The diagnosis is established by ultrasonographic findings showing increased hepatic echogenicity compatible with fatty infiltration. Prompt delivery is indicated, after which acute fatty liver of pregnancy usually resolves rapidly with no long-term sequelae in survivors. Recurrence in subsequent pregnancies is uncommon but has been reported.

KEYPOINTS

Focal Hepatic Lesions


What is the general approach to evaluating a focal hepatic lesion? How is an amebic hepatic abscess diagnosed and managed? What is the most common benign hepatic tumor found in adults? How are hemangiomas, focal nodular hyperplasia, and adenomas managed? Which metastatic tumor to the liver should be considered for hepatic resection?

Focal lesions of the liver are frequently found incidentally during abdominal imaging studies for other disorders (Table 34). Even though the patient may

The serum albumin level progressively decreases throughout a normal pregnancy, whereas the serum alkaline phosphatase level increases during the third trimester. The severity of hepatitis caused by herpes simplex, herpes zoster, and hepatitis E viruses may be increased during pregnancy. Intrahepatic cholestasis of pregnancy develops in the second and third trimesters, whereas the HELLP syndrome (hemolysis, elevated liver enzymes, low platelets) and acute fatty liver of pregnancy occur only during the third trimester. Treatment of intrahepatic cholestasis of pregnancy is directed towards improving the well-being of the fetus and relieving pruritus in the mother. Definitive therapy for both the HELLP syndrome and acute fatty liver of pregnancy is delivery of the fetus.

71

Focal Hepatic Lesions

TA B L E 3 2 Criteria for Predicting Need for Liver Transplantation in Patients with Fulminant Hepatic Failure

Acetaminophen-Induced Fulminant Hepatic Failure Prothrombin time >100 sec Arterial blood pH <7.3 Elevated serum creatinine level

NonAcetaminophen-Induced Fulminant Hepatic Failure Prothrombin time >50 sec Age <7 yr or >40 yr Serum bilirubin >17.5 mg/dL Duration of jaundice >7 days before onset of encephalopathy

Knox TA, Olans LB. Liver disease in pregnancy. N Engl J Med. 1996;335:659-76. PMID: 8678935 Heinonen S, Kirkinen P. Pregnancy outcome with intrahepatic cholestasis. Obstet Gynecol. 1999;94:189-93. PMID: 10432125 Barton JR, Sibai BM. HELLP and the liver diseases of pre-eclampsia. Clin Liver Dis. 1999;3:31-48. No PMID number. Reyes H. Acute fatty liver of pregnancy: a cryptic disease threatening mother and child. Clin Liver Dis. 1999;3:69-82. No PMID number. Nino-Murcia M, Olcott EW, Jeffrey RB Jr, Lamm RL, Beaulieu CF, Jain KA. Focal liver lesions: pattern-based classification scheme for enhancement at arterial phase CT. Radiology. 2000;215:746-51. PMID: 10831693 van Leeuwen DJ, Shumate CR, Reeders JWAJ. Space-occupying lesions in the liver. In: van Leeuwen DJ, Reeders JWAJ, Ariyama J, eds. Imaging in Hepatobiliary and Pancreatic Disease: A Practical Clinical Approach. WB Saunders Co; 2000:19-46.

be asymptomatic, it is important to identify the lesion as precisely as possible. This is best done in conjunction with a radiologist (Nino-Murcia et al.). Focal hepatic lesions can be classified into three broad areas: cysts and abscesses, benign tumors, and malignant tumors (van Leeuwen et al.).

Hepatic Cysts and Abscesses


Simple hepatic cysts are benign congenital lesions that do not communicate with the biliary tract and are found in approximately 1% of adults. One cyst or several cysts may be present. If multiple cysts are found, fibropolycystic disease should be considered. Benign cysts have a regular wall and fluid density within the lesion. Aspiration is invariably associated with recurrence of the fluid. Further evaluation and therapy are therefore seldom needed. Hydatid cysts are identified by the presence of daughter cysts and calcification, and serologic testing for echinococcal disease will help establish the diagnosis. Hepatic abscesses can be either pyogenic or amebic. The latter is usually found in travelers returning from subtropical areas. Both types may be associated with right upper quadrant abdominal pain and fever. An amebic abscess can often be treated with metronidazole alone. Pyogenic abscesses usually require ultrasound-guided aspiration of pus for Gram stain and culture. A percutaneous catheter should be left in place if the abscess is large and is not adequately drained by aspiration. Surgery is rarely needed.

Benign Hepatic Tumors


Hemangioma, which is the most common benign tumor of the liver, occurs in approximately 4% of the population. It is usually asymptomatic but may cause pain as a result of infarction or bleeding within the tumor. Occasional hemangiomas may be large enough to cause symptoms due to mass effect. The diagnosis is often established by ultrasonography. However, large hemangiomas
TA B L E 3 3 Child-Turcotte-Pugh (CTP) Classification for Assessment of Severity of Liver Disease

Points for Increasing Abnormality Clinical/Biochemical Measurements Encephalopathy Ascites Serum bilirubin (mg/dL) Serum bilirubin (mg/dL) for primary biliary cirrhosis Serum albumin (g/dL) INR Scoring (sum of points) CTP class 1 None None <2 14 >3.5 <1.7 56 A 2 Mild Mild 23 410 2.83.5 1.72.3 79 B 3 ModerateAdvanced ModerateAdvanced >3 >10 <2.8 >2.3 1015 C

Modified from Child CG III, Turcotte J. In Child CG III, ed. The Liver and Portal Hypertension. Philadelphia: Saunders; 1964 and from Pugh RN, Murray-Lyon IM, Dawson JL, Pietroni MC, Williams R. Transection of the oesophagus for bleeding oesophageal varices. Br J Surg. 1973;60:646-9.

72

Focal Hepatic Lesions

TA B L E 3 4 Radiologic Characteristics of Focal Hepatic Lesions

Type Simple cyst Abscess

Ultrasonography Hypoechoic with regular wall and uniform interior Hypoechoic with internal echoes and debris

CT Scan Homogeneous water density; does not enhance after contrast Regular (amebic) or irregular (pyogenic) shape; hypodense with thick rim; does not enhance after contrast Hypodense; early peripheral enhancement, then progressive filling after contrast; hyperdense on delayed images Hypodense or isodense with stellate hypodense central scar; brief homogeneous arterial enhancement after contrast Hypodense or isodense; irregular enhancement after contrast

MRI/Other T1-weighted image hypointense T2-weighted image hyperintense T1-weighted image hypointense T2-weighted image hyperintense

Hemangioma

Well-demarcated, hyperechoic, and homogeneous

T1-weighted image hypointense T2-weighted image hyperintense

Focal nodular hyperplasia

Variable echogenicity; central scar occasionally seen

T1-weighted image isointense T2-weighted image isointense with hyperintense central scar Hyperintense area on T1- and T2-weighted images if hemorrhage is present; hypointense area on T1-weighted image followed by hyperintense area on T2-weighted image if necrosis is present; cold spot on technetium-99m sulfur colloid scan

Adenoma

Well-dermarcated; variable echogenicity

Hepatocellular carcinoma

Variable echogenicity but usually hypoechoic

Hypodense; arterial enhancement T1-weighted image hypointense early after contrast T2-weighted image hyperintense

(>4 cm) are more heterogeneous and may require triphasic spiral CT scanning or technetium-99m red blood cell single-photon emission CT (Horton et al.). Treatment is not indicated unless the patient has significant symptoms or there is a risk of rupture. Focal nodular hyperplasia, the second most common benign hepatic tumor, occurs in approximately 0.4% of the population. Although controversial, some data suggest that estrogens adversely affect the growth rate and potential for hemorrhage of focal nodular hyperplasia. On imaging studies, the lesion is characterized by a central stellate scar and may be identified by ultrasonography, CT scanning, or MRI. Because most patients remain asymptomatic and the lesion does not change, treatment is seldom required. Hepatic adenoma is a rare tumor that occurs most often in women of childbearing age. It is associated with diabetes mellitus, glycogen storage disease, pregnancy, and the use of oral contraceptives. The tumor often causes abdominal pain, and a large or subcapsular tumor may cause intraperitoneal bleeding as a result of rupture. Pregnant women may be especially susceptible to the development of an hepatic adenoma. The adenoma may decrease in size after withdrawal of oral contraceptives or may continue to grow and occasionally undergo malignant degeneration. Regular follow-up imaging studies to monitor changes are therefore indicated. Single hepatic adenomas are usually amenable to surgical resection, but orthotopic liver transplantation may be required if multiple lesions are present. Other hepatic abnormalities may also present as a focal lesion in the liver. Focal fatty infiltration or focal absence of fat in a diffusely fatty liver may produce a mass effect. Inflammatory pseudotumor is an acquired lesion that probably results from a localized infection. Angiomyolipoma, hamartoma, biliary cystadenoma, and intrahepatic bile duct adenoma are rare abnormalities that may cause a focal hepatic lesion.

Horton KM, Bluemke DA, Hruban RH, Soyer P, Fishman EK. CT and MR imaging of benign hepatic and biliary tumors. Radiographics. 1999;19:431-51. PMID: 10194789

73

Genetic Hepatic Diseases

KEYPOINTS

Malignant Hepatic Tumors


The liver is a common site of metastasis for many malignant tumors. Metastatic colorectal cancer involves the liver in approximately 70% of cases and is isolated to the liver in 15% to 20% of cases. Resection of the hepatic metastasis is usually indicated, as 5-year survival may increase significantly postoperatively. Primary hepatic malignancies may also cause focal hepatic lesions (see earlier discussion of Hepatocellular Carcinoma). If liver transplantation is being considered for a cirrhotic patient who develops a focal lesion with radiologic characteristics of hepatocellular carcinoma, liver biopsy should not be done because of the possibility of spreading tumor cells outside the liver. Intrahepatic cholangiocarcinomas may arise from a bile duct within the hepatic parenchyma and may become quite large before causing symptoms. Because patients with cholangiocarcinoma have a high rate of recurrent cancer in a transplant graft, transplantation is done in only a few institutions under protocol conditions.

Even in an asymptomatic patient, an incidental focal hepatic lesion should be evaluated as precisely as possible in conjunction with a radiologist. An amebic hepatic abscess should be considered in a traveler returning from a subtropical area who has abdominal pain and fever and a hypoechoic hepatic lesion on abdominal ultrasonography. Hemangioma is the most common benign hepatic tumor in adults (~4% of the population) and appears as a hyperechoic, homogeneous, and well-demarcated lesion on abdominal ultrasonography. Hemangiomas and focal nodular hyperplasia are usually asymptomatic and do not require surgical intervention. Hepatic adenomas, which tend to grow and cause significant complications, often require resection. Metastatic colorectal cancer may be confined to the liver, and surgical resection may significantly improve the prognosis.

Genetic Hepatic Diseases


What is the most common genetic mutation in white persons with clinical and biochemical features of hereditary hemochromatosis? What is the characteristic pathologic feature in liver biopsy specimens of patients with 1-antitrypsin deficiency? How is Wilsons disease diagnosed?

Bacon BR, Powell LW, Adams PC, Kresina TF, Hoofnagle JH. Molecular medicine and hemochromatosis: at the crossroads. Gastroenterology. 1999;116:193-207. PMID: 9869618 Tavill AS; American Association for the Study of Liver Diseases; American College of Gastroenterology; American Gastroenterological Association. Diagnosis and management of hemochromatosis. Hepatology. 2001;33:1321-8. PMID: 11343262 Kim WR, Brown RS, Terrault NA, El-Serag H. Burden of liver disease in the United States: summary of a workshop. Hepatology. 2002;36:227-42. PMID: 12085369

Hereditary hemochromatosis is the most common genetic disease in the white population. Two mutations in the HFE gene (C282Y and H63D) are usually associated with this disorder. More than 85% of individuals with clinical and biochemical features of hereditary hemochromatosis are homozygous for the C282Y mutation (Bacon et al.). Determination of transferrin saturation is used to screen for hereditary hemochromatosis. If the fasting transferrin saturation exceeds 45% and the serum ferritin level is also elevated, genotyping for HFE should be done. Clinical features of hereditary hemochromatosis are protean and include hepatomegaly, diabetes mellitus, increased skin pigmentation, arthritis, and cardiac disease. Although liver biopsy is no longer needed to establish the diagnosis (Figure 12), it should be considered to assess the degree of fibrosis in C282Y homozygotes who have any of the following features: age greater than 40 years, clinical and/or biochemical evidence of liver disease, or a serum ferritin level greater than 1000 ng/mL. Liver biopsy may also be useful in patients with clinical or biochemical features of hereditary hemochromatosis who are not homozygous for the C282Y mutation. Treatment is phlebotomy until excess iron stores are depleted, as determined by the transferrin saturation and serum ferritin level. This is followed by phlebotomy to maintain a normal serum ferritin level. Patients with cirrhosis should undergo regular screening for hepatocellular carcinoma, as the incidence of this complication is not significantly reduced by phlebotomy. First-degree relatives of patients with hereditary hemochromatosis should be screened by genetic testing and measurement of transferrin saturation (Tavill). Homozygous 1-antitrypsin deficiency (phenotype ZZ) is the most common genetic liver disease in children and occurs in 1:1600 to 1:2800 infants born in the United States (Kim et al.). The diagnosis is established by determining the serum 1-antitrypsin level and phenotype. Approximately 10% to 20% of homozygotes have significant liver disease during childhood, and individuals who survive childhood may develop cirrhosis in mid to late adulthood. These individuals are also at increased risk for developing hepatocellular carci-

74

Genetic Hepatic Diseases

FIGURES 12A12D. Pathologic features of genetic liver diseases. A, Prussian blue stain of a liver biopsy specimen from a patient with hereditary hemochromatosis showing extensive iron deposition in hepatocytes. B, Hematoxylin and eosin stain of cardiac tissue from a patient with hereditary hemochromatosis showing deposition of iron (arrow) in the muscle. C, Periodic acidSchiff (PAS) stain of a liver biopsy specimen from a patient with 1-antitrypsin deficiency showing numerous PAS-positive globules of varying size in hepatocytes (arrows). D, KayserFleischer ring in the cornea of a patient with Wilsons disease (bracketed by arrows). (See Color Plates 7A7D at back of book.)

FIGURE 12A

FIGURE 12B

FIGURE 12C

FIGURE 12D

75

Gallstones (Cholelithiasis) and Acute Cholecystitis

Graziadei IW, Joseph JJ, Wiesner RH, Therneau TM, Batts KP, Porayko MK. Increased risk of chronic liver failure in adults with heterozygous alpha 1-antitrypsin deficiency. Hepatology. 1998;28:1058-63. PMID: 9755243 Loudianos G, Gitlin JD. Wilsons disease. Semin Liver Dis. 2000;20:353-64. PMID: 11076401

KEYPOINTS

More than 85% of white persons with clinical and biochemical features of hereditary hemochromatosis are homozygous for the C282Y mutation in the HFE gene. The presence of periodic acidSchiff (PAS)positive inclusions in hepatocytes is the characteristic finding in liver biopsy specimens from persons who are either homozygous or heterozygous for 1-antitrypsin deficiency. The primary findings in patients with Wilsons disease are a decreased serum ceruloplasmin level and copper deposition in the corneas (Kayser-Fleischer rings).

noma. 1-Antitrypsin deficiency also causes emphysema (especially in cigarette smokers), and clinical symptoms of emphysema usually begin after the third decade of life. Heterozygous 1-antitrypsin deficiency (phenotype MZ) is associated with cryptogenic cirrhosis in adults (Graziadei et al.). Liver biopsy specimens from both homozygous and heterozygous individuals show the presence of characteristic periodic acidSchiff (PAS)positive inclusions in hepatocytes (see Figure 12). Patients with severe liver disease associated with 1-antitrypsin deficiency should be evaluated for liver transplantation. Wilsons disease is an autosomal recessive disorder caused by mutations in the ATP7B protein that transports copper in the hepatocyte (Loudianos and Gitlin). The mutation prevents incorporation of copper into ceruloplasmin or excretion of copper into bile. Patients consequently accumulate excess copper in the liver and subsequently in the brain, kidneys, and corneas (copper deposition in the corneas is termed Kayser-Fleischer rings; see Figure 12). Wilsons disease should be considered in any individual younger than 40 years of age who has chronic liver disease. The presence of hemolytic anemia in a young person with fulminant hepatic failure requires prompt evaluation. The diagnosis is usually established by finding a decreased serum ceruloplasmin level and documenting copper deposition in the corneas by slit-lamp examination. In difficult cases, measurement of hepatic copper content and urine copper excretion may be helpful. In patients in whom Wilsons disease is detected early, lifelong copper chelation therapy, using oral penicillamine or trientine, significantly improves the prognosis. Oral administration of zinc salts is also used to inhibit intestinal absorption of dietary copper. Individuals who present with fulminant hepatic failure or decompensated cirrhosis require evaluation for liver transplantation.

Gallbladder and Bile Ducts


Everhart JE, Khare M, Hill M, Maurer KR. Prevalence and ethnic differences in gallbladder disease in the United States. Gastroenterology. 1999;117:632-9. PMID: 10464139 Ko CW, Sekijima JH, Lee SP. Biliary sludge. Ann Intern Med. 1999;130:301-11. PMID: 10068389

Gallstones (Cholelithiasis) and Acute Cholecystitis


What are the risk factors for development of gallstones? Should all patients with gallstones be treated? What are the most common complications of gallstones?

TA B L E 3 5 Risk Factors for the

Development of Gallstones Age Female sex Pregnancy Obesity Native American ancestry Mexican-American ancestry Decreased physical activity Cholangitis Biliary strictures Ceftriaxone therapy Rapid weight loss Total parenteral nutrition

Gallstone disease (cholelithiasis) is one of the most common gastrointestinal disorders (Everhart et al.). Approximately 20.5 million persons between 20 and 74 years of age who live in the United States are estimated to have gallstones. Of these, 14.2 million are women and 6.3 million are men. Gallstones are classified as either cholesterol or pigmented stones. More than 80% of stones are cholesterol or cholesterol predominant (mixed). Pigmented stones are either black or brown. Black pigmented stones, composed primarily of calcium bilirubinate, occur most often in patients with hemolytic diseases and cirrhosis. Brown pigmented stones are formed during biliary stasis caused by infections. Risk factors for the development of gallstones are shown in Table 35. Biliary sludge (microlithiasis) may mimic gallstones clinically. Sludge is composed of cholesterol monohydrate crystals, calcium bilirubinate granules, or other calcium salts (Ko et al.) and is most commonly detected by ultrasonography or microscopic examination of bile. Risk factors include functional or mechanical causes of bile stasis such as prolonged fasting, rapid weight loss, total parenteral nutrition, and distal common bile duct obstruction. If the risk factors persist, gallstones may eventually form. The development of symptoms or complications may necessitate cholecystectomy.

76

Gallstones (Cholelithiasis) and Acute Cholecystitis

Clinical Presentation and Management of Gallstones


Most patients have asymptomatic gallstones that are discovered incidentally (usually by ultrasonography) and do not require treatment. Although the vast majority of patients are asymptomatic, symptoms develop at a rate of approximately 1% to 2% per year. The most common symptom is biliary pain or colic. Approximately 50% of patients with one episode of untreated symptomatic gallstones will have repeated episodes. Cholecystectomy (ideally laparoscopic cholecystectomy) is therefore indicated for most symptomatic patients.

Complications of Gallstones
Patients with symptomatic gallstones have a 1% to 2% annual risk of developing complications, the most common of which are acute cholecystitis, Mirizzis syndrome, and cholecystoenteric fistula.

Acute Cholecystitis
Acute cholecystitis is the most common complication of cholelithiasis. It results from the impaction of a stone within the cystic duct with subsequent distention and inflammation of the gallbladder. More than 50% of patients with acute cholecystitis also have a secondary bacterial infection of the bile and gallbladder. Most patients with a secondary infection have biliary pain that persists for more than 6 hours and is often accompanied by fever. Murphys sign (interruption of deep inspiration when pressure is applied beneath the right costal arch) is a relatively specific finding for acute cholecystitis that can be elicited by palpation during physical examination or by palpation with the transducer during abdominal ultrasonography. Common bile duct stones should be suspected when the serum total bilirubin level exceeds 4 mg/dL. Ultrasonography frequently shows a thickened gallbladder wall and pericholecystic fluid. Nonvisualization of the gallbladder by biliary scintigraphy may confirm the presence of cystic duct obstruction. Patients with acute cholecystitis require hospitalization. Immediate management includes administration of intravenous fluids and antibiotics, followed by definitive therapy with laparoscopic cholecystectomy. Repeated episodes of acute cholecystitis may result in chronic inflammation characterized by a shrunken gallbladder containing stones or sludge. Laparoscopic or open cholecystectomy is required.

Mirizzis Syndrome
Mirizzis syndrome is an uncommon complication of cholelithiasis caused by an impacted stone in either the cystic duct or the neck of the gallbladder. The impaction causes extrinsic compression of the adjacent bile duct or the development of a fistula to this duct. Patients frequently have jaundice and recurrent cholangitis. Biliary tract imaging shows dilatation of the common hepatic and intrahepatic bile ducts. The distal common bile duct is normal. A contracted gallbladder containing stones is also a typical finding. Endoscopic stenting of the common bile duct and stone removal with subsequent laparoscopic cholecystectomy have proved to be as effective as open cholecystectomy for treating patients with Mirizzis syndrome (Abou-Saif and Al-Kawas; Hazzan et al.; Kok et al.).

Abou-Saif A, Al-Kawas FH. Complications of gallstone disease: Mirizzi syndrome, cholecystocholedochal fistula, and gallstone ileus. Am J Gastroenterol. 2002;97:249-54. PMID: 11866258 Hazzan D, Golijanin D, Reissman P, Adler SN, Shiloni E. Combined endoscopic and surgical management of Mirizzi syndrome. Surg Endosc. 1999;13:618-20. PMID: 10347304 Kok KY, Goh PY, Ngoi SS. Management of Mirizzis syndrome in the laparoscopic era. Surg Endosc. 1998;12:1242-4. PMID: 9745064

Cholecystoenteric Fistula
Acute cholecystitis may be complicated by erosion of a gallstone through the gallbladder wall into another organ. Although the duodenum is the most common site, the fistula can also occur in other locations. A gallstone in the small intestine may cause obstruction, most often in the terminal ileum and at the ileocecal valve. Relief of obstruction may be attempted endoscopically or by extracorporeal shock-wave lithotripsy, but surgery is frequently required.

KEYPOINTS

Approximately 20.5 million persons in the United States have gallstones. Most persons have asymptomatic gallstones that usually do not require treatment. Acute cholecystitis is the most frequent complication of gallstone disease.

77

Common Bile Duct Stones (Choledocholithiasis)

Acalculous Cholecystitis
As its name indicates, acalculous cholecystitis involves inflammation of the gallbladder without detectable stones. The disorder may be either acute or chronic. Although it is not technically a complication of gallstone disease, acute acalculous cholecystitis may sometimes be confused with cholelithiasis. Classic symptoms of biliary colic are frequently reported. However, acute acalculous cholecystitis often occurs in critically ill burn or trauma patients who require mechanical ventilation. Some patients, especially those who are immunosuppressed, have a bacterial or viral cause, such as Salmonella or cytomegalovirus infection. The presentation ranges from isolated fever or hyperamylasemia to classic signs and symptoms of acute calculous cholecystitis. Acute acalculous cholecystitis has a mortality rate of 10% to 50%, compared with a mortality rate of 1% for calculous disease. Early recognition and intervention are therefore crucial. Definitive therapy requires urgent cholecystectomy. Surgical or percutaneous cholecystostomy or endoscopic placement of a nasobiliary catheter into the gallbladder may be used as temporizing measures. Chronic acalculous cholecystitis is characterized by a reduced rate of gallbladder emptying, known as gallbladder dyskinesia (Merg et al.). The primary symptom is right upper quadrant abdominal pain, and some patients have features that overlap with those of functional bowel disease (Chen et al.). Gallbladder dyskinesia is diagnosed by radionuclide scintigraphy that demonstrates a cholecystokinin-stimulated gallbladder ejection fraction of less than 35% (Zeissman). Cholecystectomy improves symptoms in many individuals with delayed gallbladder emptying, but the overlap with functional bowel disease in some patients may influence resolution of pain. The National Institutes of Health Consensus Conference on Endoscopic Retrograde Cholangiopancreatography noted the frequent association of chronic acalculous cholecystitis and sphincter of Oddi dysfunction (sphincter of Oddi dysfunction is discussed in greater detail below). Patients with chronic right upper quadrant abdominal pain should first be evaluated by nuclear scintigraphy and determination of the gallbladder ejection fraction in order to diagnose biliary dyskinesia before considering endoscopic retrograde cholangiopancreatography with manometry to diagnose sphincter of Oddi dysfunction.

Merg AR, Kalinowski SE, Hinkhouse MM, Mitros FA, Ephgrave KS, Cullen JJ. Mechanisms of impaired gallbladder contractile response in chronic acalculous cholecystitis. J Gastrointest Surg. 2002;6:432-7. PMID: 12022997 Chen PF, Nimeri A, Pham QH, Yuh JN, Gusz JR, Chung RS. The clinical diagnosis of chronic acalculous cholecystitis. Surgery. 2001;130:578-83. PMID: 11602887 Zeissman HA. Cholecystokinin cholescintigraphy: clinical indications and proper methodology. Radiol Clin North Am. 2001;39:997-1006. PMID: 11587066

Common Bile Duct Stones (Choledocholithiasis)


How do gallstones and common bile duct stones differ? How are patients with common bile duct stones managed?

Abboud PA, Malet PF, Berlin JA, Staroscik R, Cabana MD, Clarke JR, et al. Predictors of common bile duct stones prior to cholecystectomy: a meta-analysis. Gastrointest Endosc. 1996;44:450-5. PMID: 8905367

Common bile duct stones (choledocholithiasis) are stones that have migrated from the gallbladder or have formed de novo in the common bile duct. Gallstones may migrate into the biliary tree in 10% to 15% of patients treated by cholecystectomy, resulting in pancreatitis, cholangitis, or secondary biliary cirrhosis. Choledocholithiasis is difficult to diagnose prior to imaging studies such as endoscopic retrograde cholangiography, magnetic resonance cholangiography, or operative cholangiography. A meta-analysis using nine predictors of choledocholithiasis demonstrated that no single predictor was reliable for establishing the presence of common bile duct stones (Abboud et al.). Independent predictors of choledocholithiasis include twofold elevations in serum bilirubin, aspartate aminotransferase, and alkaline phosphatase levels. These serologic studies have sensitivities of 74% to 81% but specificities of only 25% to 48%. Ultrasonographic demonstration of a dilated common bile duct has a sensitivity of 53% and a specificity of 90% for diagnosing choledocholithiasis. The optimal management of patients with choledocholithiasis depends on the presence of comorbid conditions and the availability of experts in the fields

78

Cholangitis

of laparoscopy, endoscopy, and interventional radiology. Preoperative endoscopic retrograde cholangiopancreatography (ERCP) is indicated for patients with cholangitis or for those with pancreatitis complicated by cholangitis. When the required equipment and expertise are available, laparoscopic transcystic bile duct exploration is effective in detecting and removing common bile duct stones in greater than 90% of patients. Laparoscopic choledochotomy with stone extraction, with or without T-tube placement, is an option if transcystic bile duct exploration is unsuccessful. Laparoscopic treatment of common bile duct stones appears to be as effective as postoperative therapeutic ERCP and may be more cost effective. Open common bile duct exploration is an option for patients with complex stone disease or for those in whom ERCP may not be feasible. ERCP may be contraindicated in patients with previous enteric reconstruction (Billroth II procedure, Roux-en-Y biliary anastomosis), complex stones greater than 1 cm in diameter, or a biliary stricture. Choledocholithiasis is the most common cause of acute pancreatitis worldwide. Most patients with mild pancreatitis will pass the stone spontaneously, and cholecystectomy is recommended to prevent further episodes of pancreatitis. ERCP sphincterotomy without cholecystectomy may suffice in very elderly or high-risk patients, as this procedure significantly reduces recurrent pancreatitis.

KEYPOINTS

Common bile duct stones (choledocholithiasis) are stones that have migrated from the gallbladder or originate in the common bile duct. Management of common bile duct stones requires a multidisciplinary approach and depends on a patients clinical findings and any comorbid conditions.

Cholangitis
Which antibiotic regimens are effective for treating acute cholangitis? What are the clinical features of recurrent pyogenic cholangitis? Which microorganisms are commonly associated with AIDS cholangiopathy?

Case 9 A 26-year-old Hispanic woman has a 3-day history of abdominal pain, fever, and jaundice. On physical examination, the patient is jaundiced. Her temperature is 38.6 C (101.5 F), and pulse rate is 102/min. Abdominal examination discloses right upper quadrant discomfort on palpation. The leukocyte count is 12,800/L, serum total bilirubin is 3.6 mg/dL, and serum aspartate aminotransferase is 187 U/L. Abdominal ultrasonography shows gallstones and a dilated common bile duct of 8 mm. Acute cholangitis is associated with biliary obstruction and the subsequent development of a suppurative infection within the biliary tree. The bile is normally sterile. However, biliary obstruction may result in an altered immune state and the entry of enteric bacteria into the biliary tract. Obstruction is most often due to gallstones. Approximately 6% to 9% of patients with gallstone disease will develop cholangitis. Pain, fever, and jaundice (Charcots triad) occur in 50% to 100% of patients. The patient in Case 9 has these typical symptoms. About 14% of patients also develop hypotension and mental confusion. These five symptoms together are called Reynolds pentad. Mortality is extremely high in patients with Reynolds pentad unless bile duct decompression is performed emergently (Hui et al.). The diagnosis is established by laboratory studies and imaging techniques. Although the serum total bilirubin value is usually greater than 2 mg/dL, it may be normal in patients with early cholangitis. Bacteremia occurs in 21% to 83% of patients, as reported in various series, and bile cultures grow bacteria in greater than 80% of patients. The most common causative pathogens are aerobic and anaerobic gram-negative bacilli and enterococci. Hepatobiliary imaging

Hui CK, Lai KC, Yuen MF, Ng M, Lai CL, Lam SK. Acute cholangitis predictive factors for emergency ERCP. Aliment Pharmacol Ther. 2001;15:1633-7. PMID: 11564004.

79

Biliary Tract Neoplasms

Hanau LH, Steigbigel NH. Acute (ascending) cholangitis. Infec Dis Clin North Am. 2000;14:521-46. PMID: 10987108 Alpini G, McGill JM, Larusso NF. The pathobiology of biliary epithelia. Hepatology. 2002;35:1256-68. PMID: 11981776 Kumar KS, Nair, S, Hertan HI, Grover H. Isolated intrahepatic biliary dilation in a patient with acquired immune deficiency syndrome (AIDS): AIDS cholangiopahy versus incidental unilobar Carolis disease. J Clin Gastroenterol. 2001;32:79-81. PMID: 11154178

KEYPOINTS

Acute cholangitis is diagnosed by both laboratory tests and hepatobiliary imaging studies. The treatment of choice for acute cholangitis is antibiotic therapy and endoscopic retrograde cholangiopancreatography with sphincterotomy. Recurrent pyogenic cholangitis is a rare disorder that occurs almost exclusively in Southeast Asia. AIDS cholangiopathy most commonly affects HIV-infected patients with CD4 cell counts of less than 200/L.

using CT or ultrasonography helps differentiate cholecystitis from cholangitis and also assists in identifying a hepatic abscess or biliary obstruction (Hanau and Steigbigel). Antibiotic therapy, which includes coverage for enterococci, should begin immediately in patients with severe infection. Effective antibiotic regimens include ureidopenicillins (piperacillin and mezlocillin) plus metronidazole or an aminoglycoside, third-generation cephalosporins, or fluoroquinolones. Initial treatment should be based on drug sensitivity patterns within the local community and then adjusted after blood culture and sensitivity results are available. Endoscopic retrograde cholangiopancreatography (ERCP) with sphincterotomy is essential for removal of impacted stones. Recurrent pyogenic cholangitis, also known as oriental cholangiohepatitis, is a rare disorder of unknown etiology characterized by the development of pigmented intrahepatic bile duct stones, biliary strictures, and repeated episodes of suppurative cholangitis. This disease occurs almost exclusively in Southeast Asians between 20 and 40 years of age. The most common pathogens are gramnegative bacteria, although Clonorchis sinensis is occasionally found. Pyogenic cholangitis is treated with antibiotics. Complications include secondary biliary cirrhosis and cholangiocarcinoma. Recurrent episodes of cholangitis may be prevented by surgical removal of the gallbladder and removal of intrahepatic duct stones during ERCP or via transhepatic cholangiography. AIDS cholangiopathy most often affects HIV-infected patients who have a CD4 cell count of less than 200/L. Sclerosing cholangiopathy, papillary stenosis, cholecystitis, and extrahepatic biliary strictures may also occur together or independently in these patients (Alpini et al.; Kumar et al.). AIDS cholangiopathy is associated with infections due to Cryptosporidium, Isospora, Mycobacterium avium complex, cytomegalovirus, and HIV itself. The most common clinical feature is right upper quadrant abdominal pain, but patients may present with cholangitis. Two thirds of patients have hepatomegaly and abnormal liver chemistry test results. Serum alkaline phosphatase values are greatly elevated compared with the results of other liver chemistry studies. Most patients with papillary stenosis have symptomatic improvement after ERCP sphincterotomy.

Biliary Tract Neoplasms


What are the three most common biliary tract neoplasms? What are the risk factors for these neoplasms?

Neoplasms of the biliary tree can arise from various locations, including the gallbladder, bile ducts (cholangiocarcinoma), and ampulla of Vater. The clinical presentation and diagnosis are often challenging, as some patients present initially with biliary obstruction and others are asymptomatic.
de Groen PC, Gores GJ, LaRusso NF, Gunderson LL, Nagorney DM. Biliary tract cancers. N Engl J Med. 1999;341:1368-78. PMID: 10536130 Kapoor V. Incidental gallbladder cancer. Am J Gastroenterol. 2001;96:627-8. PMID: 11280526 Bartlett DL. Gallbladder cancer. Semin Surg Oncol. 2000;19:145-155. PMID: 11126379 Donahue JH. Present status of the diagnosis and treatment of gallbladder carcinoma. J Hepatobiliary Pancreat Surg. 2001;8:530-4. PMID: 11956904

Gallbladder Carcinoma
Gallbladder carcinoma is the fifth most common gastrointestinal cancer in the United States, with an incidence of 2.5 cases per 100,000 U.S. population and approximately 2800 deaths per year. It occurs most often in elderly persons and affects women three times more often than men. At least 80% to 90% of patients have gallstones, suggesting that cholelithiasis is a risk factor (Table 36). Most (80%) of gallbladder carcinomas are adenocarcinomas; the remainder include squamous cell carcinoma and small cell carcinoma (de Groen et al.). The clinical presentation ranges from an incidental carcinoma found at the time of cholecystectomy to a highly aggressive cancer that is not amenable to treatment (Kapoor). Gallbladder polyps larger than 10 mm, and especially

80

Complications of Laparoscopic Cholecystectomy

those larger than 18 mm, are more likely to be malignant. Preoperative diagnosis can be challenging but should include CT scans and cholangiography to determine tumor resectability. Metastasis is common and most often involves the lungs and bones. Unfortunately, most patients present with advanced gallbladder carcinoma, making the overall 5-year survival rate 5% to 10%. To date, the only treatment that provides a 5-year survival benefit is surgical excision of all detectable tumor. Early gallbladder carcinoma (stages 0 and I) is usually managed by laparoscopic cholecystectomy. Stage II, III, and IV cancers require extended or radical cholecystectomy, including excision of adjacent hepatic tissue and regional lymph nodes (Bartlett; Donahue; Kaushik). Most cancers are resistant to neoadjuvant, adjuvant, and palliative chemoradiation. When used, chemotherapy regimens most commonly include 5-fluorouracil.

Kaushik SP. Current perspectives in gallbladder carcinoma. J Gastroenterol Hepatol. 2001;16:848-54. PMID: 11555095 Orter M. Photodynamic therapy for cholangiocarcinoma. J Hepatobiliary Pancreat Surg. 2001;8:137-9. PMID 11455469

TA B L E 3 6 Risk Factors for the

Development of Biliary Neoplasms Gallbladder Age >50 years Female sex Obesity Gallstones Gallbladder polyps Gallbladder wall calcification (porcelain gallbladder) Anomalous pancreatico-biliary duct junction Typhoid carrier state Bile Ducts (Cholangiocarcinoma) Age >50 years Recurrent pyogenic cholangitis Primary sclerosing cholangitis Choledochal cyst Carolis disease Bile duct adenoma Bile duct papilloma Intraductal gallstones Opisthorchis viverrini infection Clonorchis sinensis infection Familial adenomatous polyposis AIDS Exposure to thorium dioxide (Thorotrast) Typhoid carrier state Smoking

Cholangiocarcinoma
Cholangiocarcinoma is a rare tumor with an incidence of 1 case per 100,000 U.S. population. Approximately 60% to 80% of cholangiocarcinomas arise near the porta hepatis (Klatskin tumors), 20% are located in the distal bile duct, and less than 5% are intrahepatic. Most cholangiocarcinomas occur in patients between 50 and 70 years of age. Risk factors are shown in Table 36. Approximately 90% of patients have obstructive jaundice. Patients with advanced disease may have hepatomegaly or a distended palpable gallbladder (Courvoisiers sign). Although serum CA 19-9 and carcinoembryonic antigen levels are usually elevated, these tests have poor specificity. Cholangiocarcinoma confined to the intrahepatic ducts may be amenable to hepatic resection, and about 35% of patients with perihilar and ductal tumors can be treated with resection and subsequent Roux-en-Y hepaticojejunostomy. Median survival for patients with perihilar tumors ranges from 12 to 24 months. Preoperative or postoperative chemotherapy does not improve survival or quality of life. Radiation therapy is associated with modest improvement in survival rates when combined with other therapeutic modalities. Endoscopic or percutaneous placement of a metal biliary stent for decompression may offer palliative relief of symptoms. Based on a retrospective study, photodynamic therapy appears to offer prolonged palliation and survival compared with stenting alone (Orter).

Adenocarcinoma of the Ampulla of Vater


The incidence of adenocarcinoma of the ampulla of Vater is 2.9 cases per 1 million population in the United States. Risk factors include the presence of familial adenomatous polyposis and Peutz-Jeghers syndrome. Obstructive jaundice occurs early and may be accompanied by occult gastrointestinal bleeding. CT scans, ERCP, and endoscopic ultrasonography are effective for diagnosis and staging. Resection is feasible in more than 85% of patients, and the 5-year survival rate is approximately 50%.

KEYPOINTS

Complications of Laparoscopic Cholecystectomy


How do complication rates following laparoscopic cholecystectomy compare with those following open cholecystectomy? How are bile duct complications of laparoscopic cholecystectomy diagnosed and treated?

Laparoscopic cholecystectomy has become the standard surgical approach for the treatment of gallstones. A prospective, randomized trial has demonstrated the benefits of laparoscopic surgery compared with open cholecystectomy

The three most common biliary neoplasms are gallbladder carcinoma, cholangiocarcinoma, and adenocarcinoma of the ampulla of Vater. Surgical excision of all detectable tumor is the only treatment that provides a minimal 5-year survival benefit for patients with gallbladder carcinoma. Treatment for cholangiocarcinoma includes resection with subsequent Roux-en-Y hepaticojejunostomy, radiation therapy, biliary stenting, and, perhaps, photodynamic therapy. Resection is feasible for more than 85% of patients with adenocarcinoma of the ampulla of Vater.

81

Acute Gastrointestinal Bleeding

McMahon AJ, Russel IT, Baxter JN, Ross S, Anderson JR, Morran CG, et al. Laparoscopic versus minilaparotomy cholecystectomy: a randomised trial. Lancet. 1994;343:135-8. PMID: 7904002 Schafer M, Lauper M, Krahenbuhl L. Trocar and Veress needle injuries during laparoscopy. Surg Endosc. 2001;15:275-80. PMID: 11344428

KEYPOINTS

Although infrequent, bile duct injuries are higher after laparoscopic cholecystectomy than after open cholecystectomy. Bile duct injuries must be diagnosed promptly so that the duct can be repaired as quickly as possible.

(McMahon et al.). However, certain complications are unique to laparoscopic cholecystectomy. One prospective study found that trocars and needles used to gain minimally invasive access to the abdominal cavity caused injuries in nearly 2% of patients undergoing laparoscopic procedures (Schafer et al.). Bile duct injuries are rare with open cholecystectomy procedures (0.1% to 0.3% of all open procedures). However, the recent widespread use of laparoscopic cholecystectomy has resulted in a marked, although still infrequent, increase in the percentage of bile duct injuries (0.4% to 0.6 % of all laparoscopic procedures). Surgical injuries usually cause either bile leakage into the peritoneum or bile duct obstruction. Findings in the early postoperative period include fever, abdominal pain and tenderness, ascites, nausea, elevated serum bilirubin levels, and sometimes frank jaundice. Intraperitoneal bile fluid collections can be demonstrated by ultrasonography, CT scanning, or biliary scintigraphy. Scintigraphy is 83% to 87% accurate for predicting the presence of bile leaks after laparoscopic cholecystectomy. ERCP is of value in both diagnosing and treating bile leaks and obstruction. Prompt diagnosis of bile duct injury is essential. Injuries that are not recognized for several days require complex reconstruction rather than the straightforward repair that can be done when the complication is detected immediately.

Gastrointestinal Bleeding
Gastrointestinal bleeding is classified as acute or chronic and as originating from either the upper or the lower gastrointestinal tract. All patients with any type of gastrointestinal bleeding require careful evaluation and management.

Acute Gastrointestinal Bleeding


What are the usual causes of acute upper and lower gastrointestinal bleeding? Which tests should be performed for a patient with acute upper gastrointestinal bleeding? Is emergent upper endoscopy required for all patients with acute upper gastrointestinal bleeding? Which therapies are most effective for managing a patient with continuous bleeding? What are the usual causes of acute lower gastrointestinal bleeding? Which tests are indicated for patients with acute lower gastrointestinal bleeding? Is urgent diagnostic testing necessary for all patients with acute, self-limited lower gastrointestinal bleeding?

Patients with acute gastrointestinal bleeding typically present with overt signs of hemorrhage, such as melena (black, tarry stools), hematochezia (marooncolored stools), and hematemesis (vomiting blood). Although melena and hematemesis are generally associated with upper gastrointestinal bleeding, some patients with melena have a bleeding site distal to the ligament of Treitz. Similarly, although hematochezia more often is a sign of lower gastrointestinal bleeding, patients with brisk upper gastrointestinal bleeding may present with hematochezia. Acute gastrointestinal bleeding may be an emergency situation, especially when patients are at risk for cardiac, pulmonary, renal, or neurologic complications related to acute blood loss. Patients are also at increased risk of complications related to diagnostic and therapeutic upper endoscopy. The goals of management are to minimize the risk of complications while providing effective therapy to control the bleeding.

82

Acute Gastrointestinal Bleeding

The first steps in the care of a patient with acute gastrointestinal bleeding and symptoms of hemodynamic compromise (fatigue, palpitations, chest pain, and/or dyspnea) are directed towards volume resuscitation. Initial vital signs help determine the severity of blood loss. Marked tachycardia, tachypnea, and hypotension suggest at least a 15% loss in circulating blood volume. The goals of volume resuscitation are to restore normal volume and prevent complications of red blood cell loss, including myocardial infarction, congestive heart failure, and cerebrovascular accident. There is no absolute hemoglobin or hematocrit value that determines when transfusions are appropriate. A younger patient in excellent health may tolerate a lower hemoglobin or hematocrit value better than an older patient with known coronary artery disease.

TA B L E 3 7 Common Causes of

Acute Upper Gastrointestinal Bleeding Ulcerations and Erosions Peptic ulcer disease Medication-induced ulcers Viral-induced ulcers Reflux esophagitis (complicated gastroesophageal reflux disease) Stress erosions Vascular Lesions

Acute Upper Gastrointestinal Bleeding


Upper gastrointestinal bleeding is typically categorized as due to variceal or nonvariceal causes. Common causes are listed in Table 37. As initial resuscitative efforts are begun, a focused history and physical examination can help determine the cause of bleeding. Patients with cirrhosis and portal hypertension may be experiencing a variceal hemorrhage, particularly when bleeding is abrupt and of large volume. Patients taking aspirin or other nonsteroidal antiinflammatory drugs (NSAIDs) may have ulcer-related bleeding. The medical history should also include the type and duration of bleeding, the presence of abdominal pain and/or fever, and whether the patient has a history of ulcers, bleeding or coagulation disorders, or cardiopulmonary, renal, or neurologic disease. Patients should also be asked what medications they are currently taking and whether they are having chest pain, palpitations, dyspnea, or other symptoms that might indicate coronary ischemia. The physical examination should focus on cardiac, pulmonary and airway, abdominal, and rectal examinations. Laboratory studies should include a complete blood count, serum electrolytes, blood urea nitrogen, serum creatinine, a coagulation profile, and blood type and cross match. All patients over 50 years of age and patients of any age with a relevant history of cardiac disease require an electrocardiogram. Nasogastric aspiration should be done. An aspirate that is positive for fresh blood or coffee-ground material indicates that bleeding is from an upper source (esophagus, stomach, duodenum). Once the patient is stabilized hemodynamically, upper endoscopy should be performed. Some patients with voluminous bleeding may require endoscopy before hemodynamic stabilization in order to carry out endoscopic therapy and avoid urgent surgery (Jensen). If a peptic ulcer is found at the time of endoscopy, a biopsy specimen should be obtained for urease testing for Helicobacter pylori. If a specimen is not obtained, a serologic study should be done for H. pylori antibodies. A patient with a positive test should be considered for H. pylori eradication therapy (see discussion of H. pylori in the Stomach and Duodenum section of this syllabus). Finally, failure to control active bleeding at the time of endoscopy requires prompt laparotomy. Case 10 A 31-year-old man comes to the emergency department because of painless melena and the passage of three black, tarry stools over the past 24 hours. He has not had abdominal pain. The patient has been taking ibuprofen, 600 mg four times daily, for the past 14 days because of a sports-related shoulder injury. Medical history and review of systems are otherwise unremarkable.

Varices Dieulafoys lesion Vascular malformations Aortoenteric fistula MalloryWeiss Tear Neoplasia Cancer Ulcerated smooth muscle or stromal tumor Lymphoma

Jensen DM. Endoscopic control of nonvariceal upper gastrointestinal hemorrhage. In: Yamada T, Alpers DH, Laine L, Owyang C, Powell DW, eds. Textbook of Gastroenterology. 3rd ed. Philadelphia: Lippincott Williams & Wilkins; 1999:2857-79.

83

Acute Gastrointestinal Bleeding

On physical examination, temperature is 36.6 C (97.9 F), pulse rate is 90/min, respiration rate is 18/min, and blood pressure is 128/76 mm Hg. Examination is normal except for black stool in the rectal vault. Hemoglobin is 12.0 g/dL. Other routine laboratory studies are normal. Upper endoscopy, performed in the emergency department, shows a 1-cm ulcer in the duodenal bulb with a clean, white base. This patient has acute upper gastrointestinal bleeding from a duodenal ulcer that was presumably caused by recent use of an NSAID. He is young and healthy. His hemoglobin level of 12.0 g/dL may be misleading because a degree of hemoconcentration is likely. However, given his overall good health, he is unlikely to develop cardiac or neurologic complications of acute blood loss if the bleeding is controlled. Upper endoscopy is used to determine the cause and location of upper gastrointestinal bleeding. Endoscopic findings are also used to estimate the likelihood of rebleeding from peptic ulcers (Table 38). Since this patient is in good health and has only a slight risk of rebleeding because of the clean ulcer base, he can be discharged home on appropriate medical therapy with close follow-up. Emergency department upper endoscopy would be less essential in older patients or those with comorbid conditions because such patients would most likely be hospitalized, even if a clean ulcer base was found. Although upper endoscopy should generally be performed as soon as possible in patients with acute, ongoing upper gastrointestinal bleeding, this procedure should be delayed in patients who are unstable and require volume resuscitation. It should also be delayed in patients who require endotracheal intubation for airway protection. Emergent upper endoscopy is usually not needed in hemodynamically stable patients without evidence of ongoing bleeding. The patient in Case 10 presumably developed an NSAID-induced duodenal ulcer. A mucosal biopsy specimen was not obtained for testing for H. pylori because many endoscopists are reluctant to obtain such specimens in a patient with acute bleeding. Instead, they prefer to focus solely on the task of identifying and treating a bleeding site and do not wish to introduce another potential site of bleeding (that is, the biopsy site itself). However, the risk of bleeding is quite low when one or two mucosal biopsy samples are obtained from the surface of the gastric antrum. It is therefore reasonable to obtain endoscopic biopsy specimens when a patient may have H. pylori or, if this is not feasible, to schedule serologic testing for H. pylori at a later date.

TA B L E 3 8 Risk of Early, Clinically Detectable Rebleeding from Peptic Ulcers

Based on Initial Endoscopic Findings* Endoscopic Findings Active bleeding Pigmented protuberance (visible vessel) Adherent clot Clean ulcer base
*Based on data from following sources: Laine L, Peterson WL. Bleeding peptic ulcer. N Engl J Med. 1994;331:717-27. Jensen DM. Endoscopic control of nonvariceal upper gastrointestinal hemorrhage. In: Yamada T, Alpers DH, Laine L, Owyang C, Powell DW, eds. Textbook of Gastroenterology. 3rd ed. Philadelphia: Lippincott Williams & Wilkins; 1999:2857-79. Consensus Conference: Therapeutic endoscopy and bleeding ulcers. JAMA 1989;262:1369-72.

Likelihood of Rebleeding 55%90% 43%50% 12%33% 3%5%

84

Acute Gastrointestinal Bleeding

Case 11 A 74-year-old woman comes to the emergency department because of two episodes of maroon-colored stools over the past 8 hours. She is dyspneic but has no chest or abdominal pain. The patient has congestive heart failure, atrial fibrillation, rheumatic heart disease that required prosthetic mitral valve replacement, and sick sinus syndrome for which a pacemaker has been placed. She also has hypothyroidism. Current medications are warfarin, furosemide, spironolactone, atenolol, and L-thyroxine. On physical examination, temperature is 37.0 C (98.6 F), pulse rate is 130/min and irregular, respiration rate is 20/min, and blood pressure is 90/40 mm Hg. Cardiopulmonary examination shows bibasilar crackles and an irregularly irregular cardiac rhythm. Hemoglobin is 9.7 g/dL, and the INR is 4.2. An electrocardiogram shows atrial fibrillation with no acute ST changes. A nasogastric tube placed in the emergency department discloses bright red blood that does not clear with lavage. Resuscitation with intravenous fluids and blood products is instituted immediately. The patient is transported to the intensive care unit to rule out a myocardial infarction. Upper endoscopy shows a large gastric ulcer with a pigmented protuberance (visible vessel) within its base and no active bleeding. The pigmented protuberance is injected with dilute epinephrine solution and then cauterized with a heater probe device. An intravenous proton pump inhibitor is begun and is then changed to an oral proton pump inhibitor. The patient is transferred to a regular medical floor within 24 hours. Plans are made for repeat upper endoscopy with biopsies for gastric cancer and H. pylori when her condition has stabilized and she returns to her baseline level of health. This patient presents with relatively brisk bleeding from the upper gastrointestinal tract. The passage of maroon-colored stools (hematochezia) rather than melena (black, tarry stools) does not rule out bleeding from an upper gastrointestinal source. Aggressive volume resuscitation is provided prior to upper endoscopy in order to prevent cardiac complications of acute blood loss and to minimize the likelihood of sedation-related complications. Endoscopic examination shows a pigmented protuberance (raised, pigmented structure) within a gastric ulcer. This protuberance, also referred to as a visible vessel, is an artery into which the ulcer has eroded. Because the risk of rebleeding from a pigmented protuberance is high (see Table 38), endoscopic therapy with epinephrine and heater probe cauterization is appropriate. Consultation with a surgeon is recommended for patients who are acceptable surgical candidates and who have a high risk of rebleeding or who have ongoing bleeding despite endoscopic intervention.The patient in Case 11 also receives medical therapy with a proton pump inhibitor, which will further decrease the likelihood of recurrent bleeding (Khuroo et al.). Because gastric cancer cannot be excluded, she requires repeat upper endoscopy with biopsy when she is stable. Biopsy specimens for H. pylori testing could have been obtained at the time of initial endoscopy. However, since repeat endoscopy is planned, specimens may also be obtained during the second procedure.

Khuroo MS, Yattoo GN, Javid G, Khan BA, Shah AA, Gulzar GM, Sodi JS. A comparison of omeprazole and placebo for bleeding peptic ulcer. N Engl J Med. 1997;336:1054-8. PMID: 9091801

85

Acute Gastrointestinal Bleeding

Case 12 A 47-year-old man comes to the emergency department because of the abrupt onset of hematemesis. He has a history of alcohol abuse and was actively drinking just prior to developing hematemesis. There is no documentation of cirrhosis, and he takes no medications. On physical examination, he is somewhat obtunded and confused. Temperature is 36.9 C (98.4 F), pulse rate is 110/min and regular, respiration rate is 18/min, and blood pressure is 110/70 mm Hg. A few spider angiomata are present on the anterior chest. The patient is not jaundiced. Examination is otherwise normal except for black, tarry stool in the rectal vault. Hemoglobin is 11.0 g/dL, platelet count is 140,000/L, INR is 2.0, and serum albumin is 2.6 g/dL. After endotracheal intubation and aggressive fluid resuscitation are performed, the patient is transferred to the intensive care unit. Upper endoscopy in the intensive care unit shows four large varices without active bleeding. No other potential bleeding source is identified. Endoscopic variceal band ligation is performed, and intravenous octreotide is administered. This patient has a history of alcoholism and has bleeding varices documented by endoscopy. However, the hematemesis could be due to another disorder, such as an ulcer or a Mallory-Weiss tear. In fact, peptic ulcer disease is a common cause of upper gastrointestinal bleeding in patients with cirrhosis. In addition to receiving vigorous fluid resuscitation, the patient was appropriately intubated. Intubation was needed to protect his airway from aspiration, which could have occurred because of his obtunded mental status or as a result of endoscopy. Variceal band ligation is generally safer and more effective than variceal sclerotherapy (Laine and Cook), and patients with variceal bleeding have better outcomes when intravenous octreotide is administered in conjunction with endoscopic therapy (Besson et al.).

Laine L, Cook D. Endoscopic ligation compared with sclerotherapy for treatment of esophageal variceal bleeding. A meta-analysis. Ann Intern Med. 1995;123:280-7. PMID: 7611595 Besson I, Ingrand P, Person B, Boutroux D, Heresbach D, Bernard P, et al. Sclerotherapy with or without octreotide for acute variceal bleeding. N Engl J Med. 1995;333:555-60. PMID: 7623904

Acute Lower Gastrointestinal Bleeding


The most common causes of acute lower gastrointestinal bleeding that leads to hemodynamic compromise are bleeding diverticula, vascular ectasia, proctopathy due to radiation therapy or colitis, ischemic colitis, and colonic neoplasia. Because most episodes of acute lower gastrointestinal bleeding are self-limited, emergent testing is usually required only for patients with persistent bleeding. In addition to the initial history, physical examination, and laboratory studies discussed above, the patient should be asked about abdominal pain, fever, and diarrhea (which may indicate an inflammatory condition within the colon) and a history of radiation therapy (which raises the possibility of radiation colonopathy or proctopathy). Use of NSAIDs is associated with an increased risk of both upper and lower gastrointestinal bleeding. Many tests are available to evaluate acute lower gastrointestinal bleeding. Testing is especially appropriate for a patient with continued bleeding, since localizing the bleeding site increases the likelihood of a directed intervention to arrest the bleeding. Flexible sigmoidoscopy and colonoscopy provide a direct view of the mucosal surfaces and allow hemostatic therapy to be applied to a bleeding site. The overall reported accuracy is approximately 70%. Colonoscopy has several disadvantages. Compared with emergent upper endoscopy, emergent colonoscopy detects fewer lesions that are amenable to endoscopic hemostatic therapy Whether a colon purge (preparation to clear the colon of blood and feces) is appropriate prior to emergent colonoscopy is con-

86

Acute Gastrointestinal Bleeding

troversial. The acutely bleeding patient may not be able to tolerate the preparation, but colonoscopy without preparation may increase the risk of complications or missed lesions. Arteriography may be performed to localize the site of bleeding and, in experienced hands, may also be used to provide embolic therapy. The reported accuracy rate is approximately 60% (Zuccaro). However, the rate of bleeding necessary for extravasation of contrast into the bowel is at least 0.5 to 1.0 mL/min. Lower rates of bleeding or intermittent bleeding not occurring at the time of angiography may result in a negative study. Another disadvantage of arteriography is that a potentially unstable patient must be transported to the radiology suite, where the level of medical care and monitoring is inferior to that in the intensive care unit. Contrast-induced renal failure, emboli from a dislodged clot, and bleeding may also occur. Although radionuclide studies can detect lower rates of bleeding, the exact site of bleeding is not readily apparent in many patients. Some centers use radionuclide scans prior to arteriography, reasoning that if the nuclear medicine study is negative, angiography is likely to be negative as well. An algorithm for the evaluation of acute lower gastrointestinal bleeding is presented in Figure 13. Case 13 A 56-year-old woman comes to the emergency department because of the acute onset of painless hematochezia over the past 14 hours. She feels lightheaded and has palpitations but has not had abdominal pain. Medical history is noncontributory. On physical examination, pulse rate is 130/min and regular, respiration rate is 18/min, and blood pressure is 86/56 mm Hg. Cardiopulmonary examination is normal. Examination of the abdomen is normal except for the presence of hyperactive bowel sounds. Rectal examination is significant for bright red blood on the examining glove; no masses are palpated

Zuccaro G. Management of the adult patient with acute lower gastrointestinal bleeding. Am J Gastroenterol. 1998;93: 1202-8. PMID: 9707037

Patient Presents with Hematochezia

Initial Evaluation

Signs of Hemodynamic Compromise


YES NO

Nasogastric Tube or EGD


NO

Has Bleeding Stopped?


YES

Evidence of UGI Bleeding


YES

NO or Unsure

Colonoscopy

Elective Evaluation

Treat as Appropriate

Source Identified YES NO

Treat as Appropriate

Consider Arteriogram Small Bowel Studies or Surgical Consultation

FIGURE 13. Algorithm of evaluation of a patient with hematochezia.


EGD = esophagogastroduodenoscopy (upper endoscopy); UGI = upper gastrointestinal tract

87

Acute Gastrointestinal Bleeding

Jensen DM, Machicado GA. Diagnosis and treatment of severe hematochezia. The role of urgent colonoscopy after purge. Gastroenterology. 1988;95:1569-74. PMID: 3263294 Luk GD, Bynum TE, Hendrix TR. Gastric aspiration in localization of gastrointestinal hemorrhage. JAMA 1979;241:576-8. PMID: 310892 Cuellar RE, Gavaler JS, Alexander JA, Brouillette DE, Chien MC, Yoo YK, et al. Gastrointestinal tract hemorrhage. The value of a nasogastric aspirate. Arch Intern Med. 1990;150:1381-4. PMID: 2196022

Hemoglobin is 9.6 g/dL, blood urea nitrogen is 38 mg/dL, serum creatinine is 1.0 mg/dL, and serum electrolytes are normal. An electrocardiogram shows no acute changes. Vital signs improve marginally after vigorous fluid resuscitation. However, the patient continues to pass red blood per rectum. Colonoscopy without preparation is done. The colonoscope is passed to the cecum; no bleeding lesions are seen. Intubation of the terminal ileum shows fresh red blood coming from a more proximal site. Upper endoscopy is subsequently performed and shows an actively bleeding duodenal ulcer. As illustrated by the patient in Case 13, red blood per rectum does not always indicate a colonic source of bleeding, especially when bleeding is brisk. Approximately 10% of patients with acute gastrointestinal bleeding and red blood per rectum may have an upper gastrointestinal source for the bleeding (Jensen and Machicado). A nasogastric tube is often passed when an upper gastrointestinal bleeding source is a possibility. However, use of a nasogastric tube to exclude an upper gastrointestinal bleeding site is reliable only when the return is nonbloody and contains bile (bile indicates that duodenal contents are sampled) (Luk et al.; Cuellar et al.). Passing a nasogastric tube is reasonable for a patient with maroon or red stools while the history and physical examination are obtained and resuscitation measures are continuing. This was not done for the patient in Case 13, which led to a delay in diagnosis. Her hyperactive bowel sounds are consistent with an upper gastrointestinal bleeding source. The elevated blood urea nitrogen level and normal serum creatinine value are also consistent with upper gastrointestinal bleeding and are due to the absorption of blood proteins from the gastrointestinal tract. Case 14 A 74-year-old man is evaluated in the emergency department because of two episodes of painless hematochezia in the past 8 hours. He has no fever or other signs of systemic illness. During total colonoscopy 3 months ago, a 5-mm benign adenoma in the ascending colon was removed. The colonoscopic examination was otherwise normal except for the presence of numerous sigmoid diverticula. On physical examination, temperature is 36.8 C (98.2 F), pulse rate is 88/min, respiration rate is 18/min, and blood pressure is 140/76 mm Hg. Examination is normal except for a trace of red blood on the examining glove during digital rectal examination. Hemoglobin is 12.1 g/dL. An electrocardiogram shows normal rhythm and no acute changes. Passage of a nasogastric tube returns nonbloody material that contains bile. The patient is observed in the emergency department and has no further episodes of red blood per rectum. He is admitted to an observation floor for 24 hours, during which time no further bleeding occurs, and he is discharged home. Most episodes of acute lower gastrointestinal bleeding resolve spontaneously. Because this patient apparently stopped bleeding upon reaching the hospital, emergent intervention was not needed. Furthermore, he underwent colonoscopy 3 months ago. It is extremely unlikely that the current episode of hematochezia was due to removal of the polyp since post-polypectomy bleed-

88

Chronic Obscure Gastrointestinal Bleeding

ing generally occurs within 2 weeks of the procedure. Diverticula are therefore most likely causing this patients lower gastrointestinal bleeding. Diverticular bleeding is arterial in nature, is painless, and generally stops spontaneously (McGuire). Most patients do not experience recurrent bleeding. If this patient does have recurrent bleeding and the bleeding site can be identified, segmental colectomy may be indicated.

KEYPOINTS

Chronic Obscure Gastrointestinal Bleeding


What are the most common causes of obscure gastrointestinal bleeding? Does all obscure gastrointestinal bleeding originate from the small intestine? Which studies of the small intestine are most appropriate when searching for a source of obscure bleeding? What new endoscopic technique is available for evaluating patients with small intestinal bleeding?

At times, conventional endoscopic studies (upper endoscopy and colonoscopy) do not reveal the source of gastrointestinal blood loss. Some patients have occult blood in the stool and a slow, steady decrease in the hemoglobin value. Others have discrete episodes of melena or maroon-colored stools. In either case, when upper endoscopy and colonoscopy do not reveal the source of blood loss, the patient may be categorized as having obscure gastrointestinal bleeding. Several factors should be considered. It is often assumed that the bleeding must be originating from the small intestine. In many patients, however, the bleeding site is in the upper gastrointestinal tract or colon and is not recognized by the endoscopist. Some causes of obscure bleeding within reach of the upper endoscope include linear erosions with large hiatal hernias (Cameron lesions), Dieulafoy lesions, gastric antral vascular ectasia, and vascular lesions along the medial wall of the second portion of the duodenum, an area not always well visualized with a standard upper endoscope. In immunosuppressed patients or patients with AIDS, additional upper gastrointestinal tract lesions to consider include idiopathic esophageal ulcers, ulcers due to cytomegalovirus, and Kaposis sarcoma. Colonic causes to consider in these patients include cytomegalovirus colitis and anorectal lesions with excessive bleeding due to associated thrombocytopenia (Chalasani and Wilcox). Because of poor bowel preparation or technical factors, colonoscopy is not always successfully carried out to the cecum. There are many causes of obscure bleeding of the small intestine, including vascular lesions (angiectasias), mass lesions (smooth muscle tumors, primary or metastatic cancers, lymphoma), Meckels diverticulum, and radiation-induced enteropathy. Case 15 A 77-year-old woman is evaluated because of severe anemia and extreme fatigue when walking. She has not had overt signs of gastrointestinal bleeding. Physical examination is normal except for brown stool in the rectum that is positive for occult blood. Hemoglobin is 6.2 g/dL, and serum iron studies show severe iron deficiency. Colonoscopy is normal. Upper endoscopy shows evidence of a large hiatal hernia with multiple erosions at the level of the diaphragm. A radiograph of the small bowel is normal. Although an inexperienced endoscopist may decide that this patient has obscure gastrointestinal bleeding, she is slowly losing blood because of the presence of erosions within a large hiatal hernia. The key to the diagnosis is the presence of erosions at the level of the diaphragm. These erosions, which may come

Patients with acute gastrointestinal bleeding typically present with melena, hematochezia, and/or hematemesis. Although melena and hematemesis usually occur in patients with acute upper gastrointestinal bleeding, melena may also develop in some patients with a bleeding site distal to the ligament of Treitz. Nasogastric aspirate reliably excludes upper gastrointestinal bleeding only when the aspirate contains bile (indicating that the duodenal contents are sampled) but does not contain blood. Emergent upper endoscopy is usually not required in patients with acute gastrointestinal bleeding who are hemodynamically stable and have no evidence of ongoing bleeding. The most common causes of acute lower gastrointestinal bleeding in hemodynamically compromised patients are bleeding diverticula, vascular ectasia, proctopathy due to radiation therapy or colitis, ischemic colitis, and colonic neoplasia. Most episodes of acute lower gastrointestinal bleeding are self-limited, and emergent diagnostic studies are usually required only for patients with persistent bleeding.

McGuire HH Jr. Bleeding colonic diverticula. A reappraisal of natural history and management. Ann Surg. 1994;220:653-6. PMID: 7979613 Chalasani N, Wilcox CM. Gastrointestinal hemorrhage in patients with AIDS. AIDS Patient Care STDS. 1999;13:343-6. PMID: 1084284

89

Chronic Obscure Gastrointestinal Bleeding

Cameron AJ, Higgins JA. Linear gastric erosion. A lesion associated with large diaphragmatic hernia and chronic blood loss anemia. Gastroenterology. 1986;91:338-42. PMID: 3487479

and go, are known as Cameron lesions. They are believed to be caused by mucosal trauma related to movement of the diaphragm, but vascular insufficiency or damage by gastric acidpepsin may also be a factor (Cameron and Higgins). The diagnosis for the patient in Case 15 is one of exclusion. The presence of a hiatal hernia does not rule out colon cancer or a small bowel lesion as a contributing cause for anemia, and colonoscopy and small bowel radiography were therefore appropriate tests. Once the hiatal hernia is identified as a cause of anemia, therapy is generally begun with an antireflux regimen, including a long-term proton pump inhibitor, and oral iron supplementation. Surgical repair should be considered if complications of the hernia may develop (for example, incarceration, dysphagia) or if the anemia does not respond to medical therapy. Nonresponse to medication can often be expected because the erosions are not necessarily acid-related.

Tests for Gastrointestinal Bleeding from the Small Intestine


The clinician must first determine whether the bleeding source is within the colon or the upper gastrointestinal tract and therefore within reach of a standard endoscope. If bleeding is not from these sites, attention is turned to the small intestine, which because of its length cannot be examined using standard endoscopic techniques. The commonly utilized tests in the evaluation of small intestinal bleeding are listed in Table 39. Choice of these tests should be tailored to the individual patient. In a young patient who is otherwise healthy, structural abnormalities of the small intestine (for example, Meckels diverticulum, small bowel tumors, Crohns disease) are more likely than are vascular lesions. A small bowel radiograph or Meckels scan (technetium-99m pertechnetate scan) is therefore reasonable. Conversely, an elderly patient or a patient with renal disease is at greater risk for vascular lesions of the small intestine. Push enteroscopy or capsule endoscopy is reasonable in the initial evaluation of these patients. Although push enteroscopy can typically visualize an area from 30 to 100 cm distal to the ligament of Treitz, a significant amount of the small intestine cannot be seen. Small vascular lesions are often found within reach of
TA B L E 3 9 Comparison of Tests Evaluating the Small Intestine in Patients with Gastrointestinal Bleeding

Test Small bowel radiography Enteroclysis Push enteroscopy Intraoperative endoscopy Capsule endoscopy Meckels scan (technetium-99m pertechnetate scan) Technetium-labeled red blood cell scan Mesenteric arteriography

Advantages Is commonly available Will detect more structural lesions Provides direct endoscopic views Can be used for both biopsy and therapy Visualizes entire small intestine Is easy procedure for patients Visualizes entire small intestine Is commonly available Is commonly available Is associated with low morbidity May visualize structural or vascular lesions even without active bleeding

Disadvantages May miss some structural lesions Will miss vascular lesions Is uncomfortable Will miss vascular lesions Visualizes limited amount of small intestine (approximately 30100 cm of jejunum) Associated with morbidity and mortality Cannot provide therapy Operator cannot control generated images Is specific for Meckels diverticulum Sensitivity is only 62%81%* May not locate bleeding site precisely May not detect slow or intermittent bleeding Associated with morbidity (e.g., contrastinduced renal failure, emboli) Is often normal in absence of bleeding

* Data from Lin S, Suhocki PV, Ludwig KA, Shetzline MA. Gastrointestinal bleeding in adult patients with Meckels diverticulum: the role of technetium 99m pertechnetate scan. South Med J. 2002;95:1338-41 and Kong MS, Chen CY, Tzen KY, Huang MJ, Wang KL, Lin JN. Technetium-99m pertechnetate scan for ectopic gastric mucosa in children with gastrointestinal bleeding. J Formos Med Assoc. 1993;92:717-20.

90

Chronic Obscure Gastrointestinal Bleeding

the enteroscope and are coagulated at the time of the procedure (Foutch et al.). Capsule endoscopy is a new technique in which the patient swallows a small capsule that takes frequent pictures as it passes through the small intestine and transmits the photographs to a receiver. The clinician can review these images and search for one or more bleeding lesions (Scapa et al.). Case 16 A 24-year-old woman is evaluated after having had three discrete episodes of maroon-colored stools. She has not had abdominal pain and has not required blood transfusions. Upper endoscopy and colonoscopy with intubation of the terminal ileum were negative. A Meckels scan (technetium-99m pertechnetate scan) is positive, and the patient subsequently undergoes surgery for bleeding associated with a Meckels diverticulum. This patients age determines the type of the small bowel evaluation. Because she is young, bleeding related to a Meckels diverticulum is likely, and the appropriate test is ordered once upper endoscopy and colonoscopy are negative. Because the sensitivity of the technetium-99m pertechnetate scan is only 62% to 81%, a negative scan does not rule out a Meckels diverticulum if the clinical features are compatible with this diagnosis (Lin et al.; Kong et al.). If this patient were older, a vascular lesion might be causing the bleeding. If a vascular lesion is likely, endoscopy (push enteroscopy or capsule endoscopy) would be a reasonable choice. For a patient still at risk for other structural abnormalities (for example, small bowel tumor, Crohns disease), an endoscopic approach is reasonable, but small bowel radiographs also have an acceptable diagnostic yield.

Foutch PG, Sawyer R, Sanowski RA. Push-enteroscopy for diagnosis of patients with gastrointestinal bleeding of obscure origin. Gastrointest Endosc. 1990;36:337-41. PMID: 2210273 Scapa E, Jacob H, Lewkowicz S, Migdal M, Gat D, Gluckhovski A, Gutmann N, Fireman Z. Initial experience of wirelesscapsule endoscopy for evaluating occult gastrointestinal bleeding and suspected small bowel pathology. Am J Gastroenterol. 2002;97:2776-9. PMID: 12425547 Lin S, Suhocki PV, Ludwig KA, Shetzline MA. Gastrointestinal bleeding in adult patients with Meckels diverticulm: the role of technetium 99m pertechnetate scan. South Med J. 2002;95:1338-41. PMID: 12540005 Kong MS, Chen CY, Tzen KY, Huang MJ, Wang KL, Lin JN. Technetium-99m pertechnetate scan for ectopic gastric mucosa in children with gastrointestinal bleeding. J Formos Med Assoc. 1993;92:717-20. PMID: 7904846
KEYPOINTS

In many patients with obscure gastrointestinal bleeding, the bleeding site is in the upper gastrointestinal tract or colon rather than the small intestine. Common causes of obscure bleeding from the small intestine include mass lesions, vascular lesions, Meckels diverticulum, and radiation-induced enteropathy. Capsule endoscopy is a new procedure for evaluation of the small intestine.

91

Das könnte Ihnen auch gefallen